set 7

Réussis tes devoirs et examens dès maintenant avec Quizwiz!

Mediators of inflammation thar are released by mast cells in the early phase of inflammation include a) histamine b) GM-CSF c) leucotriens d) a and b are correct e) a b and c are correct

A

Naive lymphocytes enter lymph node trough? a) artery b) afferent lympathic vessel c) efferent lympathic vessel d) artery or afferent lymphatic vessel e) artery or efferent lymphatic vessel

A

Name circulating cells with cytoplasmic granules that produce heparin, histamine and other mediators involved in inflammation. a) basophil b) B cell c) mast cell d) macrophage e) dendritic cell

A

The effector molecules produced by Tc cells a) perforins b) C9 c)prostaglandins d) a and b are correct e) lack of correct answers

A

The isotype of Ig is determined by a) the constant part of the heavy chain b) the light chain c) the Fab fragment d) the CDR regions e) lack of correct answer

A

Thymic nurse cells A. are present in the cortex of the thymus B. secrete IL-7 C. express notch-1 receptor D. A and B are correct E. B and C are correct.

A

Which cytokine synthesized by activated mast cells (as well as Th2 cells) promotes eosinophilia and eosinophilic inflammation? a) Il-5 b) tumor necrosis factor alfa c) interferon alfa d) transforming growth factor beta e) lack of correct answer

A

Which of the following proteins is present in azurophilic granules of neutrophils? a) myeloperoxidase b) calprotectin c) histone d) NADPH oxidase e) a, b, c and d are correct

A

Recombination signal sequence a) are recognized by RAG proteins b) code for protein c) form coding joint during somatic recombination d) a and b are correct e) a-c are correct

A (A: Recombination signal sequences are conserved sequences of *noncoding* DNA. C: form signal joint. )

Reticulate body of chlamydia pneumonia a) can replicate in infected cells b) can infect cells c) ist he same as elementary body d) answers a and b are correct e) lack of correct answer

A (B: elementary body can infect cells.)

MHC class I molecule: a) is constitutively expressed by every cell but erythrocytes b) presents long peptides c) interacts with CD4 co-receptor by β1 domain d) interacts with CD8 co-receptor by β1 domain e) is a homodimer f) presents exogenous antigens, cleaved in proteasome

A (B: presents short peptides. D: interacts with CD8 on alpha 3 domain. E: heterodimer)

.β2-microglobulin a) is a component of MHC class I b) is a component of MHC class II c) is highly polymorphic d) answers a and c are correct e) answers b and c are

A (C: its non-molymorphic)

The production of IFN-y is stimulated by a) IL15 b) TGF c) IL10 d) answers a and b c are correct e) lack of correct answer

A (IFN-gamma is released by e.g Th1 cells, Tc, macrophages and *NK cells*. NK cells are stimulated by *IL-15* B+C: TGF and IL-10 inhibits T cells aka release of IFN.)

The MHC class I loci is/are a) HLA-A b)HLA-DR c) β2-microglobulin d) HDL-DP e) a and c are correct

A (MHC I LOCI: HLA-A HLA-B HLA-C All of the come from chromosome 6. B: MHC II loci C: this comes from chr. 15)

Mature B cell a) may enter lymph node b) undergoes negative selection in bone marrow c) has only IgM on their surface d) answers a and b are correct e) answers a and c are correct

A (c: IgM and IgD.)

Viruses can inhibit the immunse response by a) inhibition of complement mediated pathways b) upregulating the expression of MHC class II molecules c) increasing the expression of adhesion molecules on infected cells d) answers a and c are correct e) answers a and b and c are correct

A (?)

Cells of the Myeloid lineage, especially those containing cytoplasmic granules include what 3? A.) Eosinophils, Basophils, and Neutrophils B.) Macrophages, Mast Cells, and Neutrophils C.)Macrophages, Mast Cells, Eosinophils D.)Macrophages, Mast Cells, and Lymphocytes

A.) Eosinophils, Basophils, and Neutrophils

What are released by some commensal microbes found on skin that inhibit growth of some bacteria? A.) Fatty acids B.) Protein C.) Neutrophils D.) Mast cells

A.) Fatty acids

Which is a common pathogen of the stomach? A.) H. pylori B.) streptococcus C.) Staphylococcus D.) Bacteriodes

A.) H. pylori

IgG class is determined by what type of isotype, heavy chain or light chain? A.) Heavy B.) Light

A.) Heavy

Which disease state is Cl deficiency in? A.) Hereditary Angioedema B.) Neisseria bacteremia C.) Anaphylaxis D.) Severe recurrent infections

A.) Hereditary Angioedema

The epitope binds to what Ig(s) to trigger a conformational change in the "tail" of the Fc portion of the antibody. A.) IgA, IgG, or IgM B.) IgA only C.) IgG only D.) IgM only E.) None of the above

A.) IgA, IgG, or IgM

Which Ig is present in low serum concentration and absorbed into the surface mast cells, monocytes, and eosinophils? A.) IgE B.) IgD C.) IgG D.) Ig A

A.) IgE

In_______ immunity the body responds to antigen the same way even after repeat exposures, whereas in ________ immunity the body has increased effectiveness/response after repeat exposures. A.) Innate, Adaptive B.) Adaptive, Innate

A.) Innate, Adaptive

Which of the following is a function of Toll Receptor Cells? A.) They are flaggers that recognize foreign molecules B.) Active in phagocytosis of molecules C.) They are the portion of the cell that engulfs the microbe D.) None of the above

A.) They are flaggers that recognize foreign molecules

Which of the following is a correct pairing of a soluble molecule with its mircocidal action in the respiratory tract? A.)beta-defensins increase microbial susceptibility to phagocytosis B.)DNAse enzymatically damages microbial membranes C.)Fatty acids of commensal microbes stops peptidoglycan D.)Lacrimal secretions facilitate ingestion of microbes by phagocytosis E.)Lysozyme degrades DNA and RNA produced by pathogenic microbes

A.)beta-defensins increase microbial susceptibility to phagocytosis

T cells can express the following surface molecules, EXCEPT: A.immunoglobulin B.CD3 C.CD4 D.CD8 E.all of the above F.none of the above

A.immunoglobulin

Thymic dendritic cells play an important role by expressing tissue specific proteins (antigens) that are unique to different tissues/organs in the body. (slide 149) a. true b. false

b. false *MEDULLARY THYMIC EPITHELIAL cells (MTEC) play an important role by expressing tissue specific proteins (antigens) that are unique to different tissues/organs in the body.

Tolerance to self or negative selection is responsible for about 95% of the double positive (DP) T cell deaths. (slide 141) a. true b. false

b. false *MHC restriction or positive selection is responsible for about 95% of the double positive (DP) T cell deaths. Negative selection is responsible for about 2-5% of the DP T cell deaths.

Once an invader has been vanquished, about 75% of the T cells which responded to the attack usually die off. (slide 113) a. true b. false

b. false *Once an invader has been vanquished, more than 90% of the T cells which responded to the attack usually die off.

T cells can recognize both soluble molecules and peptides presented by MHC I or MHC II molecules. (slide 13) a. true b. false

b. false *T cells ONLY recognize peptides presented by MHC I or MHC II molecules.

T cells are double positive (DP) cells before entering and maturing in the thymus. (slide 139) a. true b. false

b. false *T cells lack TCRs, CD4s, and CD8s when they are created in bone marrow. Once IN the thymus, they mature into double positive (DP) cells expressing both CD4 and CD8.

T cells that are nude are the most common lymphocyte in the thymus (80%). (slide 139) a. true b. false

b. false *T cells that are double positive (DP) cells are the most common lymphocyte in the thymus (80%). Also nude comprise about 5% of the maturation cascade of T cells in the thymus.

Medullary thymic epithelial cells (MTEC) check to see if T cells recognize any of the self peptides displayed by MHC molecules on these cell's surface. (slide 148) a. true b. false

b. false *THYMIC DENDRITIC cells check to see if T cells recognize any of the self peptides displayed by MHC molecules on these cell's surface.

The connection between the receptor and the nucleus is strong in naïve T cells. (slide 23) a. true b. false

b. false *The connection between the receptor and the nucleus is WEAK in naïve T cells.

iTreg are thought to require direct cell to cell contact and are antigen specific. (slide 122) a. true b. false

b. false *nTreg are thought to require direct cell to cell contact and are antigen specific.

The combination of co-stimulation molecules will only depend upon the pathogen. (slide 22) a. true b. false

b. false *The combination of co-stimulation molecules will depend upon the pathogen and the area of the body (regional identity).

Cytokines that activate macrophages for antibody production is/are a)IFN-y produced by Th1 cells b) IL4 produced by Th2 cells c) IFN-y prduced by TH2 cells d) answers a and c are correct e) lack of correct answer

B

Enzymatic degradation of the endocytosed proteins occurs in a) proteasome b) endosomal/lysosomal compartments c) ER d) large endosomal proteins are degraded in proteasome and small proteins are degraded in endosomal/lysosomal compartments e) lack of correct answer

B

Lytic granules of cytotoxic lymphocytes contain? a) IL-2 b) perforin c) FAS d) a and b are correct e) a, b and c are correct

B

Mutation of which of the following genes will block the development stage from "stem cell to B or T lineage precursor (lymphoid lineage)"? a) Pax-5 b) Ikaros c) CD25 d) CD3 e) Notch- 1

B

Negative selection a) is mediated by epithelial cells of thymic cortex b) is mediated by dentritic cells of thymic medulla c) applies to double negative thymocytes d) answers a and c are correct e) answers b and c are correct

B

Non-acute, persistent slow infection is caused by a) Epstein Barr virus b) HIV c) influenza virus d) answers a and b and c correct e) lack of correct

B

Phenotype of NK cells a) TCR+CD16+ b)TCR-CD16+ c)TCR-CD16- d)TCR-CD19+ e) TCR-CD19+

B

TH1 cells make macrophage sensitive to interferon gamma by expressing a) CD40 b) CD40L c) FAS d) FASL e) answers b and d are correct

B

The first antibody produced during the immune response by B cells is a) IgA b) IgM c) IgD d) IgG1 e) IgG2

B

The function of IgE includes a) neutralization of bacterial toxins at mucus membrnes b) enhancement of phagocytosis c) removal of immune complexes from circulation d) anwers a b and c are correct e) answers a and b and c are not correct

B

Which of the following is transmembrane glycoprotein that serves as a CD-receptor for the T cell receptor (TCR) and is also known as a helper T cell (Th) marker? A. CD3 B. CD4 C. CD8 D. CD28 E. CD14

B

Which structure is required for polymerization of IgM pentamers a) poly-iG receptor b) J chain c) secretory component d) Fc fragment e) Fab fragment

B

X translocates a peptide that binds to MHC class II molecule X is a) TAP b) HLA-DM c) proteasome d) calnexin e) invariant chain

B

All of the following are characteristics of both MHC class I and class II molecules except: A. they are expressed codominantly B. they are expressed constitutively on all nucleated cells C. they are involved in presentation of antigen D. they are expressed on the surface membrane of B cells E. B and D are correct.

B (B: this is correct for MHC I, not MHC II.)

Follicular dendritic cells a) can cause apoptosis of naive T cells b) have C3b receptors on their surface c) express MHC II d) answers a b and c are correct e) answers b and c

B (a: apoptosis of B cells. c: they do not express MHC molecules. )

MHC class I a) is composed of two chains which span the cell membrane b) binds short peptides (8-9 aminoacids) c) binds peptides derived from exogenous antigens d) answers a, b and c are correct e) answers a and c are correct

B (A: MHC II has 2 chains C: binds endogenous antigens.)

The production of IFNy is stimulated by a) TGF b) IL18 c) IL10 d) answers a b and c are correct e) lack of correct answer

B (After stimulation with IL-18, natural killer (NK) cells and certain T cells release another important cytokine called interferon-γ (IFN-γ))

Active immunity a)is conferred by adoptive transfer of antibodes or T lymphocytes specific for the microbe b)generates memory cells c)is the transfer of maternal antibodies to the fetus d)answers a and b are correct e)answers a, b and c are correct

B (a: passive immunity c: passive immunity)

What are the names of molecules attached to the surface of microbes, making them more attractive to phagocytic cells, thus facilitating microbe destruction? A.) Receptors B.) Opsonins C.) Proteins D.) Enzymes

B.) Opsonins

Which of the following is a common pathogen of the skin? A.) Niesseria B.) Staphylococcus C.) Bacteriodes D.) H.pylori

B.) Staphylococcus

What are the two isoforms of IgA? A.) alpha a and alpha b B.) alpha 1 and alpha 2 C.) alpha 1 and beta 1 D.) none of the above

B.) alpha 1 and alpha 2

What are the 5 types of heavy chains encoded on chromosome 12? A.) alpha, beta, pi, delta and mu B.) alpha, delta, epsilon, gamma, and mu C.) Il-4, Il-3, mu, alpha, cd4 D.) None of the above

B.) alpha, delta, epsilon, gamma, and mu

What are the functions of the CD3 complex? A.) to opsonize bacteria B.) to support the TCR and to transduce transmembrane signaling when the TCR is engaged C.) to produce t and b cells D.) all of the above E.) none of the above

B.) to support the TCR and to transduce transmembrane signaling when the TCR is engaged

30 year old female developed vaginal candidiasis after receiving antibiotic therapy for a sinus infection. One possible explanation for the fungal infection is antibiotic-induced reduction in vaginal A.) lysozyme secretion B.) mucus secretion C.) normal commensal bacteria D.) pH E.) RNAses and DNAses

C.) normal commensal bacteria

Light Chains are of 2 types: A.) Alpha and Delta B.) Epsilon and Mu C.)Kappa and Gamma D.)Kappa and Alpha

C.)Kappa and Gamma

Down-regulation of the immunity signaling pathway tend to involve the following mechanisms A.Receptor internalization and degradation B.Induction of negative regulators C.All of the above D.None of the above

C.All of the above

The principal function of the immune system is: A.Defense against cancer B.Repair of injured tissues C.Defense against microbial infections D.Prevention of inflammatory diseases E.Protection against environmental toxins

C.Defense against microbial infections

Fragments of a cytoplasmic pathogen are presented to T cells by a) direct engagement of cell surface pattern recognition receptors b) macropinocytosis into ynånting T cells c) MHC class I molecules to CD8+ T cells d) phagocytosis and presentation to CD4+ T cells e) lack of correct answer

c

Human RAG-1 deficiency is caused by an autosomal recessive mutation that destroys the recombinase activating genes. This results in a lack of ability to produce diverse: a. immunoglobulins b. the T-cell receptors (TcR) c. both d. neither

c

If a person was born without any functional Toll-like receptors, which of the following symptoms would they be likely to exhibit? a. increased bacterial infections b. increased virus infections c. increases in both bacterial and virus infections d. an absence of mature B cells.

c

IgD: (choose one) a. is transferred across the placenta b. exists in a divalent form in mucosal epithelial tissues c. is mainly membrane bound (not secreted) d. is the predominant Ig in primary responses e. is involved in allergic responses

c

IgM is particularly efficient at fixing complement because _____. a. it has an extra CH domain. b. it is made first in an immune response and therefore has first access to C1q. c. it has five binding sites for C1q. d. it has easy access to extravascular areas.

c

In terms of V, D and J segment arrangement, the T-cell receptor α-chain locus resembles the immunoglobulin _______ locus, while the T-cell receptor β chain locus resembles the immunoglobulin _______ locus: a. λ light chain; κ light chain b. heavy chain; light chain c. light chain; heavy chain d. δ chain; heavy chain e. κ light chain; λ light chain.

c

In which of the following tissues is IgA2 produced at approximately twice the level as IgA1? a. spleen b. mammary glands c. large intestine (colon) d. nasal mucosa e. tear ducts

c

Inflammatory cytokines cause what two events in local blood vessels: a) decreased blood flow and decreased permeability b) decreased blood flow and increased permeability c) increased blood flow and increased permeability d) none of the above

c

Neutrophils leave the blood at sites of infection because endothelial cells in these locations express: a. antibody and complement b. CD4 and MHC II c. selectin and cell adhesion molecules d. TLR and CD3.

c

Phagocytosis remove apoptotic cells in process called a) necrosis b) netosis c) efferocytosis d) pyroptosis e) lack of correct answer

c

Primary lymphoid tissues are sites where lymphocytes _____, whereas secondary lymphoid tissues are the sites where lymphocytes _____. a. are stimulated; develop and mature b. encounter pathogens; undergo apoptosis c. develop and mature; become stimulated d. undergo clonal selection; differentiate from hematopoietic stem cells e. die; are phagocytosed after death

c

SC component of IgA a) is protein synthesized by a plasma cell b) is present in monomeric IgA c) is present in some dimeric IgA d) is a J chain e) lack of correct answer

c

Specific antibodies are readily detectable in serum following primary contact with antigen after: a. 10 minutes b. 1 hour c. 5-7 days d. 3-5 weeks

c

Switching between making IgM and IgD and switching between making membrane bound IgM and secreted IgM are similar because both involve: a. gene conversion b. DNA loop excision c. RNA splicing d. post-translational processing.

c

The Brambell receptor (FcRn) allows _____ to reach the _____. a. IgM, thymus b. IgA, mucosal epithelial tissues c. IgG, fetus d. IgE, the skin

c

The T-cell co-receptor CD4 interacts with _______ bound to the surface of _______: a. MHC class I; antigen-presenting cells b. MHC class I; T cells c. MHC class II: antigen-presenting cells d. MHC class II: T cells e. none of the above.

c

The alternative complement pathway is initiated when _______. a) C1q binds to the mannose binding lectin b) C4 is cleaved by C1 esterase c) C3 is hydrolyzed by water d) Properdin is stabilized by Factor B

c

Genes expressed in both B cell and T cell precursors during development would include: a. CD3, CD4 and CD8 b. Ig-alpha, Ig-beta, and Syk c. RAG-1, RAG-2 and cytodine deaminase d. RAG-1, RAG-2, and TdT

d

During T lymphocytes maturation: a) VJ genes rearrangement occurs at the beginning b) preTα chain stimulates β chain rearrangements c) cells do not proliferate until possessing complete TCR d) cells become MHC-restricted during positive selection e) cells undergo antigen-dependent somatic recombination f) double positive cells lose one type of co-receptor during β selection.

D

Extracellular microbes: a) are neutralized antibodies b) are killed by cytotoxic T cells c) are neutralized by cytokines produced by helper T cells d) are killed by macrophages e) none of the above

D

In naive T cell recirculation, entry of the cells into peripheral lymph nodes occurs through which specialized vessels? a) efferent lymphatic b) thoracic duct c) central artery d) high endothelial venule e) sinusoid

D

Mediators of inflammation that are produced in the late phase of inflammation by mast cells include a) prostaglandins b) proteoglycans c) leucotrines d) a and c are correct e) a, b and c are correct

D

Passive immunity a)is conferred by adoptive transfer of antibodes or T lymphocytes specific for the microbe b)generates memory cells c)is the transfer of maternal antibodies to the fetus d)answers a and c are correct e)answers a, b and c are correct

D

Process of somatic recombination applies to: a) TCR b) BCR c) macrophages d) a and b are correct e) a, b, and c are not correct

D

T helper cells a) can activate T cytotoxic cells b) are CD4+ c) can secrete cytokines d) answers a, b and c are correct e) answers b and c are correct

D

The costimulatory receptor of T cells a) recognizes parts of the MHC peptide complex on APC b) the example is CD4 and CD8 c) takes part in antigen presentation d) binds to B7 molecules on APC e) answers a b and d are correct

D

The first cells that come to the site of infection are a)macrophages b)lymphocytes c)NK cells d)neutrophils e)B cells

D

The lytic granules of Tc and NK cells a) contain granzymes and granulizyne b) move to the site of cell contact after Tc recognizes infected cell c) Nk cells do not contain lytic granules d) answers a and b are correct e) answers a ad b and c are correct

D

The production of the receptors by T and B cells: A. depends on the antigen presence B. occurs in generative lymphoid organ C. requires the activity of RAG-1 and RAG-2 D. answers B and C are correct E. answers A, B and C are correct

D

Immunological tolerance in the B cell repertoire is called _____ when it develops in the primary lymphoid organs, and ______ tolerance when it is induced outside of the bone marrow. a. primary, secondary b. apoptotic, anergic c. stromal, follicular d. central, peripheral

d

The Fc or constant crystallized fragment is produced by cleavage of the above (answer to 100) and contains what portions? A.) CH2 only B.) CH3 only C.) CH4 only D.) CH2, CH3, CH4

D.) CH2, CH3, CH4

What term is used for the collective system where enzymes and proteins, that function in both the innate and adaptive branches of the immune system as a soluble means of protection, against pathogens that evade cellular contact? A.) Fixing B.) Agglutination C.) Phagocytosis D.) Complement

D.) Complement

Which interferon increases MHC I and II expression in antigen (Ag) present in all cells? A.) IFN-Mu B.) IFN- Alpha C.) IFN-Beta D.) IFN-Gamma

D.) IFN-Gamma

Epitope binding before Fc receptor engagement is not required for: A.) Carrier molecules B.) Hapten-carrier conjugates C.) Haptens D.) IgE E.) IgG

D.) IgE

Which Ig is the most abundant in plasma? A.) IgA B.) IgM C.) IgD D.) IgG

D.) IgG

Mature B-Cells have one isotype with the exception that unstimulated B cells express which 2 Ig's? A.) IgM and IgA B.) IgA and IgD C.) IgM and IgG D.) IgM and IgD

D.) IgM and IgD

Secretions of the lacrimal glands contain what which help protect the eye A.) Amylase B.) Bicarbonate C.) Carboxylase D.) Lysozymes

D.) Lysozymes

What are "common receptors" called, where there are about 100, and are part of the innate immune system? A.) Natural Killer Cells B.) Recognition cells C.) Mucous Cells D.) Pattern Recognition Receptors (PRR's)

D.) Pattern Recognition Receptors (PRR's)

Process of somatic hypermutations a) always leads to improved affinity of Ig to antigen b) occurs in lymph node IgM+IgD+Bcells c) occurs in bone marrow IgM+IgD+B cells d) answers a and b are correct e) lack of correct answer

E (IgV)

In order to complete maturation, B cells migrate from the bone marrow to the _____ of ______. a. cortex, the thymus b. marginal zone, thoracic ducts c. red pulp, the spleen d. follicles, lymph nodes

d

Antigen binding groove in MHC class I molecules: a) binds lipids b) is built of α1 and β1 domains c) is formed by α chain d) presents peptide antigens e) c and d are correct

E (a: binds peptides b: alpha1 + alpha2)

In order to complete maturation, B cells migrate from the bone marrow to the ________ of _______ a. cortex, the thymus b. marginal zone, thoracic ducts c. red pulp, the spleen d. follicles, lymph nodes.

d

CD40L- CD40 interaction a) inhibits the expression of B7 on B cell b) stimulates the expression of B7 on B cell c) inhibits the expression of B7 on T cell d) stimulates the expression of B7 on T cells e) lack of correct answer

E (stimulates expression of B7 on dendritic cells and macrophages)

In order to complete maturation, B cells migrate from the bone marrow to the ________ of _______ . a. cortex, the thymus b. marginal zone, thoracic ducts c. red pulp, the spleen d. follicles, lymph nodes.

d

In those with the disease, leukocyte adhesion deficiency (LAD), neutrophils are not able to fight infection because: a. They cannot generate toxic oxygen compounds b. They are incapable of phagocytosis c. They express no Fc receptors d. They cannot get to the site of infection

d

Passive immunity a) is conferred by adaptive transfer of antibodies or T lymphocytes specific for the microbe b) generates memory cells c) protects from the infection d) a and c are correct e) a-c are correct

d

Secretory IgA and _____ can bind to the poly-Ig receptor and be transported into the lumen of the gut or across other mucosal surfaces. a. IgG b. IgE c. IgD d. IgM

d

IgA a) are found in blood circulation b) cn cross thourgh placenta c) can cause mast cell degranulation d) answers a b and c are correct e) lack of correct answer

E ((Iga is found in mucos, IgG crosses placenta, C3a C5a causes mast cell degranulation))

The AM in ITAM involved in lymphocyte activation stands for: a. antigen modulation b. attenuation module c. antibody motif d. activation motif

d

The function of negative selection of thymocytes in the thymus is to eliminate: a. single-positive thymocytes b. double-positive thymocytes c. alloreactive thymocytes d. autoreactive thymocytes e. apoptotic thymocytes

d

The hypervariable or complementarity determining regions (CDR): a. are restricted to light chains b. are in the constant part of the Ig molecule c. bind to Fc receptors d. are involved in antigen binding e. occur at the c terminal end of the Ig peptide chains

d

The major function of immature dendritic cells is to: a. secrete antibody b. activate neutrophils c. present antigen d. take up antigen

d

The membrane attack complex consists of: a. OH b. Colicins c. C3b3b, Bb d. C5b,6,7,8,9 e. Properdin

d

The role of the invariant chain in clas II MHC complexes is: a) to display self proteins on the surface of APCs b) to target the MHC complex to the golgi c) to procide targeting to lysosomes and a substrate for lysosomal enzymes d) to prevent the MHC from binding an endogenous peptide before it enconters an antigens fragment e) to process proteins into small peptide

d

Type IV hypersensitivity is also known as delayed type hypersensitivity. The elicitation phase involves: a. B cell proliferation b. The release of pre-formed mediators c. The lysis of mast cells d. Macrophage activation and cytotoxic T cell generation

d

Tyrosine residues present on the cytoplasmic portion of proteins that can be phosphorylated by protein kinases in the activation of B cells and T cells are called: a. tDTs b. STATs c. BRDs d. ITAMs e. JAKs.

d

What is the function of the enzyme TdT? a. class switching b. membrane to secreted switching c. the loss of CD4 or CD8 d. the random insertion of nucleotides.

d

What is the primary purpose of inflammation? a. to help doctors make their car payments b. to resolve immune responses once the foreign invader is eliminated c. to promote VDJ recombination in B cell precursors d. To get immune components to the site of infection

d

What types of hypersensitivity could occur in a person who did not have any functional T cells? a. Type I b. Types II and III c. Type IV d. None of the types of hypersensitivity could occur

d

Which of the following is correct regarding tumor specific antigens TSAs a) are also present in high concentration on normal somatic cells b) often found on normal fetal cells as well as on tumor cells c) belong to tissue specific differentiation antigens d) result from point mutation in self protein e) lack of correct answer

d

Which of the following is not a characteristic of septic shock? a. organ failure b. decreased blood pressure c. compromised blood supply to vital organs d. requires complement activation

d

Which of the following is the first stage of T-cell receptor gene rearrangement in α:β T cells? a. Vα → Dα b. Dα → Jα c. Vβ → Dβ d. Dβ → Jβ e. Vα → Jα.

d

Which of the following is the membrane-bound form of C3 convertase of the alternative pathway of complement activation? a. iC3 b. C3a c. C3b d. C3bBb.

d

Which of the following is the predominant route by which pathogen-associated antigens are brought from a site of infection into a lymph node? a. efferent lymphatics b. artery c. vein d. afferent lymphatics

d

Which of the following molecules is expressed on the surface of activated Th1 cells and promotes macrophage activation? a) interferon-y b) IL-10 c) IL-12 d) CD40L e) CD40

d

Which of the following pairs is mismatched? a. T-cell activation: cell division and differentiation b. effector B cell: plasma cell c. plasma cell: antibody secretion d. helper T cell: kills pathogen-infected cells

d

Which of the following plays a role in changing the antigen binding site of a B cell after antigenic stimulation a) junctional diversity b) combinatorial diversity c) germ line diversity d) somatic hypermutation e) lack of correct answer

d

Which of these pairs are mismatched? a. cytosol: intracellular pathogen b. surface of epithelium: extracellular pathogen c. nucleus: intracellular pathogen d. lymph: intracellular pathogen

d

Which of the following is an example of G protein-coupled receptor (GPCR)? A) T cell antigen receptor (TCR) B) The B cell antigen receptor (BCR) C) CD4 D) Interlukin-2 (IL-2)receptor E) Chemokine receptor CCR7

E) Chemokine receptor CCR7

Which of the following is likely to be accepted? A) allograft B) autograft C) syngeneic graft D) graft between identical twins E) b., c., and d. are likely to be accepted.

E) b., c., and d. are likely to be accepted.

Cell surface markers involved in cell migration include: A. Integrins B. Selectins C. ICAMs D. VCAMs E. All of the above

E. All of the above

Examples of PAMPs include: A. ssRNA B. Pilin C. Flagellin D. LPS E. All of the above

E. All of the above

Functions of the Immune response include: A. Sensing and recognition of foreign and self products B. Targeted killing and removal of microbes C. Selective neutralization and avoidance of pathogenic microbes D. Tolerance to selfe. E. All of the above

E. All of the above

What are the common symptoms of inflammation? A. Rubor (redness) B. Calor (warmth) C. Tumor (swelling) D. Dolor (pain) E. All of the above

E. All of the above

Which one activates classical complement?

IgM and IgG

Which of the following statements about immune responses to tumors is true?

Immunodeficient individuals are more likely to develop certain forms of cancer than are immunocompetent individuals

Large organic molecules that are proteins or large polysaccharides and, less commonly lipids are which of the following?

Immunogens Antigens

Calcineurin inhibitors are commonly used immunosuppressive drugs in organ as well as hematopoietic stem cell transplantation (HSCT). However, inhibiting T-cell signaling might not only be beneficial for immune reconstitution after HSCT, explain why.

Impaired T-cell signaling results in impaired selection in the thymus.

Which one is a pre-analytical error?

Incorrect labeling of specimens

Which of the following is not a postanalytical error?

Incorrect test request

What can decrease complement levels?

Infections

Hypocomplementia can be caused by all except:

Inflammatory conditions, trauma, or acute illness

Standard precautions dictate that you treat all specimens as if they contaminated with all except:

Influenza

A family of proteins produced rapidly by many cells in response to viral infection is which of the following? Passive immunity Interferon Active immunity All of the above

Interferon

-passive immunity

Which immunology concepts best describes the following situation: While on vacation in Australia, Dr. Sarkar goes on a walk about and is bitten by a poisonous Tiger snake. He fully recovered as he received prompt medical attention. He was infused with tiger snake anti-venom which consists of a mixture of antibodies against the venom

D

Which is not a chemical barrier of the skin involved in innate immunity A: Sebum B: Sweat C: Cerumen D: Saliva E: all of them

B cell and cytotoxic T cell

Which of the following cell types are derived from lymphoid progenitor cells? -B cell - neutrophil -red blood cells cytotoxic T cell -macrophage

T-cells

Which of the following cell types require MHCII- mediated antigen presentation to become activated? -natural killer cells - B cells - T cells - all of the above

E

Which of the following is a symptom of inflammation? A: Rubor B: Dolor C: Calor D: Tumor E: All

C

Which of the following is not part of innate immunity? A: Phagocytes B: PMNs C: CTLs D: Eosinophils E: Mast cells

Foreigness

Which of the following is true of antibody or B cell epitopes? A: Usually polar B: Often consist of sequential amino acids C: often consist of non-sequential amino acids D: Usually from the surface of antigen molecule E: All

E

Which of the following is true of antibody or T cell epitopes? A: usually polar B: often consists of sequential amino acids C: usually from the inferior of an antigen molecule D: usually from the exterior of an antigen molecule E: All

Neutrophil

Which phagocyte is present in greatest concentration in the blood?

C4

Which protein does not participate in the alternative complement cascade?

Factor D

Which protein does not participate in the classical complement pathway?

!!!Decreased synthesis of Ig could be associated with: a) disturbance in number of activated T cells b) retarded phagocytosis c) overproduction of B cells d) disturbed activation of complement e) lack of correct answer(lack of clue on PD)

d (Jag skulle säga D pga IgM and IgG are needed to activate complement in the classical way)

Most antigens are carried to lymph nodes: a) in the lymph fluid associated with dendritic cells b) free in the bloodstream c) in the bile associated with bile pigments d) in the bloodstream associated with neutrophils

a

Protective antibodies generated in response to influenza virus bind to _____ of the viral envelope. a. hemagglutinin and neuraminidase b. polysaccharides c. variable surface glycoproteins d. superantigens

a

Which type of hypersensitivity is correctly matched with its cause? a. type I: IgE-mediated b. type II: effector T cells c. type III: histamine d. type IV: soluble immune complexes

a

Which of the following are associated with soluble antigen? a. type I hypersensitivity b. type II hypersensitivity c. type III hypersensitivity d. type IV hypersensitivity e. mast-cell activation.

a or c

The immune response is often made up of activation and inhibition signals in order to start and stop reactions to pathogens. How is this done with regards to: a) T cell responses and b) NK cell responses?

a) Activation of T cells require cytokines, TCR interaction, CD28 - B7 Inhibition CTLA-4 - B7, PD1 - PDL1, cytokines like IL-10, TGF-beta b) NK activation by NKG2D, KIR-L, DNAM-1, NCRs NKp30, NKp44, NKp46, cytokines IL-12, - 15, -18, -2 NK inhibition KIR-S, CD94/NKG2A, Ly-49A, C, I, G2 (mouse), cytokines TGF-beta, IL-10

!!!!Which is not in favour of tumor growth? a) Internalization of MHC I b) lowered tryptophan in tumor microenviroment c) secretion of TGF- by tumor cells d) increased expression of Fas on tumor cells e) combination of two(we dont remeber;/) is correct

d (?)

Langerhans cells a) are dendritic cells of the skin b) up-regulate B7 expression while maturing c) are macrophages d) a and b are correct e) a and c are correct

d (?)

Which of the following may provide some protection against development of allergies and asthma? (slide ?) a. Get a dog b. Clean your house c. Move to the city d. Have only one child

a. Get a dog *not in the notes, but he also said the more siblings you have and the younger you are will increase your likelihood against the development of allergies and asthma

Which antibody does not stimulate inflammation and does not fix compliment? (slide 117) a. IgA b. IgE c. IgG d. IgM

a. IgA *macrophages and IgA both act with restraint beneath the intestinal wall in order to avoid overreaction

Which cell can continuously produce modest amounts of antibodies and provide life-long antibody protection? (slide 192) a. Long-lived plasma cell b. Short-live plasma cell c. Activated B cell d. Central memory T cell

a. Long-lived plasma cell *there is no T cell equivalent to a long-lived plasma B cell

Where are macrophages likely to hang out in a lymph node? (slide 70) a. Marginal sinus b. Cortex c. Paracortex d. Medullary sinus

a. Marginal sinus *also known as the subcapsular sinus

Where are macrophages loaded in the spleen? (slide 81) a. Marginal sinuses b. Periarteriolar lymphocyte sheath (PALS) c. Between marginal sinus & PALS d. Splenic vein

a. Marginal sinuses

Where do resident dendritic cells live in the spleen? (slide 80) a. Marginal sinuses b. Periarteriolar lymphocyte sheath (PALS) c. Between marginal sinus & PALS d. Splenic vein

a. Marginal sinuses

What is the negative test maturing T cells must pass in the thymus? (slide 141) a. Must not recognize self peptides b. Must only recognize peptides presented on MHCs c. Must recognize self peptides d. Must not recognize peptides presented on MHCs

a. Must not recognize self peptides

What interleukin from inducible regulator T cells (iTreg) binds TCR on T cells and reduces proliferation rate of CTLs and makes them less vicious? (slide 109) a. TGFβ b. TNFα c. IL-10 d. IL-17

a. TGFβ *the distinguishing factor between TGFβ & IL-10 is that TGFβ reduces CTL killing and IL-10 blocks co-stimulatory signals

Perforin frees granzyme B within a target cell and triggers other enzymes causing what result? (slide 45) a. Target cell suicide (apotosis) b. Respiratory burst of H2O2 (hydrogen peroxide) c. Cytokine production from the target cell d. Anergizes (neuters) the target cell

a. Target cell suicide (apotosis)

hich of the following is the innate immunity mechanism that prevents viral infections? a) Type I interferons b) CD8+ cytolytic T lymphocytes c) antibody d) natural killer cells e) all of above

e

Choose the correct statement/s about superantigens A. Superantigens bind to the peptide binding groove of MHC molecule B. superantigens induce polyclonal activation of T-lymphocytes C. superantigens may induce exhaustion of T lymphocytes which is due to increased expression of inhibitory receptors. D. A and C are correct E. B and C are correct

e (A: they bind outside the peptide groove of MHC C: increased expression of stimulatory receptors Superantigens stimulate T cells do produce mass storm of cytokines, which leads to apoptosis of T cells. )

During maturation of dendritic cells a) MHC class II are redistributes form intracellular endosomes toward the plasma membrane b) CCR7 receptor starts to be expressed c) there is upregulation of costimulatory molecules d) a and b are correct e) a, b and c are correct

e (?)

Place the following phases of a B cell's life history in the correct chronological order: a. Negative selection b. Attacking infection c. Finding infection d. Searching for infection e. Repertoire assembly f. Positive selection

e, a, f, d, c, b

Helper T cells can join in to activate B cells. (slide 65) a. true b. false

a. true *Once a B cell is activated in a lymph nodule, the number of B cells can double every six hours, and helper T cells may also join in to activate B cells. This is also associated with swollen lymph nodes.

Cytokines have a very limited range. (slide 40) a. true b. false

a. true *Once a helper T cell starts making a certain type of cytokine profile, it is committed. Cytokines have a very limited range (local impact only). Very possible to defend against multiple types of invaders in different body locations simultaneously.

Experienced T cells have more lipid rafts than naïve T cells. (slide 24) a. true b. false

a. true *They also maintain these rafts and do not need co-stimulation for reactivation.

The connection between the receptor and the nucleus is weak in naïve T cells. (slide 23) a. true b. false

a. true *activation with co-stimulation creates a better connection

During battle conditions, T cells need the combination of TGFβ and IL-6 to commit to becoming Th17. (slide 118) a. true b. false

a. true *if there was only TGFβ, things would be calm instead

All TCRs on mature T cells are identical. (slide 13) a. true b. false

a. true *the slide does state there are some exceptions, but identical is underlined so if this is a question... make of it what you want

nTreg are thought to require direct cell to cell contact and are antigen specific. (slide 122) a. true b. false

a. true *they function to provide protection against T cells with the potential to react against self antigens and cause autoimmune disease that escaped negative selection in the thymus

Both helper and killer T cells require activation before they can function. (slide 8) a. true b. false

a. true *they need the help of antigen presenting cells (APCs)

Traditional T cells express what receptors. (slide 13) a. αβ b. γδ c. Εζ d. φχ

a. αβ *vast majority of T cell receptors expressed (95%)

!!!!Fill in the sentence; Tumour cells transfected with the gene encoding ____ have been used as a vaccine that promotes dendritic cell presentation of tumour antigens. a) TCR b) GM-CSF c) IL-2 d) IL-4 e) CAR

b

!!!Mycobacterial Hsp 60 antigen is recognized by macrophage via? a) MHC b) TLR c) mannose receptor d) complement receptor e) NOD receptor

b

A child who had been born with a genetic defect such that they could not express MHC class II on the surface of cells in the thymus would lack which kind of mature T cells? a. cytotoxic T cells b. CD4 T cells c. plasma cells d. all of the above

b

A group of cytokines produced by virus-infected cells that act on nearby cells to inhibit viral replication are called: a) defensins b) interferons c) Toll-like responders d) Tumor Necrosis Factors

b

Activated T cells (not naive) would be most likely to exit the blood through: a. high endothelial vessels (HEV) b. inflamed endothelium c. hassals' corpuscles d. the thoracic duct

b

All the following characteristics of both MHC class I and class II molecules except a) they are expected codominantly b) they are expressed constantibely on all nucleated cells c) they are involved in presentations of antigen d) they are expressed on the surface membrane of B cells e) b and d are correct

b

An antibicrobial peptide that protects epithelial surfaces from pathogens is called: a. glycoprotein b. defensin c. proteoglycan d. lysozyme e. sebum

b

An example of type I hypersensitivity is _____. a. contact dermatitis b. hayfever c. serum sickness d. tuberculin reaction

b

Anti-bacterial proteins that contain both a positively charged and hydrophobic portions are called: a) chemokines b) defensins c) scavenger receptors d) lysozyme

b

C3d and iC3b are breakdown products of _____ which binds to _____ of the B-cell co-receptor. a. C3a; CR2 b. C3b; CR2 c. C3c; CD81 d. C3c; CD19 e. C3b; CD19.

b

Clonal selection occurs when a B-lymphocyte encounters: a. cytokines b. antigen c. selectins d. complement e. chemotactic factors

b

Endogenous peptides are paired with ______ in the ________. a. MHC I, Golgi b. MHC I, endoplasmic reticulum c. MHC II, Golgi d. MHC II, endoplasmic reticulum

b

Examples of granulocytes include all of the following except: a. neutrophil b. monocyte c. basophil d. eosinophil

b

If the alpha chain locus for the T cell receptor rearranges, the ability to rearrange the delta chain will be lost because: a. this is done by RNA splicing b. the delta locus is encoded within the alpha locus c. they are on separate chromosomes d. the cell has become a natural killer cell.

b

IgA: (choose one) a. is transferred across the placenta b. exists in a divalent form in mucosal epithelial tissues c. is mainly membrane bound (not secreted) d. is the predominant Ig in primary responses e. is involved in allergic responses

b

In innate immune responses to Gram negative bacteria, LPS is delivered to _____ on the surface of macrophages by the soluble protein _____. a. TLR9, LPS binding protein b. TLR4, LPS binding protein c. TLR9, C-reactive protein d. TLR4, C-reactive protein

b

Junctional diversity during gene rearrangement results from the addition of: a. switch region nucleotides b. P and N nucleotides c. V, D, and J nucleotides d. recombination signal sequences e. mutations in complementarity-determining regions

b

Microbial polysaccharides may bind to TLR (toll-like receptors) on macrophages and stimulate their phagocytic activity. This is an example of: a. adaptive immunity b. innate immunity c. passive immunity d. both a and b e. all of the above

b

Name the process that is necessary in immunity against most tumors and against viruses that do not infect antigen presenting cells, or impair dendritic cell normal function. a) positive selection b) cross-presentation c) immune suppression d) anergy e) exhaustion

b

Secretory component on dimeric IgA: a. is a remnant of the surrogate light chain b. is a remnant of the poly-Ig receptor c. is composed of polysaccharide d. is a remnant of the J chain

b

Switching from making IgM to making IgG requires: a. somatic hypermutation b. DNA loop excision c. RNA splicing d. post-translational processing.

b

T cells undergo positive selection at the _____ stage of development. a. double positive b. double negative c. triple positive d. pro-T

b

The C3 convertase of the alternative complement pathway is: a) C1 b) C3bBb c) C4bC2a d) C5b67

b

The _____ occurs 6-8 hours after an initial type I allergic response in which mast cells produce leukotrienes, chemokines, and cytokines. a. Arthus reaction b. late-phase reaction c. delayed-type hypersensitivity reaction d. condition known as anaphylactic shock e. skin rash known as chronic urticaria.

b

The immunoglobulin heavy-chain gene contains _______ segments, whereas the immunoglobulin light-chain gene contains _______ segments. a. κ; λ b. VDJ; VJ c. VJ; VDJ d. P; N e. RAG-1; RAG-2

b

The main purpose of inflammation is to: a) regulate affinity maturation b) get cells and molecules to the site of damage c) wall off infectious agents that can't be phagocytose d) stimulate VJ joining of antibody heavy chain genes

b

The molecules mediating signal transduction following antigen binding to cell surface immunoglobulin on a B-cell are called: a. Ig Fc b. Ig-alpha and Ig-beta c. MHC d. CD4 e. CD8

b

Type I hypersensitivity is also known as immediate type hypersensitivity. The initial phase of this response is quick because it involves: a. Changes in gene expression b. The release of pre-formed mediators c. The lysis of mast cells d. A proteolytic cascade

b

What induces the production of type I interferon by virus-infected cells? a. virus DNA b. virus RNA c. Toll-like receptors d. alpha 2 macroglobulin.

b

What order do each of these act in neutrophil migration from the blood into infected tissues? a. chemokine, selectin, ICAM b. selectin, chemokine, ICAM c. ICAM, chemokine, selectin d. chemokine, ICAM, selectin

b

What type of hypersensitivity reaction would result from a drug that bound to your red blood cells and led to their removal by lysis and phagocytosis? a. type I b. type II c. type III d. types II and III.

b

What type of hypersensitivity reaction would result from a mismatched blood transfusion and lead to anemia? a. type I b. type II c. type III d. type IV.

b

What type(s) of hypersensitivity would be diminished in a person who expressed a non-functional form of C3? a. Type I b. Types II and III c. Types III and IV. d. Type IV only

b

Which of the following activities are most closely associated with natural killer cells? (select all correct answers) a. production of TNF-alpha b. lysis of virus infected cells c. phagocytosis of bacteria d. release of reactive oxygen intermediates e. production of IFN-gamma

b

Which of the following are not associated with soluble antigen? a. type I hypersensitivity b. type II hypersensitivity c. type III hypersensitivity d. types I and III

b

Which of the following can be used to enhance antigen presentation by dendritic cells? a) TCR b) GM-CSF c) IL-2 d) antid-CD3 antibody e) TFG-y

b

Which of the following cell types is not considered a professional antigen-presenting cell? a. macrophage b. neutrophil c. B cell d. dendritic cell

b

Which of the following cell types is not considered a professional antigen-presenting cell? a. macrophage b. neutrophil c. B cell d. dendritic cell e. all of the above are professional antigen-presenting cells.

b

Which of the following describe M cells in the gut? a. They derive their name from mucus cells. b. They are located in the dome of a Peyer's patch. c. They deliver antigens and pathogens from the lymphoid tissue to the luminal side of the gut mucosa. d. They are a type of macrophage.

b

Which of the following does not accurately describe complement components? a. soluble proteins b. made by the spleen c. some function as proteases once activated d. activated by a cascade of enzymatic reactions.

b

Which of the following is not considered to be an example of immunotherapy? A) administration of anti-CD20 humanized monoclonal antibody B) vaccination against hepatitis B and HPV C) radiation treatment D) administration of IL-2 E) administration of anti-CTLA4 humanized monoclonal antibody

C) radiation treatment

Humanization of monoclonal antibodies is done in order to A) avoid a HAMA response. B) create antibodies suitable for clinical treatment of human diseases. C) create antibodies that are useful for doing an ELISA. D) All of the above are true. E) Only a. and b. are true.

E) Only a. and b. are true.

Secondary lymphoid organs A) are sites where B and T lymphocytes are activated. B) include the lymph nodes, the spleen, and MALT. C) are tissues where all cells of the blood are formed. D) All of the above are true. E) Only a. and b. are true.

E) Only a. and b. are true.

Immunoglobulin gene rearrangement during B cell development results in A) an increased affinity of antibody of antigen. B) assembly of a complete antibody coding sequence that is suitable for expression. C) diversity of antibody specificities. D) All of the above are true statements. E) Only b. and c. are true

E) Only b. and c. are true

The function of positive selection in the thymus is A) to eliminate autoreactive T cells. B) to promote development of T cells with functional TCRs capable of interacting with self MHC. C) to promote thymocyte differentiation to either the CD4 or CD8 lineage. D) All of the above are true. E) Only b. and c. are true

E) Only b. and c. are true

The diseases diphtheria and tetanus A) are caused by viruses. B) can not be prevented by vaccination. C) are caused by protozoan parasites. D) are considered to be opportunistic infections. E) are caused by toxins secreted by gram positive bacteria.

E) are caused by toxins secreted by gram positive bacteria.

Monoclonal antibodies are used in cancer treatment because of their ability to _____. A) target tumor cells for immune responses such as ADCC or opsonization B) induce formation of anti-tumor memory cells C) enhance expression of tumor-specific antigens D) be conjugated to cytotoxic drugs to kill tumor cells E) both a. and d.

E) both a. and d.

When an individual receives a kidney transplant, the concern will be to control the development of _____. A) graft-versus-host disease B) transplant rejection C) allorecognition D) allergic reactions E) both b. and c.

E) both b. and c.

Which of the following is an example of a G protein-coupled receptor (GPCR)? (A) T cell antigen receptor (TCR) (B) The B cell antigen receptor (BCR) (C) CD4 (D) Interlukin-2 (IL-2) receptor (E) Chemokine receptor CCR7

(E) Chemokine receptor CCR7

*BILDE* A

A. A- Lck, B- ZAP-70, C- phosphorylated ITAM B. A-CD45, B- CD4, C-dephosphorylated ITAM C. A- ITAM, B- epitope, C- phosphorylated Lck D. A- Lck, B- CD45, C- dephosphorylated ITAM E. A- CD3, B- Lck, C- phosphorylated ZAP-70

From the video: Neutrophils can undergo a dramatic form of cell death called NETosis, killing bacteria but also causing damage to nearby tissues. Epithelial cells are then replaced by: A. Stem cells found in intestinal crypts B. Stem cells found in peyers patches C. Stem cells found in vili D. Multiplying M cells, which are stem cells E. Migrating epithelial cells coming from the mesenteric lymphnode

A. Stem cells found in intestinal crypts

T cells can express the following surface molecules, EXCEPT: A. immunoglobulin B. CD3 C. CD4 D. CD8 E. all of the above

A. immunoglobulin

What is formed when one heavy chain is cleaved by PAPIN? A.) 2 Identical FAB fragments and 1 FAC fragment B.) 2 FAB fragments and a F(ab')sub 2 fragment C.) 1 F(ab')sub 2 fragment D.) none of the above

A.) 2 Identical FAB fragments and 1 FAC fragment

How is the innate immune system complex activated? A.) Alternative pathway or Mannan-binding lectin pathway B.) A & B C.) Classical pathway D.) None of the above

A.) Alternative pathway or Mannan-binding lectin pathway

Epitope binding before Fc receptor engagement is not required for what Immunoglobulin? A.) IgE B.) IgA C.) IgE D.) IgM

A.) IgE

The B-lymphocyte antigen receptor is: A.) IgM B.) IgG C.) IgA D.) IgD E.) IgE

A.) IgM

What is the first Ig to be formed following antigenic stimulation? A.) IgM B.) IgZ C.) IgL D. None of the above

A.) IgM

Which isotype is effective at both immobilizing an antigen and in activating the classical pathway of complement? A.) IgM B.) IgA C.) IgD D.) IgE

A.) IgM

Immunogens usually ____ the intensity of the epitope-specific immune response? A.) Increase B.) Decrease C.) Neutralize D.) None of the above

A.) Increase

Which is a common pathogen of the scalp? A.) Malassezia B.)Streptococcus C.)Proteus D.) Niesseria

A.) Malassezia

A 16 year old boy comes into the ER with acute appendicitis, what cell count do you think will be increased? A.) Neutrophils B.) Eosinophils C.) Mast cells D.) White blood cells E.) Red Blood cells

A.) Neutrophils

Engulfment of microbes and other particular matter, as a part of the body's cleansing mechanism is A.) Phagocytosis B.) Micropinocytosis C.) A & B D.) None of the above

A.) Phagocytosis

Antigen presentation is an important aspect of immune function. Several cell types are capable of presenting the antigen to immune cells. What cell type is least likely to be called as a professional antigen-presenting cell? A.)T lymphocyte B.)B lymphocyte C.)Macrophage D.)Dendritic cell E.)Langerhans cell

A.) T lymphocytes

An immunologist has developed an antibody that reacts with a protein that is expressed on developing immune cells. If the researcher decides to stain various lymphoid tissues to look for expression of this protein using the antibody that he developed, what tissue would most likely give a strong reaction? A.) Thymus B.) Tonsils C.) Spleen D.) Lymph nodes E.) Peyer's patches

A.) Thymus

Immunoglobulins are synthesized by B lymphocytes and secreted by plasma cells. A.) True B.) False

A.) True

Interferons are proteins. A.) True B.) False

A.) True

Phagolysosomes contains enzymes capable of generating free radical that can efficiently kill microbes. A.) True B.) False

A.) True

In the class I MHC pathway of antigen presentation, peptides generated in the cytosol are translocated into the endoplasmic reticulum in which of the following ways A.By ATP-dependent transport via TAP B.By passive diffusion C.By receptor-mediated endocytosis D.Through membrane pores E.Via the proteasome

A.By ATP-dependent transport via TAP

What cell adhesion protein is found on the endothelial cells and binds to ligands on the neutrophil? This cell adhesion protein functions like Velcro to grab neutrophils to slow them down. The neutrophils are then described as "rolling." A.Selectin B.Integrin C.ICAM D.GPCRs E.TNF

A.Selectin

IgG

ADCC is caused by this antibody

Vaccinations fall into what category of immunity?

Active artificial

IgM

Agglutination of bacteria is best performed with this isotope

A low AH50 level suggests a deficiency in; Factor B Properdin Factor D

All of the above

An Enzyme-Linked Immunosorbent Assay (ELISA) is used to detect and quantify: Proteins Antibodies Peptides Hormones All of the above

All of the above

Microfluidics can be used to measure: A.) Cell count B.) Cell phenotype C.) Cell Deformability D.) Cell size E.) All of the above

All of the above

Which one of the following statements about peptide binding to MHC molecules is true?

An MHC molecule has only one peptide-binding site, which accommodates only a single peptide at a time.

allegen

An immunogen that elicits a hypersensitivity response is termed a(n) ___________

Subsequent exposure to the same antigen produces a

Anamnestic response

All are true of the Anamnestic response when compared to a primary antibody reaponse EXCEPT

Anamnestic response has a longer lag phase

A synonym for immunoglobulin are

Antibodies

If cytokines are associated with Tcells, then—— are associated with B cells?

Antibodies

Immunogens are more commonly referred to as

Antigens

Chronic inflammation can be a symptom of

Autoimmune diseases Viral infections

!!!!!For each region of a class I MHC molecule listed I-IV, match the lettered label (A-D) from the diagram below I) peptide binding groove II) CD8-binding site III) 𝛽2- microglobulin IV) polymorphic region a) I-D, II-C, III-A, IV-B b) I-C, II-A, III-D, IV-B c) I-B, II-A, III-C, IV-D d) I-C, II-B, III-D, IV-A e) I-A, II-B, III-D, IV-C

B

!!!!!This drug is commonly used to prevent transplant rejection. It functions by inhibiting calcineurin. a) Ipilimumab b) Tacrolimus c) IFN-𝛼 d) Abatacept e) lack of correct answer

B

ADCC a) is a process of phagocytosis b) leads to death of a target cell c) is a process which requires IgA produced by mast cells d) is a process by which the mannose- bingin lectin pathway of complement is activated e) all answers are correct

B

Anaphilatoxins include a) C3b b) C5a c) IgG d) C4b e) none

B

At the pre-B cell stage VDJ genes for light chain of Ig a) are in a germline configuration b) are being rearranged c) are after rearrangement d) are before rearrangement e) lack of correct answer

B

CD19 is a marker of which cells? A. monocytes B. B cells C. NK cells D. T cells E. Lack of correct answer

B

Chooose the correct sentence/s a) TGF-B is produced by TH2 cells b) M tubercolosis inhibits lysosome fusion with the phagosome c) type 1 response induces the production of IgA d) answers a and b are correct e) answers b and c are correct

B

It enchances phagocytosis and removes immune complexes from circulation. It is a) C3 b) C3b c) Ca d) C5 e) answers a and c and d are correct

B

The part of a antibody that recognizes an antigen is called a) epitope b) paratope c) framework subregion d) CDR1 e) BCR

B

The rolling phase during neutrophil migration is mediated by: A. integrins B. selectins C. CD3 D. A, B and C are correct E. no correct answer

B

Weak antigen binding by TCR causes production of a) IL-12 b) IL4 c) TNF d) IFN-y e) a b and d are correct

B

Which of the following are present in the cSMAC zone of immunological synapse? A. TCR, ICAM, CD45 B. TCR, CD28, MHC C. ICAM, CD28, CD2 D. MHC, TCR, ICAM E. CD28, CD4, ICAM

B

MHC class II molecule: a) is constitutively expressed by every cell but erythrocytes b) presents long peptides c) interacts with CD4 co-receptor by β1 domain d) interacts with CD8 co-receptor by β1 domain e) is a homodimer f) presents exogenous antigens, cleaved in proteasome

B (A: APCs C: Interacts with CD4 on beta 2 domain. E: its a heterodimer.)

Choose the cytokines required for B cell development a) IL-10 b) IL-7 c)TGF-B d) answers a and b are correct e) answers a and c are correct

B (B: IL-7 is important for B and T cell development. C: inhibits B cell proliferation.)

CTLA-4: a) is a costimulatory molecule b) binds to B7 on APC c) is expressed after T cell becomes activated d) answers a, b and c are correct

B (C: constantly expressed on t reg cells)

Immature B cell: a) may enter lymph mode b) has IgM on it's surface. c) undergoes negative selection in lymph node d) answers a and b are correct e) answers a, b and c are correct

B (C: in bone marrow)

Which of the following adhesion molecules directs naive T cells into lymph nodes? a) ICAM-1 b) L-selectin c) P-selectin d) VCAM-1 e) PECAM

B (Naive T cells have CCR7 and L-selectin, which helps them stick to the lymph node)

The highest polymorphism in MHC class II molecules a) is concentrated in α1 domain b) is concentrated in β1 domain c) is concentrated in β2 domain d) is concentrated in α2 domain e) is concentrated in β2 microglobulin

B (The polymorphic residues are located in the α1 and β1 segments....In human class II molecules, most of the polymorphism is in the β chain)

The mechanism that is used to eradicate the established viral infection include a) production of neutralizing antibodies b) production of specific cytotoxuc T cells c) type 1 interferons d) answers b and c are correct e) answers a and b and c are correct

B (a+c: protection against infection

Elementary body of chlamydia pneumonia a) can replicate in infected cells b)can infect cells c) is the same as reticulate body d) answers a and b are correct e) lack of correct answer

B (a: reticulate body can replicate)

Which of the following is a cytokine known to have antitumour activity a) epidermal growth factor b) interferon-y c) transofrming growth factor B d) interleukin 10 e) b and c are correct

B (?) (Forrige var E)

Which of the following mechanisms contributes most to both Ig and TCR diversity? A) Multiple possible combinations of the different V, D, and J segments B) Changes in the nucleotide sequences at the junctions between recombined V, D, and J segments C) Somatic mutation of variable genes D) Isotype switching E) Polymorphism

B) Changes in the nucleotide sequences at the junctions

Which of the following is true about receptor editing? A) It does not involve RAG 1/ RAG 2. B) It can rescue self reactive B cells from negative selection. C) It applies only to the heavy chain. D) It rarely alters the antigenic specificity of the Ig receptor

B) It can rescue self reactive B cells from negative selection.

Which of the following statements about IgA is false? A) It is abundant in secretions. B) It is effective at sensitizing mast cells and basophils. C) It can confer passive immunity in nursing infants. D) It is relatively resistant to destruction from proteases (enzymes that break down proteins).

B) It is effective at sensitizing mast cells and basophils.

Which type of vaccine would be most effective at evoking a Tc memory response against a virus? A) Whole organism killed. B) Live-attenuated. C) Sub-unit. D) toxoid.

B) Live-attenuated.

Which of the following statements about cytokines is not true? A) Cytokines are secreted proteins that function in cell-cell communication. B) The primary effect of the interferons is to promote chemotaxis. C) TNF- is associated with inflammation. D) Cytokines are responsible for fine tuning of immune responses.

B) The primary effect of the interferons is to promote chemotaxis.

In preparing for isolation of a monoclonal antibody, an immunologist first immunizes a mouse three separate times over the course of several weeks. What was the most likely reason to do this? A) To maximize the production of IgM antibodies. B) To increase the odds of isolating antigen specific B cells. C) There is no good reason, as a single immunization would have been sufficient. D) Antigen specific B cells will not be formed during the first immunization.

B) To increase the odds of isolating antigen specific B cells.

The innate immune system can recognize_____ microbial molecules. A. 100 B. 1000 C. 10^6 D. 10^7

B. 1000

IgM is the predominant Ig produced in the mucosal immune system. A. True B. False

B. False

What cell adhesion protein-ligand pair creates a very strong interaction stopping the neutrophil from rolling. A. Selectin-VCAM B. Integrin-ICAM C. C5a-LPS D. GPCR-Cell Adhesion Ligand (CAL) E. Arrestin-C5a

B. Integrin-ICAM

When a BCR (B cell receptor) binds to an epitope, the specialized cytoplasmic tails of Ig alpha and Ig beta initiate an intracellular signaling cascade that may lead to beta cell what? A.) Death B.) Activation C.) Proliferation D.) Stasis E.) None of the above

B.) Activation

What are the 3 lines of defense of the immune system in the order that a pathogen would encounter them (1st line->2nd line->3rd line). A.)Adaptive immune system->Innate immune system->Barriers B.) Barriers-> Innate immune system-> Adaptive immune system C.) Barriers-> Adaptive immune system-> Innate immune system D.) Barriers-> Passive immunity-> Active immunity

B.) Barriers-> Innate immune system-> Adaptive immune system

Which receptor is expressed on the surface of approximately 2/3s of mature T cells? A.) CD8 B.) CD4 C.) CD3 D.) CD9

B.) CD4

Which of the following proteins involved in attenuation of immune signaling become associated with immunoreceptor tyrosine inhibitory motifs (ITIMs)? A.E3 ubiquitin ligases B.SH2 domain-containing tytrosine phosphatases (SHP-1, SHP-2) C.Suppressors of cytokine signaling (SOCS) D.CTLA-4 E.PD-1

B.SH2 domain-containing tytrosine phosphatases (SHP-1, SHP-2)

CD28

B7-1, B7-2 B7-1 and B7-2 on antigen-presenting cells bind to CTLA-4 on activated T cells, stimulating negative signals in the T cells.

Neutrophils are associated with

Bacterial infections Phagocytosis

Which cell has high concentration of heparin and histamine in their granules?

Basophils

What is the protein that nude T cells, in bone marrow, express that protects against apoptosis? (slide 139)

Bcl-2 *Bcl-2 keeps from killing you... you're welcome ;) Also, note that this is not the case for T cells in the thymus which matured into double positive (DP) cells. They express very little Bcl-2 and lots of Fas protein... They gonna die if they get caught.

Decrease in CRP does NOT indicate

Beginning of infection

Which is the most reliable indicator for increasing risk of opportunistic infections and other AIDS related disorders in HIV infected individuals?

Blood CD4+ T cell count

!!!!A state of T-lymphocyte unresponsiveness that occurs following peptide + major histocompatibility complex (pMHC) engagement is known as a) allergy b) apoptosis c) anergy d) autoimmunity e) hypersensitivity

C

!!!Which of the following bacteria invade M cells to cross the epithelial barrier? a) E. coli b) Helicobacter c) Shigella d) Mycobacteria e) a and b are correct

C

!!!Which one is NOT acute phase protein? a) Fibrinogen b) CRP c) defensin d) serum amyloid A e) Mannose-binding lectin

C

!!!Which segment codes for isotype expressed in inmature B cell? a) Cα b) Cβ c) Cμ d) Cγ e) Cε

C

!!!type 1 interferons: a) are involved in acute reaction to G- bacteria b) mediate[something] c) mediate anti-viral response d) is an opsonin

C

CD8 a) is a costimulatory molecule of Tc cells b) is a part TCR complex in Th cells c) binds to MHC class I d) answers a and b and c are correct e) answers b and c are correct

C

Costimulatory molecules a) are required for activation of effector T cells b) are CD4 and CD28 c) are CD28 and B7 molecuoles d) are expressed only by APC e) are expressed only by lymphocytes

C

DC cells: a) present antigens only to CD4+ T cells b) present only peptide antigens c) after antigen binding produce cytokines d) present antigens to B lymphocytes due to crosspriming e) undergo maturation in blood and then migrate into tissues f) express CD40 L.

C

Fill in the following sentence; Phagocytosis is primarily mediated by _____ and ____ during an immune response to an infection. Both of these cell types express Fcy receptors that bind to the constant region on ____ antibodies and phagocytose the antigen-coated microbodies. a) NK cells, B cells, IgG b) neutrophils, NK cells, IgE c) neutrophils, macrophages, IgG d) Macrophages, T cells, IgA e) T cells, mast cells, IgE

C

Superantigen a) binds to TCR o only one lymphocyte clone but with a very high affinity b) activates many types of APC c) activates many lymphocyte clones d) a and e are correct e) binds to antigen binding groove

C

CD4 a) is a costimulatory molecule of Th cells b) is a part of TCR complex in Tc cells c) binds to MHC class II d)answers a b and c are correct e) contains ITAM motifs

C (A: its just a signaling molecule B: CD3 is a part of the TCR complex. E: ITAM on CD3)

Costimulatory molecules: a) are expressed by immature DC b) are expressed by activated T lymphocytes c) are expressed by activated macrophages d) create TCR complex with T lymphocyte antigen receptor e) are crucial for anergy induction in T lymphocytes f) are responsible only for cell activation.

C (A: mature DC B: T cells always express their costimulatory molecule, CD28. D: they dont create a TCR complex. E: Anergy occur when costim. are inhibited F: also release of cytokines.)

HLA-G a)belongs to classical MHC class I molecules b) is highly polymorphic molecule c)is involved in recognition by NK cells d) answers a and c are correct e)answers a, b and c are correct

C (A: non-classical mHC I B: not polymorphic HLA-G is in the placenta, and will protect the fetus from NK cells.)

IgG a) are synthesized by NK cells b) are usually dimers in mucosae c) can be recognized by Fcy receptors d) induce degranulation of mast cells e) are not able to cross the placenta

C (B: IgA D: IgE)

A rare human immunodeficiency disease called hyper-IgM syndrome is caused by a defect in the CD40 ligand gene. These patients have abnormally high levels of IgM but almost no other isotypes. Which of the following statements is true regarding these patients? A) B cell development in the bone marrow does not follow normal selection mechanisms. B) Their B cells are defective in VDJ recombination. C) B cells do not undergo isotype switching due to an absence of T cell co-stimulatory signals. D) Responses to TI-2 antigens are impaired.

C) B cells do not undergo isotype switching due to an absence of T cell co-stimulatory signals.

Which of the following statements about inflammation is false? A) Inflammation is an important initiating immune response that also enhances delivery of oxygen and clotting factors to a site of infection or injury. B) Inflammation promotes the infiltration of leukocytes to a site of infection. C) Inflammation is never harmful. D) Activated mast cells and macrophages send out signaling molecules that contribute to initiation of an inflammatory response.

C) Inflammation is never harmful.

Which of following statements about IgM antibodies is false? A) It is expressed on the surface of naïve B cells. B) It is the most abundant isotype secreted in a primary response. C) It is the most abundant isotype in breast milk. D) The secreted form is a pentamer.

C) It is the most abundant isotype in breast milk.

Which of the following statements about TLRs is true? A) They are only found in humans. B) They are created by gene rearrangement during B cell and T cell development. C) They are PRRs. D) They are found only on the surface of APCs.

C) They are PRRs.

Cancers develop their own blood supply and become vascularized through _____. A) apoptosis B) neoplasia C) angiogenesis D) metastasis E) malignant transformation

C) angiogenesis

Acute rejection of a kidney graft involves the activation of recipient T cells by _____ of _____ origin. A) dendritic cells; recipient B) B cells; recipient C) dendritic cells; donor D) macrophages; recipient E) B cells; donor

C) dendritic cells; donor

Which of the following cells isa phagocyte that releases cytokines to battle against pathogens? A. Macrophage B. Neutrophill C. All of the above D. None of the above

C. All of the above

Shortly following the initial infection, what cells and molecules will you be least likely to find in the area of infection A. Dendritic Cells B.Phagocytes C. B-Lymphocytes D. Complement proteins E. NK Cells

C. B-Lymphocytes

CTLA-4 on regulatory or responding T cells binds to ______ molecules on APCs or removes these molecules from the surface of the APCs. A. CD28 B. MHCI C. B7 D. MHCII E. CD4

C. B7

Anergy is defined as: A. Apoptosis B. Block in activation C. Functional unresponsiveness D. All of the above E. None of the above

C. Functional unresponsiveness

Secretory IgA protects external mucosal surfaces by: A. Triggering mast cells B. Recruiting phagocytic cells C. Preventing microbial adherence to the mucosa D. Binding to epithelial cells E. Its secretory piece

C. Preventing microbial adherence to the mucosa

What type of cell(s) are acidic, have cytoplasmic granules, contain histamine, and cause muscle contraction? A.) Neutrophils B.) Lymphocytes C.) Basophils D.) All of the above E.) None of the above

C.) Basophils

Pepsin or Papain enzymatically cleaves which Ig into fragments? A.) IgM B.) IgA C.) IgG D.) IgE E.) None of the above

C.) IgG

Which immunoglobulin first appears in a new born, and is passively acquired from the mother because it crosses the placenta? A.) IgM B.) IgD C.) IgG D.) IgE

C.) IgG

Which one activates MAC?

C5

Superantigen: a) binds with MHCmolecules in the same way as agretop b) stimulates all lymphocyte clones c) can cause food poisoning d) lack of correct answer e) answers a-c are correct

C? (B: stimulates only T cells.)

B cells can get the second signal from T-helper cells by ______ on the T cell binding to ____ on the B cell.

CD40 ligand, CD40

CD8+ T cells and NK cells both recognize MHC class I molecules. Describe the receptors involved, and the functional outcomes of the interaction. (4p)

CD8+ T cells use the T cell receptor (TCR) and CD8 molecule to bind to MHC class I. The TCR bind both to the MHC class I and the peptide, and the CD8 molecule stabilize this binding by interacting with conserved parts of the MHC molecule, i.e. the 3 domain. MHC class I interaction with correct peptide leads to activation (together with costimulation) of the naïve cell, and to killing of target cells via perforin/granzyme by the effector cells. NK cells express inhibitory and activating MHC class I specific receptors called KIR, killer immunoglobulin like receptors. (Inhibitory receptor has an intracellular signaling motif called ITIM, immunotyrosine inhibitory motif, while activating receptors associates with adaptor molecules with ITAM, immunotyrosine activating motif). Balance with activating receptors, e.g. NKG2D. Normal expression of MHC I give inhibition via KIR, while if reduced expression of MHC I (and in combination with upregulation of activating ligands) lead to killing of target cells by the same mechanism as T cells; perforin/granzyme.

Proficiency testing is incorporated by

CLIA

The U.S congress enacted ----- in response to concerns about laboratory testing errors.

CLIA'88

Adaptive immunity is composed of

Cell components Humoral components

Which congenital neutrophil abnormality is related to albinism?

Chediak-Higashi Syndrome

CD4

Class II MHC CD4 on T cells bindsto a nonpolymorphic region of class II MHC during T cell recognition of antigen. This molecular interaction promotes signaling events required for T cell activation.

Thymocytes mature as they move from the _____ to the _____.

Cortex, medulla

IgA

Corynebacterium diphteriae colonizes the mucous membrane in the throat. Which of the antibody isotope can prevent that colonization?

What is cross-priming? Describe briefly why you think that it is important for mounting CD8 T cell responses.

Cross-priming is when an antigen presenting cell (APC) take up, by e.g. phagocytosis, either particles of dead infected cells, or particles of microbes, and present this material, peptides, on MHC class I molecules. The dogma is that antigen presented on MHC class I should be from intracellular proteins. If this would be the only pathway to present antigen on MHC class I, this would mean that only APCs infected themselves by an microbe can function as an APC to activate naive CD8+ T cells via antigens bound to MHC class I together with CD80/86. Cross-priming allows APCs to function activate naive CD8+ T cells with antigens taken up externally.

!!!!Choose the T cell signaling molecule that becomes an active transcription factor on dephosphorylation. a) CD3 b) Lck c) ZAP-70 d) NFAT e) NF- kB

D

!!!!Neutropenia could be treated with: a) IL-1 b) IL-11 c) erythropoietin d) GM-CSF e) TNF-α

D

!!!!Treg 1 induced immunosuppression is mediated by: a) perforin and granzyme b) IL-1 c) TNF-α d) IL-10 e) phagocytosis

D

!!!!Which cytokine is not Th1 related? a) TNF-α b) TNF-β c) IFN-γ d) TGF-β (PD claim so, needs verification) e) lack of correct answer

D

!!!Which process is resposible for TCR diversity? a) Somatic hypermutation b) something c) somatic recombination d) junctional diversity( Im personally not sure,but lot of people claims so) e) all processes are involved

D

Alpha chain of TCR is encoded by gene segments: a) V b) D c) C d) answers a and c are correct e) answers a, b, and c are correct

D

Antibodies and T lymphocytes are the respective mediators of which two types of immunity? a) innate and passive b) passive and active c) specific and nonspecific d) humoral and cell-mediated e) adult and neonatal

D

BCR a) presents antigens to T cells b) is composed of antibody bound to B cell by receptor for Fc fragment c) is produced in bone marrow d) answers a b and c are correct e) lack of correct answer

D

CD28 a) is expressed on T cell b) transmits activating signal to T cell c) is expressed on APCs d) a and b are correct e) a-c are correct

D

It increases vascular permeability and may act as chemotactic factor. It is: a) C3 b) C3b c) C3a d) C5 e) answers a, c and d are correct

D

J-chain a) is present in polymeric forms of IgM b) promotes the polymerization c) is de facto J gene segment d) answers a and b are correct e) answers a, b and c are correct

D

Light chain of immunoglobulin is encoded by gen segment/s a) VJκ b) VDJκ c) Cκ d) a and c are correct e) b and c are correct

D

Light chain of immunoglobulin is encoded by gene segment/s: a) VJκ b) VDJκ c) Cκ d) answers a and c are correct e) answers b and c are correct

D

TCR recognizes antigens a)that are presented by APC b)that are presented by BCR c)that are bound to MHC d)answers a and c are correct e)answers b and c are correct

D

The Fab fragment of an antibody A. has the ability to bind to receptors for Fc fragment B. is present on both heavy and light chains C. undergoes somatic hypermutation in germinal centre D. answers B and C are correct E. answers A, B and C are correct

D

What is the effect of anaphylatoxins on immune cells? A. reduction of immune compressors removal B. enhancement of phagocytosis C D. degranulation of mast cells

D

Which of the following antibodies activates eosinophils in parasitic infections? A. IgA B. IgM C. IgG D. IgE E. IgO

D

Memory T cells a) express CD45RO b) express CD62L c) dont express CCR7 d) answers a and b and c are correct e) answers b and c are correct

D (Memory T cell express CD45RO and CD62L/L-selectin, but do not express CCR7.)

Th1 cells a) secrete IFNy b) differentiate upon IL12 activation c) need GATA-3 transcription factor d) answers and b are correct e) answers a b and c are correct

D (Th2 need GATA-3)

Pathogen associated molecular patterns (PAMPs) A) are recognized by PRRs. B) are general structures common to pathogens. C) typically function as adhesion molecules or as receptors for cellular signaling. D) Both a. and b. are true.

D) Both a. and b. are true.

Chemical and physical agents that increase mutation rates by damaging DNA and increase the likelihood of developing cancer are known as _____. A) oncogenes B) malignant transformers C) antitumor suppressor agents D) carcinogens E) tumor-associated agents

D) carcinogens

L-selectin is expressed on: A. leukocytes B. T cells C. endothelial cells D. A&B E.All of the above

D. A and B

_________ increase the expression of integrin ligands on the endothelium A.TNF B.IL-1 C.IL-4 D.A&B E.All of the above

D. A and B

All of the following components can be part of innate immune responses EXCEPT: A. natural killer cells B. alternate pathway of complement activation C. MAC D. B cells E. Macrophages

D. B cells

The directional movement of cells in response to chemicals is called A. Chemokinesis B. Rolling C. Transmigration D. Chemotaxis E. Phagocytosis

D. Chemotaxis

The signaling pathways triggered by Toll-like receptors typically result in activation of which of the following pairs of transcription factors? A. NFAT and T-bet B. AP-1 and GATA-3 C. Fos and STAT-6 D. NF-κB and IRFs E.Lck and Jun

D. NF-κB and IRFs

What are M cells? A. A type of mucosal B cell that secretes IgM B. Lamina propria macrophages that ingest bacteria that have invaded through the intestinal epithelium C. Alveolar macrophage that secrete surfactant D. Specialized intestinal epithelial cells that transport antigens or microbes from the gut lumen into Peyer's patches E. A type of intestinal dendritic cell that tolerizes T cells to food antigens.

D. Specialized intestinal epithelial cells that transport antigens or microbes from the gut lumen into Peyer's patches

What is the pass rate of T cell graduation?

1 out of 30... just like sublux, ay yoooo THESE ARE THE JOKES KIDS

If the first tube of a 2-fold serial dilution has a 1:2 dilution, the fifth tube will have a

1:32 dilution

How many classes of MHC's are there?

2

How many stages can phagocytosis be divided into?

6

Standard deviation dictate range percentages of______ except

80%

Th1 cells make macrophage sensitive to interferon gamma by producing a) TNF b) CD40 c) FAS d) FASL e) answers b and d are correct

A

The process that is enhanced by the binding of both antibodies and complement fragments such as C3b by phagocytes is known as a) opsonization b) complement activation c) neutralization d) a and c are correct e) lack of correct answer

A

Which of the following cells load peptide fragments into MHC class II molecules? A. CD4+ T cells B. CD8+ T cells C. dendritic cells D. gammedelta T cells E. neutrophils

A

How are Ig landmarks cleaved? A.) pepsin B.) papain C.)papule D.) a and b E.) all of the above

D.) a and b

What are 3 types of granulocytes? A.) Neutrophils B.) Eosinophils C.) Basophils D.) a, b, & c E.) None of the above

D.) a, b, & c

The 1st line of defense to pathogens in the body is provided by__________ A.) Mechanical barriers B.) Chemical barriers C.)Biological barriers D.) all of the above

D.) all of the above

strong antigen binding by TCR stimulates T cell to produce a) IFNy b) IL4 c) IL5 d)IL12 e) a b and d are correct

A

Thymus independent antigen a) does not require CD40-CD40L interaction b) is only derived from proteins c) produces the production of IgM d) answers a and c are correct e) answers a and b and c are correct

A (b: derived from non-proteins.)

The immunoglobulin that can bind to the Fc epsilon receptor on cell surfaces and is most commonly implicated in medical conditions such as allergy, hives, asthma, and anaphylactic shock is: A.)IgA B.)IgG C.)IgM D.)IgE E.)IgD

D.)IgE

What are M cells? A.A type of mucosal B cell that secretes IgM B.Lamina propria macrophages that ingest bacteria that have invaded through the intestinal epithelium C.Alveolar macrophage that secrete surfactant D.Specialized intestinal epithelial cells that transport antigens or microbes into Peyer's patches E.A type of intestinal dendritic cell that tolerizes T cells to food antigens.

D.Specialized intestinal epithelial cells that transport antigens or microbes into Peyer's patches

CTLs are the major defense against which of the following class of organisms? A.Extracellular bacteria B.Fungi C.Protozoans D.Viruses E.Helminths

D.Viruses

immunogen

A substance that can generate an immune response is called an _____________

Hapten

A substance that can react with products of the immune system but CANNOT generate an immune response

IgG

A virus in the blood stream is best neutralized by this immunoglobulin isotope

___________ is a DNA virus associated with development of cervical cancer. A) Human papillomavirus (HPV) B) Human immunodeficiency virus (HIV-1) C) Human herpes virus 8 (HHV8) D) Epstein-Barr virus (EBV) E) Hepatitis B virus

A) Human papillomavirus (HPV)

Which of the following isotypes is not associated with a clear effector function? A) IgD B) IgE C) IgA D) IgM E) IgG

A) IgD

Which of following cell types do not typically serve as antigen presenting cells (APCs)? A) eosinophils B) B cells C) Dendritic cells D) Macrophages

A) eosinophils

Immunoglobulin gene rearrangement during B cell development A) first occurs on the heavy chain genes. B) rarely yields self reactive receptors. C) will always yield a fully functional antibody. D) does not begin until the pre-B cell stage.

A) first occurs on the heavy chain genes.

A typical antibody molecule A) has two identical antigen binding sites. B) can only bind to processed antigens presented in MHC molecules. C) is only found in a membrane bound form on the surface of a B cell. D) is composed of two polypeptide chains

A) has two identical antigen binding sites.

Humoral

The use of snake venom antitoxin is an example of _____________ immunity

MHCs are highly diverse. Why can this cause a problem?

They can create problems with organ transplantation

IgG

This antibody is best at causing opsonization

IgE

This isotope is thought to play a role in the host's defense against parasitic worms

Accuracy describes how close a test result is to the true value

True

Ig E antibody, TH2 cells

Type I

Explain how TNF can be useful and harmful to the host upon infection.

Useful: TNF is produced by mononuclear phagocytes and recruits neutrophils and monocytes to site of infection and to activate those cells to kill microbes. Stimulate endothelial cells to express adhesion molecules and induce production of chemokines. Harmful: Large systemic amounts induce fever, acute phase proteins and can eventually lead to intravascular thrombosis and clinical shock.

VLA-4

VCAM-1 VLA-4, an integrin expressed on T cells, binds to VCAM-1, an Ig superfamily member expressed on activated endothelium. VLA-4-VCAM-1 interactions promote T cell recruitment into inflammatory sites.

Active artificial immunity is acquired through

Vaccinations

antibody

What is an antigen receptor on a B cell?

Cortex

Where are most B cells found in a lymph node?

Thymus

Where do T cells become educated (mature)?

Bone Marrow

Where does the B cell develop?

Erythropoetin

Which factor involved in hematopoiesis stimulates the production of erythrocytes?

IgG1

Which of the following IgG subclasses is the major subclass with a long half-life and ability to pass through placenta?

IgM

Which of the following immunoglobulins exists as a pentamer and can activate complement as a single molecule when bound to an antigen?

IgA

________ deficiency is the most common immunodeficiency

complement

_________ binds to the (CH2 and CH3)2 antibody domain

Tumor

_________ is a term of inflammation that means swelling

DiGeorge's

___________ Syndrome is due to a lack of both cellular and humoral immunity

!!!!Name circulating cells with cytoplasmic granules that produce heparin, histamine and other mediators involved in inflammation. a) basophil b) B cell c) mast cell d) macrophage e) dendritic cell

a

Women can show no signs of certain genetic immune deficiencies but can pass them on to some of their sons because: a. These defects involve X-linked recessive traits b. These defects involve X-linked dominant traits c. These defects usually involve multiple gene defects d. These defects are aggravated by excessive numbers of fart jokes

a

_______ involves successive rearrangement at the light-chain loci when reactivity to self antigen occurs during B-cell development. a. Receptor editing b. Somatic hypermutation c. Chromosomal translocation d. Clonal deletion e. Anergy.

a

_______ of thymocytes is necessary to produce a T-cell repertoire capable of interacting with self-MHC molecules. a. Positive selection b. Negative selection c. Apoptosis d. Receptor editing e. Isotype switching.

a

Which of the following proteins is present in specific granules of neutrophils? a) lactoferrin b) calprotectin c) histone d) NADH-oxidase e) a, b, c and d are correct

a (however it has also the other studs but not in the SPECIFIC granules)

One of the preferred viral targets for HIV therapy is: a. reverse transcriptase b. matrix protein c. gp120 d. CD4

a or c

During the course of an attack, B cells can change the class of antibodies they produce? (slide 191) a. True b. False

a. True

The general route for lymphocytes to enter and exit a lymph node would be:

arteries --> HEV --> lymph nodes --> efferent lymphatics -> veins

Monocytes that enter a site of infection differentiate into: a) basophils b) macrophages c) mast cells d) plasma cells

b

The five isotypes of immunoglobulin differ from each other in their _____: a. light-chain constant regions b. heavy-chain constant regions c. light-chain variable regions d. heavy-chain variable regions

b

Where are B cells likely to hang out in a lymph node? (slide 71) a. Marginal sinus b. Cortex c. Paracortex d. Medullary sinus

b. Cortex *primary and secondary lymphoid follicles are located here as well

There are no efferent lymphatics bringing lymph to the spleen. (slide 80) a. true b. false

b. false *There are no AFFERENT lymphatics bringing lymph to the spleen.

All blood cells derive from a common precursor called the ____ cell. a) lymphocyte progenitor b) mast c) hematopoietic stem d) promyeloid

c

The class of antibody recognized by Fc receptors on the surface of mast cells and basophils is: a. IgA b. IgD c. IgE d. IgM

c

The cytokines IL4, IL5, IL13 are produced primarily by what T helper cells? (slide 35) a. Th0 b. Th1 c. Th2 d. Th17

c. Th2

!!!!Fc region of antibody contains:

constant region of heavy chains

Antibody functions include all of the following except:

cross-linking cell-bound antigens on red blood cells when blood cell types are properly matched

!!!! Letters A, B, C and D on the following picture represent: a) A-MHC; B-TCR; C-CD3; D-CD28 b) A-CD28; B-CD3; C-MHC; D-TCR c) A-CD3; B-B7; C-TCR; D-CD28 d) A-B7; B-CD28; C-MHC; D-TCR e) A-B7; B-CD3; C-MHC; D-CD4 or CD8

d

!!!!Which of the following complement fragments is a chemoattractant for neutrophils? a)C3b b) C3d c) C5b d) C5a e) C9

d

IgG3 is the best sub class of IgG in activating complement because: a. It is the most abundant subclass in the blood b. It is dimeric c. It can exchange a module composed of one heavy and light chain with another IgG3 d. It is more flexible due to a long hinge region

d

IgM: (choose one) a. is transferred across the placenta b. exists in a divalent form in mucosal epithelial tissues c. is mainly membrane bound (not secreted) d. is the predominant Ig in primary responses e. is involved in allergic responses

d

What is the only type of hypersensitivity that could occur in a patient with agammaglobulinemia? a. Type I b. Type II c. Type III d. Type IV

d

Choose the T cell signaling molecule that is inhibited by immunosuppressive drug- tacrolimus. a)CD3 b) Lck c) ZAP-70 d) NFAT e) NK-kB

d (fk506 is another name for tacrolimus)

. To innate immunity belong a) cytotoxic T cell b) memory B cell c) NK cell d) neutophil e) c and d are correct

e

What conditions would cause a T cell to be anergized (inactivated)? (slide 10) a. If it fails the positive selection test in the thymus b. If it fails the negative selection test in the thymus c. If the T cell recognizes self but doesn't get co-stimulated d. if the T cell recognizes non-self and gets co-stimulated e. if the T cell recognizes non-self with no co-stimulated

e. if the T cell recognizes non-self with no co-stimulated

Fc Receptors can: a. activate mast cells b. enhance phagocytosis c. activate NK cells d. activate eosinophils e. inhibit the activation of B cells f. all of the above

f

IgG3 activates complement very well and is sensitive to proteases because of its long _____.

hinge region

CD4+

the HLA-D molecules on a macrophage under normal circumstances will display protein fragments produced in other cells. The macrophage would present the viral antigens to _______ T cells

Lacrimal gland

the SIgA present in tears is secreted by which structure?

Western Blotting

the following serologic test uses an enzyme-liked antibody and acrylamide gels

secondary

the lag period during a __________ response is less than 1 day

2

the presence of this molecule means that the T cell that bears it can only bind antigen in the context of Class _______ HLA

IL-4

the second signal for the B cell is IL-2 or ________

B7

the second signal for the T cell is IL-1 or _______

E

the term mononuclear includes which of the following? A: Monocytes B: Dendritic cells C: Macrophages D: Lyphocytes E: All

Degradability

titanium-steel crews used in bone destruction are not good immunogens because they violate which of the following factors contributing to a good immunogen?

The most effective Antigen Presenting Cells are the ____.

Dendritic cells

IgM

During a primary infection this antibody is produced first?

!!!!!Which antibody isotype is only found in membrane form on naive B cells? a) IgA b) IgM c) IgE d) IgG e) IgD

E

B cells a) are the most effective cells for initial activation of naive T cells b) recognize antigen by BCR c) present antigen to T cell d) answers a and b are correct e) answers a and b and c are correct

E

BCR may be composed of: a) IgM b) Igα c) Igβ d) answers b and c are correct e) answers a, b and c are correct

E

Form peptide binding cleft in MHC class II a) α1 and α2 domains b) α1 and β2-microglobulins c) α1 and α3 domains d) β1 and β2 domains e) α1 and β1 domains

E

Granuloma a) is a localized inflammatory response b) consists of multinucleated cells surrounded by Tc cells c) may be formed during sarcoidosis d) answers a and c are correct e) answers a b and c are correct

E

Granuloma a) is localized inflammatory response b) consist of multinucleated cells surrounded by Th cells c) may be formed during Chrons disease d) answers b and c are correct e) answers a and b and c are correct

E

In ER X blocks MHC class II molecule, so that it can not bind peptides and misfolded proteins. X is: a) TAP b) HLA-DM c) proteasome d) calnexin e) invariant chain

E

A malignant tumor is characterized by which of the following features? A) encapsulation B) invasive C) restricted in size D) metastasizes to distant sites E) both b. and d.

E) both b. and d.

Extracellular bacteria are optimally killed by: A. Macrophages B. Complement C. Antibody D. Macrophages plus complement E. Macrophages plus antibody plus complement

E. Macrophages plus antibody plus complement

Which of the following statements about antibodies is false?

Each B cell makes several diverse types of antibodies

Which of the following below is not a component of the Class I MHC pathway?

Endocytosis of extracellular protein

What reacts specifically with an antibody or T-lymphocytes receptors?

Epitope

Describe briefly how affinity maturation of B cells takes place. What is the mechanism(s) and which cells are involved?

Expected answer: Affinity maturation is the process when antibodies with higher affinity for an antigen are selected. B cells that have been activated against an antigen, together with T cell help, will start to proliferate in the germinal center of secondary lymphoid organs. In this process the proliferating B cells will mutated their gene of the antibody with a high mutation rate, somatic hypermutation. The mutations are located primarily to the parts of the gene that code for the variable, i.e. antigen binding, domain of the antibody. This leads to that the original B cell specificity will be changed and some of the B cells clones will increase their affinity to the antigen while some will decrease their affinity. The antigen is presented by follicular dendritic cells and the mutated B cell clones will compete for antigen binding on the FDCs. Only those B cells with the highest affinity will be given a survival signal and will continue to proliferate and to become plasma and memory B cells.

Central tolerance in connected with: a) anti-inflammatory cytokines production b) anergy induction c) lack of response to atypical antigen d) immune privilege e) Treg cells influence f) negative selection

F

The degree to which antigenic determinants are recognized as no self by the immune system is known as

Foreigness

1

How strong is your stomach acid?

The major bonds formed between antigen and antibodies are

Hydrophobic

!!!!Which one is stimulative for eosinophils poliferation?

IL-5

Hapten

Immune response to a prescription drug means that the drug is a __________

Which cells produce cytokines and perform similar functions to CD4 + CD8+ effector cells but do not express TCRs?

Innate lymphoid cells

CD2

LFA-3 LFA-3 on antigen-presenting cells binds to CD2 on T cells, and this interaction provides both adhesive and signaling functions that promote T cell activation.

Dendritic cells constantly sample extracellular fluid and present proteins on their membrane. Where do these dendritic cells then migrate?

Lymph nodes

Graduated pipette is also called _____ pipette

Measuring

What is the middle value in a body of data called?

Median

Safety standards for clinical laboratories are governed by all of these agencies EXCEPT

NAACLS

Microglial cells are macrophages found in the

Nervous tissues

Which of the folllowing molecules is involved in the principal mechanism by which the adaptive immune system fights cancer?

Perforin

D

Physical barriers of innate immunity include all of the following except A: Respiratory mucosa B: Finger nails C: Whiskers D: Peritoneum E: intestinal mucosa

Sharp needles can be disposed of in

Rigid and puncture resistant red biohazard containers

A

Secondary lymphoid organs includes all of the following except: A: Thymus B: Lymph Nodes C: Tonsils D: Spleen E: Peyer's Patches

What signals are provided by the dentritic cells for activation of naïve T cells, and what is the function of these signals?

Signal 1: presentation of the specific antigen via the MHC complex (MHC class I or class II). This signal confers specificity to the T cells activation Signal 2: costimulation via the B7-CD28 interaction, which ensures that T cells are activated only when required Signal 3: polarization (in case of CD4+ T cells)

The first vaccination was created by Dr. Jenner to inoculate against

Smallpox

If bacteria escape from the secondary tissue sites bacteremia develops

True

Immunology is defined as resistance to disease, specifically infectious disease.

True

In a microscopic smear of the pus from bacterial infection site there will be mostlyneutrophilspresent.

True

Memory cells can create IgM and IgG cells

True

Microfluidics can be used to measure neutrophils migration velocity towards a chemoattractant with single cell resolution.

True

Only approximately 10% of the precursor T-cells become mature and enter the bloodstream.

True

PPE is essential in a clinical laboratory setting

True

Patients with quantitative or qualitative defects of neutrophils have a high rate of infections

True

Shoes worn in the lab and phlebotomy should be rubber soled and cover the entire foot

True

The complement system plays an important part in innate immunity to bacteria

True

B-1 B cells produce mainly which isotype of antibody? a. IgA b. IgE c. IgG d. IgM

d

B-2 B cells: a. secrete only IgM b. are found in the medulla of the thymus c. respond primarily to carbohydrate antigens d. give rise to memory B cells.

d

During a typical infection, dendritic cells first encounter pathogen in ____ and then transport the pathogen to ______ where pathogen-derived peptide fragments are presented to _____. a. secondary lymphoid tissue; infected peripheral tissue; T cells b. secondary lymphoid tissue; infected peripheral tissue; B cells c. endocytic vesicles, cell surface; MHC molecules d. infected peripheral tissue; secondary lymphoid tissue, T cells e. infected peripheral tissue; secondary lymphoid tissue; B cells

d

During a typical infection, dendritic cells first encounter pathogen in _____ and then transport the pathogen to ____ where pathogen derived peptide fragments are presented to _____: a. secondary lymphoid tissue; infected peripheral tissue; T cells b. secondary lymphoid tissue; infected peripheral tissue; B cells c. endocytic vesicles; cell surface; MHC molecules d. infected peripheral tissue; secondary lymphoid tissue; T cells e. infected peripheral tissue; secondary lymphoid tissue; B cells

d

Effector memory cells enter _____, whereas central memory cells enter _____. a. B-cell follicles; T-cell zones of secondary lymphoid tissues b. T-cell zones of secondary lymphoid tissues; B-cell follicles c. T-cell zones of secondary lymphoid tissues; inflamed tissues d. inflamed tissues; T-cell zones of secondary lymphoid tissues.

d

Following cytokine binding to cytokine receptors, STAT proteins can cross the nuclear membrane once they have: a. been cleaved by a protease b. been separated into monomers c. undergone a change in conformation to the staple form d. been phosphorylated and form a dimer

d

The body's own tissue are attached as if they were foreign is known as

Auto immune disorder

The total function strength of all interactions between an antibody and its antigen is called?

Avidity

!!!!!Choose the correct sentence/s describing mechanism of somatic mutations? a) it occurs in B as well as in T lymphocytes b) It happens in germinal centre after contact with antigen c) it happens in bone marrow after contact with antigen d) it happens in thymic medulla after contact with antigen e) a, c and d are correct

B

Choose the correct sentence/s I) Endotoxins are intrinsic parts of pathogen II) SC component protects IgA from proteolytic digeston III) Basophils are resident tissue cells that express receptors a) I, II and III are correct b) I and II are correct c) only I is correct d) I and III are correct e)Only III correct

B

Choose the correct sentences describing mechanism of somatic hypermutation? A. Occurs in B as well as in T lymphocytes B. happens in germinal center after contact with antigen C. happens in bone marrow after contact with antigen D. happens in thyme medulla after contact with antigen E. A, C and D are correct.

B

Choose the correct source of TCR diversity I. Point mutations II. Co.stimulant expression III. Somatic recombination IV. preTα expression V. Nucleotide addition at junctions VI.MHC restriction a) I, III, IV b) III, V c) I, II, III d) IV, VI e) II, III f) I, VI

B

Innate

which of the following branches of the immune system produces the most rapid response?

Choose the pro-inflammatory cytokine/s a) TGF-β b) IL-1 c) IL-10 d) answers a and c are correct e) answers a and b are correct

B

Circulating naïve lymphocytes migrate into a)inflamed tissue b)lymph node c)bone marrow d)thymus e)answers b and d are correc

B

Cytokines that activate B cells for antibody production is /are a) IL4 produced by Th1 cells b) IL4 produced by Th2 cells c) TNF produced by Th2 cells d) answers a and c are correct e) answers b and c are correct

B

Which of the following is the first antibody produced in an immune response and multiple binding sites enable higher avidity?

IgM

The diversity of the adaptive immunity and the generation of antibodies can be generatedby the process of VDJ recombination

True

The function of plasma cells is the synthesis and excretion of immunoglobulins

True

The meniscus is the curvature in the top surface of a liquid

True

The series of C5-C8 is what opens the pore in the cell wall

True

Septic shock is caused by the production of large quantities of

Tumor Necrosis Factor

All of the following are useful acute phase proteins EXCEPT

Tumor necrosis factor

Immune complexes of circulating antigens and IgM or IgG antibodies

Type II

IgM and IgG antibodies against cell surface or extracellular matrix antigens.

Type III

CD4+ T cells (TH1 and TH17 cells) and CD8+ CTLs

Type IV

4

Type _____ hypersensitivity does not involve the antibody

IgM

Which antibody is the best at activating (fixing) complement?

*BILDE* 2

Which cells on the picture are designated A and B? 1. A- T-cell, B-macrophage 2. A- B cell, B- Tcell 3. A- Tcell, B-dendritic cell 4. A- dendritic cell, B- Bcell 5. B- Bcell, B-macrophage

C4

Which complement protein is part of the classical complement cascade but not part of the alternative complement cascade?

Type 1

Which hypersensitivity involves an antibody isotope whose concentration in serum is <1%? (IgE)?

IgA

Which immunoglobulin isotope is present in serum concentration less than 1% and does not fix complement?

IgG

Which immunoglobulin predominates in the secondary immune response?

Membrane bound MCP and DAF regulate complement activation by binding to: a) C3bBb b) C9 c) C1 d) Factor P

a

TIL cells are indistinguishable from? a) myeloid suppressor cells b) macrophages c) malignant somatic cells d) dendritic cells e) T lymphocytes

e

CTLA-4 a) binds to the appropriate surface MHC b) induces clonal expansion c) stimulates transcription of IL-2mRNA d) binds B7 molecules e) b and c are correct

D

What are some common portals of microbial entry? A.) Skin B.) Lung C.) GI tract D.) All of the above E.) None of the above

D.) All of the above

What does C5A act on? A.) Phagocytic cells B.) Endothelial cells C.) Neutrophils D.) All of the above E.) None of the above

D.) All of the above

Which of the following is a common pathogen of the intestines? A.) Escherichia coli B.) Clostridium C.) Lactobacillus D.) All of the above

D.) All of the above

Complete activation of the helper T cell takes about 1 to 4 hours. (slide 28) a. true b. false

b. false *Complete activation of the helper T cell takes about 4 to 10 hours.

Complete activation of the helper T cell takes about 10 to 24 hours. (slide 28) a. true b. false

b. false *Complete activation of the helper T cell takes about 4 to 10 hours.

Cytokines have a very diffuse range. (slide 40) a. true b. false

b. false *Cytokines have a very limited range.

During non-battle conditions, T cells need the combination of TGFβ and IL-6 to commit to becoming Th17. (slide 118) a. true b. false

b. false *During battle conditions, T cells need the combination of TGFβ and IL-6 to commit to becoming Th17.

High endothelial venule cells in blood vessels usually overlap like shingles and offer less room for the passage of cells. (slide 75) a. true b. false

b. false *ENDOTHELIAL cells in blood vessels usually overlap like shingles and offer less room for the passage of cells.

Naïve T cells have more lipid rafts than experienced T cells. (slide 24) a. true b. false

b. false *Experienced T cells have more lipid rafts than naïve T cells.

Endothelial cells in blood vessels are columnar and offer more room for the passage of cells. (slide 76) a. true b. false

b. false *HIGH ENDOTHELIAL VENULE cells in blood vessels are columnar and offer more room for the passage of cells. (slide 76)

Helper T cells cannot activate B cells. (slide 65) a. true b. false

b. false *Helper T cells can join in to activate B cells.

Immunological memory is the most important biological consequence of innate immunity. (slide 179) a. true b. false

b. false *Immunological memory is the most important biological consequence of ADAPTIVE immunity.

Lymph nodes, spleen, and mucosal-associated lymphoid tissue (MALT) are primary lymphoid organs. (slide 55) a. true b. false

b. false *Lymph nodes, spleen, and mucosal-associated lymphoid tissue (MALT) are SECONDARY lymphoid organs.

_____________ microorganisms colonize mucosal surfaces but under normal circumstances do not cause disease. A) Opportunistic B) Commensal C) Parasitic D) Pathogenic

B) Commensal

Graves' disease causes _____, whereas Hashimoto's disease causes _____. A) hypothyroidism; hyperthyroidism B) hyperthyroidism; hypothyroidism C) hypoglycemia; hyperglycemia D) hyperglycemia; hypoglycemia E) glomerulonephtitis; systemic vasculitis

B) hyperthyroidism; hypothyroidism

Systemic distribution of cancer cells to other sites of the body through the bloodstream or lymph is a process known as _____. A) apoptosis B) metastasis C) angiogenesis D) neoplasia E) malignant transformation

B) metastasis

__ is the term used to describe how pathogen antigens resemble host antigens and can sometimes trigger autoimmune disease. A) intramolecular epitope spreading B) molecular mimicry C) intermolecular epitope spreading D) sympathetic senescence E) linkage equilibrium

B) molecular mimicry

Th2 cells A) secrete IL-2. B) promote humoral immunity. C) promote protection against intracellular pathogens. D) become committed to the Th2 lineage while developing in the thymus gland.

B) promote humoral immunity.

Which of following events does not occur in the bone marrow? A) rearrangement of Ig genes. B) rearrangement of TCR genes. C) negative selection of immature B cells. D) formation of neutrophils

B) rearrangement of TCR genes.

Which of the following effector functions is not associated with complement? A) opsonization B) regulation of B cell development C) activation of inflammation. D) lysis of target cells

B) regulation of B cell development

Clonal selection A) must occur during B cell development in the bone marrow so that only antigen specific antibodies are formed. B) results in clonal expansion of B cell or T cell lymphocytes that are specific for the stimulating antigen. C) is a critical mechanism of activating cells of innate immunity. D) a. and b. are both correct.

B) results in clonal expansion of B cell or T cell lymphocytes that are specific for the stimulating antigen.

TI-1 antigens A) are processed and presented in MHCII to Th cells. B) stimulate B cells by engaging BCR, co-BCR and TLR. C) activate B cells by extensive cross-linking of BCRs and co-BCRs. D) can induce memory B cell formation.

B) stimulate B cells by engaging BCR, co-BCR and TLR.

Which of the following pairs of terms do not belong together? A) double positive thymocyte/positive selection B) pre-B cell / surrogate light chain C) pro-B cell / somatic hypermutation D) double negative thymocyte / RAG expression

C) pro-B cell / somatic hypermutation

In the class I MHC pathway of antigen presentation, peptides generated in the cytosol are translocated into the endoplasmic reticulum in which of the following ways (A) By ATP-dependent transport via TAP (B) By passive diffusion (C) By receptor-mediated endocytosis (D) Through membrane pores (E) Via the proteasome

(A) By ATP-dependent transport via TAP

Toll like receptors (TLRs) located in endosomal membranes of cells recognize which of the following? (A) Nucleic acids (B) Bacterial cell wall lipotechoic acid (C) Bacterial cell wall lipopolysaccharide (D) Uric acid crystals (E) Peptides containing N-formylmethionyl residues

(A) Nucleic acids

How many kinds of complement pathways are there?

3

Which of the following is a unique property of the adaptive immune system? (A)Highly diverse repertoire of specificities for antigens (B)Self-nonself discrimination (C)Recognition of microbial structures by both cell-associated and soluble receptors (D)Protection against viral infections (E)Responses that have the same kinetics and magnitude on repeated exposure to the same microbe

(A) Highly diverse repertoire of specificities for antigens

Which of the following accurately describe the function of the selectin family of adhesion molecules? (A)Selectins support low affinity rolling of leukocytes on endothelial cells (B)Endothelial selectins increase their affinity for binding to leukocytes in response to chemokines (C)Selectins guide migration of leukocytes though interendothelial junctions (D)Selectins are expressed only on naïve T cells (E)Selectins play a direct role in clonal selection

(A)Selectins support low affinity rolling of leukocytes on endothelial cells

Attenuation of immune receptor signaling in B cells, T cells and NK cells, among others, is mediated by inhibitory receptors that frequently contain ____ in their cytoplasmic tail. (A) ITAMs (B) ITIMs (C) PIP3 (D) Akt (E) CTLA-4

(B) ITIMs

The integrins on blood leukocytes are normally in a _______ state. (A) High-affinity (extended) (B) Low-affinity (bent) (C) Naïve (D) Heterogeneous (E) Homogeneous

(B) Low-affinity (bent)

Which of the following cell types is a phagocyte? (A) Plasma cell (B) Neutrophil (C) Mast cell (D) Natural killer cell (E) T lymphocyte

(B) Neutrophil

Which of the following proteins involved in attenuation of immune signaling become associated with immunoreceptor tyrosine inhibitory motifs (ITIMs)? (A) E3 ubiquitin ligases (B) SH2 domain-containing tytrosine phosphatases (SHP-1, SHP-2) (C) Suppressors of cytokine signaling (SOCS) (D) CTLA-4 (E) PD-1

(B) SH2 domain-containing tytrosine phosphatases (SHP-1, SHP-2)

Which of the following can be accurately called a cytokine? (A) A cell surface antigen receptor on lymphocyte (B) An antibody secreted by a B cell (C) A protein secreted by a T lymphocyte that activates a macrophage (D) A lipid secreted by a Natural Killer cell that activates a B cell (E) A nuclear protein that regulates lymphocyte gene expression

(C) A protein secreted by a T lymphocyte that activates a macrophage

In the cannonical NF-κB signaling pathway downstream of the TNF receptor, TLRs, and antigen receptors, what role does IκB kinase (IKK) complex play? (A) IKK enters the nucleus and phosphorylates NF-κB that has bound to the promoters of proinflammatory genes (B) IKK binds to NF-κB, preventing it from entering the nucleus (C) Activated IKK phosphorylates IκBα, which leads to IκBα degradation, allowing NF-κB to enter the nucleus (D) IKK enters the nucleus and binds to the promoters of pro inflammatory genes (E) IKK phosphorylates NFκB, allowing NF-κB to enter the nucleus

(C) Activated IKK phosphorylates IκBα, which leads to IκBα degradation, allowing NF-κB to enter the nucleus

Clonal selection occurs when antigen is encountered by: (A) Neutrophils (B) Mast cells (C) T-cells (D) Basophils (E) Eosinophils

(C) T-cells

Lipopolysaccharide (LPS) is: (A) Secreted by Macrophages (B) Secreted by Natural Killer Cells (C) Part of the interferon system (D) A component of the outer cell membrane of Gram-negative bacteria such as E. coli. (E) Secreted by Neutrophils

(D) A component of the outer cell membrane of Gram-negative bacteria such as E. coli.

Which of the following cell types is required for all humoral immune responses? (A) Natural killer cells (B) Dendritic cells (C) Cytolytic T lymphocytes (D) B lymphocytes (E) Helper T lymphocytes

(D) B lymphocytes

______ antibodies are the first antibodies produced by B cells in response to a pathogen that has not been encountered before. (A) IgG (B) IgA (C) IgE (D) IgM (E) IgD

(D) IgM

Migration of leukocytes out of the blood into tissues mainly occurs in which type of vessel? (A) Arteries (B) Arterioles (C) Capillaries (D) Venules (E) Veins

(D) Venules

T cell exhaustion: (A) ensures t-cells can kill any nucleated cell that displays that antigen. (B) is mediated by LFA-1 binding to ICAM-1 (C) acts together with TCR recognition of antigen to enhance killing activity. (D) may have evolved as a means to attenuate the tissue-damaging consequences of chronic viral infection. (E) act on the endosomal membrane to facilitate the release of

(D) may have evolved as a means to attenuate the tissue-damaging consequences of chronic viral infection.

G protein-coupled receptors (GPCRs) are: (A) receptors that function by activating associated GTP-binding (B) polypeptides that transverse the plasma membrane seven times, because of which they are sometimes called serpentine receptors (C) able to initiate downstream signaling events (D) are relevant to immunity and inflammation include receptors for leukotrienes, prostaglandins, histamine, complement fragments C3a and C5a, bacterial formyl peptides, and all chemokines. (E) All of the above

(E) All of the above

G protein-coupled receptors (GPCRs) are: (A) receptors that function by activating associated GTP-binding (B) polypeptides that traverse the plasma membrane seven times, because of which they are sometimes called serpentine receptors. (C) able to initiate downstream signaling events. (D) are relevant to immunity and inflammation include receptors for leukotrienes, prostaglandins, histamine, complement fragments C3a and C5a, bacterial formyl peptides, and all chemokines. (E) All of the above.

(E) All of the above.

The cells that are best able to capture, transport, and present antigens to T cells are ____________. (A) Neutrophils (B) Helper T cells (C) Endothelial cells (D) Epithelial cells (E) Dendritic cells

(E) Dendritic cells

Which of the following statements about antigen presenting cells (APCs) is false? (A) APCs display antigens using MHCs on their surfaces to T cell receptors (B) APCs provide signals that guide the T cell on the intensity and nature of its response (C) APCs secrete cytokines that stimulate the T cell (D) APCs include cell types such as macrophages and dendritic cells (E) None of the above statements are false. All of the statements are true.

(E) None of the above statements are false. All of the statements are true.

The process of neutrophils leaving the blood to enter a site of infection is called: (A) Trafficking (B) Opsonization (C) Chemotaxis (D) Migration (E) Transmigration, leukocyte extravasation or diapedesis

(E) Transmigration, leukocyte extravasation or diapedesis

Innate immune response: a) develops earlier than adaptive immune response b) recognizes common structures shared by related microbes c) generates memory cells d) a and b are correct e) a and c are correct

A

Which of the following is NOT a function of the innate immune system? (A)Rapidly respond to microbial infections by promoting acute inflammation (B)Respond to viral infections by inducing the expression of type I interferons (C)Respond to microbial infections by inducing expression of T cell costimulators on antigen presenting cells (D)Respond to damaged and dying host cells by inducing acute inflammation (E)Respond to microbial infections by inducing a state of long lived memory that prevents repeat infections by the same microbe

(E)Respond to microbial infections by inducing a state of long lived memory that prevents repeat infections by the same microbe

Most of the sequence differences and variability among different antibodies are confined to ______. (a) three short stretches in the V region of the heavy chain and to three stretches in the V region of the light chain. (b) three short stretches in the V region of the heavy chain. (C) three stretches in the V region of the light chain. (D) the transmembrane region. (E) Cytoplasmic tail.

(a) three short stretches in the V region of the heavy chain and to three stretches in the V region of the light chain.

Process of somatic hypermutations a) always leads to improved affinity of Ig to antigen b) occurs in lymph node IgM+IgD+Bcells c) occurs in bone marrow IgM+IgD+B cells d) answers a and b are correct e) lack of correct answer

(a: not all of them will get higher affinity, they will not be selected.

Please name the cells that act as professional antigen presenting cells, and describe for each of these cells why they need to/the out come of, an interaction with a specific T cell.

-Dendritic cells (T cell activation) -Macrophages (to be activated by Th1 for an enhanced anti- microbial activity) -B cells (to receive survival signals from Th2 both for early production of IgM and formation of the germinal center and for class switch) -Thymic ephitelial cells (positive selection)

Which molecules are matched before organ transplantation? Why is it important with matching donor and recipient? Explain the mechanisms behind graft versus host disease, including central molecules and cells.

-HLA-typing (HLA-A, -B, -DR), match blood groups (ABO), (not necessary for the answer but correct and answered by some; test for cross-reactivity to donator cells in vitro, check for infections). -to avoid graft rejection via antibodies and/or T cells, and/or graft versus host disease via T cells. -GVHD-activation of mature donor CD8+ T-cells in the graft. A pool of mature donor CD8+ T cells will be auto-reactive against the recipients MHC class I molecules if those are mismatched, i.e. the differences in the MHC class I heavy chains will be sufficient for the donor T cells to recognize the MHC class I complex as foreign, be activated and kill the cells. If the MHC class I is matched, there will/may be a weaker GVHD due to recognition of the MHC class I bound peptides as foreign (unless genetically identical individuals, i.e. monocygotic twins).

Can you list at least three distinctive features of the MHC class I, and MHC class II molecules?

-MHC class I: one chain non-covalently linked to β2-microglobulin; it binds 8-10 aa long peptides: it is expressed in all cells (except red blood cells); activate CD8 T cells; present mainly endogenous antigens -MHC class II: two chains (a and b); it binds longer peptides (around 20 aa long, up to 30 aa); expression limited to dendritic cells, B cells, macrophages and thymic epithelial cells; activate CD4 T cells; present exogenous antigens

!!!!how many heavy chains in IgM pentamer:

10

How many binding sites does IgM have

10

The approximate percentage of T lymphocytes in the thymus is—-%

100%

A 1:10 dilution of a specimen means that for every unit of specimen there is/are

9 units of solvent

What percentage of the values will normally fall within two standard deviation on the Gaussians curve?

95%

!!!!!azurophilic granules contain: a) Mieloperoxidase b) lactoferrin c) Protectin d) TNF e) all of above

A

!!!!CD14 is a marker of which cells? a) monocytes b) B cells c) NK cells d) T cytotoxic e) T helper

A

!!!!Fill in the sentence; Langerhans cells are _____________ located in the epidermis of the skin? a) immature dendritic cells b) mature dendritic cells c) monocytes d) macrophages e) NK cells

A

!!!!Part of BCR/TCR binding antigen is called: a) paratope b) Epitope c) Agretope d) antigen dominant e) anchoring amino-acid

A

!!!There is a laceration-contaminated with Staphyloccocus, Which cells will respond first? a) Neutrophils b) Dendritic cells c) B cells d) T cells e) platelets

A

Acute infection with persistent and continuous or intermittent shedding of viruses is caused by a) Epstein Barr virus b) HIV c) influenza virus d) answer a and b and c are correct e) lack of correct answer

A

Antigens that bind to MHC class I are cut by X and delivered into ER by Y a) X - proteasome, Y - TAP b) X - proteasome, Y - HLA-DM c) X - TAP, Y proteasome d) X - MIIC, Y - TAP e) none

A

CD28 a) is a costimulatory molecule of Th cells b) is a coreceptor of Tc cells c) is a costimulatory molecule of Tc cells d) is a coreceptor of Th cells e) is a coreceptor of Tc and Th cells

A

CD28 a) is expressed on T cell b) transmits inhibitory signal to T cell c) may be expressed on APCs d) binds to CD40 on APCs e) answers a and b are correct

A

Choose the components that can take part in ADCC a) FcR b) CR1 c) CD8 d) answers a and c are correct e) lack of correct answer

A

Choose the composition/s that can take part in ADCC a)FcR b) CR1 c) CD8 d) a and c are correct e) none

A

Component of innate immune response include a) complement proteins b) antibodies c) T cells d) a and b are corerct e) b and c are correct

A

Components of adaptive immune response include: a) T cells b) NK cells c) macrophages d) a and b are correct e) b and c are correct

A

Costimulatory signalling and IL 2 production a) are required for activation of naive T cells b) are required for activation of effector t cells c) are reuired for macrophages to produce IFN-y d) a and c are correct e) a b and c are correct

A

Differences between constant regions of Ig due to usage of different C-region genes are called a) isotypes b) allotypes c) paratopes d) epitopes e) idiotypes

A

Fill in the sentence: During the somatic recombination of Ig/TCR gene segments a gene segment with ______ nucleotides spacer can be joined only with gene segment with a 12 nucleotides spacer. This reaction is mediated by ____ A. 23, RAG B. 12, TdT C. 12, RAG D. 23, TdT E. lack of correct answe

A

Find the correct sentence/s about junctional diversity a) P nucleotides are added to the shorter strand of DNA to repair asymmetric breaks. b) P nucleotides are added at random to the shorter strand of DNA to repair asymmetric breaks. c) P nucleotides are added to the longer strand of DNA to repair asymmetric breaks. d) P nucleotides are added at random to the longer strand of DNA to repair asymmetric breaks. e) lack of correct answer

A

Finn in the sentence; The area of T cell: APC contact is divided into the central supramolecular activation complex, that contains ____ and the peripheral supramolecular activation complex, in which different _____ are segregated. a) signaling molecules, adhesion molecules b) adhesion molecules, signaling molecules c) p-SMAC, signalling molecules d) c-SMAC, signalling molecules e) lack of correct answer

A

Form peptide binding cleft in MHC class I a) α1 and α2 domains b) α1 and β2-microglobulins c) α1 and α3 domains d) β1 and β2 domains e) α1 and β1 domains

A

Immunoglobulin is bound to B cell through a) transmembrane domain b) Fab fragment c) Fc fragment d) Igα and Igβ e) framework subregions

A

Immunoglobulin is bound to B cells through X and is bound to different cells (other than B) through Y. X and Y refer to a) X -transmembrane domain, T - Fab fragment b) X - Fab fragment, Y Fc fragment c) X - Fc fragment, Y Fc fragment d) X - transmembrane domain, Y - transmembrane domain e) X - BCR, Y Fc fragment

A

protein

A cell is infected with herpes simplex virus. You would expect viral antigens to be displayed on HLA molecules after virus __________ synthesis begins

Monocytes spend how many days in circulation, and then cross the endothelium to enter tissues throughout the body, where they reside up to several months as macrophages? A.) 1-2 days B.) 4 days C.) 4 weeks D.) 4 months

A.) 1-2 days

1

A T cell with CD8 on its surface can only bind antigen presented in the context of Class _____ HLA

What is a common pathogen of the eye? A.) Branhamella B.) Lactobacillus C.) Escherichia coli D.) Clostridium

A.) Branhamella

Chronic Granulomatous disease

A defect in the enzymes of this respiratory burst results in this immunodeficiency condition

Compare and contrast dendritic cell and macrophage functions

A good answer here would describe the similarities such as class II and costimulatory molecules expression (with DC having more constitutive expression) and their role as antigen presenting cells, they both have specialized types depending on their location in the body (Langerhan cells=skin DC, Kupffer cell= Liver macrophages). Some of thedifference would be that DC are superior at stimulating naïve T cells than macrophages, DC readily migrate to draining lymph nodes while macrophages often remaining in the tissues. DC can perform macropinocytosis constitutively whereas macrophages must be induced to perform. Otherwise both cells exhibit the same type uptake. Both cell types make the same type of cytokines but maybe at different levels such as IL-12 and IL-18, TNF etc. Macrophages better at making NO and ROS than DC One could also bring up pDC and say how they differ from both normal DC and macrophages

Some species have restricted antibody- or MHC repertoire if we compare to humans. Why do you think that these species have a restricted repertoire? E.g. cartilage fish has three MHC and predominantly only IgM and new world monkeys which predominantly have HLA-G MHC molecules. Speculate based on your knowledge in immunology and microbiology. What are the pros and cons of the expanded repertoire of MHC molecules and T and B cell receptors in humans?

A good answer should contain an understanding and a discussion of the function of the adaptive immune system, that it is partly shaped by the evolutionary pressure from pathogens and the environment the species are inhabitants in. Pros of an expanded repertoir of MHC molecules and T and B cell receptors can be that the adaptive immune system is able to recognize a larger number of pathogens that can be of benefit both on an individual and a population level. Cons are that more T and B cell receptors (cells) will be useless and in the case of MHC that individuals might have a less good respons to some pathogens.

Briefly describe how the innate immune and adaptive immune systems communicate with each other and explain why this is beneficial to responses against pathogens.

A good answer should contain that the innate and adaptive immune system can communicate via APC to T cells, via cytokines from innate cells to adaptive cells and vice versa, via antibodies (adaptive) to Fc-receptor expressing cells (mast cells and eosinophils - IgE, NK cells - ADCC, etc), via antibodies to the complement system, via chemokines from innate cells to attract adaptive cells and vice versa. Extra bonus for correct examples. An efficient communication between the innate and adaptive immune system is necessary for efficient, initiation, activation, direction, effectuation and termination of a protective immunity against pathogens.

Describe three examples of innate immune receptors in terms of their role in host defense against pathogens, i.e. the specificity of the receptor and what the outcome is when the receptor is engaged. Chose one cell type per receptor when answering, you are free to choose which cell type to represent the function of which receptor and you do not need to have different cells for all three examples of receptors if you wish not.

A good answer should contain three receptors, which cells they are expressed on and what the specificity of the receptors are and what happens when the receptor is engaged. Many possible correct answers. OK to have more than one receptor from one family of receptors. OK receptor families are: Scavenger receptors, e.g. Mannose binding protein Pattern recognition receptors, e.g. TLR receptors NK receptors, e.g. KIR, NKp46, NKG2D Complement receptors Fc-receptors not considered to be innate since they are dependent on antibodies, i.e. adaptive components.

Describe how NK cells recognize infected or foreign cells, i.e. how they decide to kill or not to kill those cells. How do you think that this knowledge can be used clinically?

A good answer should describe that NK cells are regulated by a balance between activating and inhibitory receptors. One important activating receptor recognize stress induced ligands on target cells, NKG2D, which associates with the adaptor molecule DAP12 or DAP10 to signal. Ligands are upregulated on target cells by viral infections, tumor formation, or other stress situations for the cells. Another activating receptor is CD16, the Fc receptor that bind the Fc part of antibodies and can induce ADCC by the NK cells.At the same time NK cells are inhibited by receptors from the Killer Immunoglobulin- like Receptors (KIR). This family of receptors are specific for, recognize, MHC class I molecules and contain both activating and inhibitory receptors. NK cells are educated to utilize inhibitory receptors specific for self MHC class I molecules, i.e. they are not allowed to be functional if they cannot be inhibited by self MHC class I molecules. NK cell killing of cells are regulated by the balance between the activating and inhibitory receptors. Absence of MHC class I leads to lack of inhibition and therefore the activating receptor signaling with dominate and activate the NK cells (missing self). NK cells can also be activated by an increase of activating signals, i.e. upregulation of stress molecules on tumor cells, or recognition of antibody coated cells, which could overcome the inhibitory signals. Clinically this knowledge can be used to either match NK cells that are from a person with another MHC haplotype to treat lymphoma, i.e. that one specifically take NK cells that should react with foreign cells to get both a missing self and a anti-tumor effect simultaneously to get an efficient rejection of the lymphoma. Or that one can either block the inhibitory receptors to induce killing of unwanted cells (e.g. lymphomas), alternatively increase activation by e.g. antibodies to get an efficient ADCC.

MHC class I and MHC class II molecules have peptide specificity. What does peptide specificity mean? How does this differ between MHC class I and MHC class II molecules?

A good answer should include that the peptide is restricted to specific MHC molecules with different binding regions (binding pockets) in the peptide binding groove. The peptide binding groove differ by that MHC class I bind short peptides of 8-10 aa (there occasionally longer peptides binding, but the anchoring residues are the same but withends of the peptide sticking out from the groove). MHC class II longer peptides, appr 15- 22 aa.

Type 3

A patient receives rattlesnake anti-venom and has never received any type of anti-venom or anti-toxin before. What type of hypersensitivity will develop?

Which statement best describes the structure and composition of an antibody A. Antibodies are protein structures composed of four polypeptide chains: two heavy chains and two light chains B. Antibodies are carbohydrate structures composed of four polysaccharide chains: two heavy chains and two light chains C. Structures composed of two polypeptide chains: one heavy chain and one light chain D. .Antibodies are carbohydrate structures composed of two polysaccharide chains: one heavy chain and one light chain. E. .Antibodies are both protein and carbohydrate structures with two heavy protein chains and two light polysaccharide chains.

A. Antibodies are protein structures composed of four polypeptide chains: two heavy chains and two light chains

Which of the following is NOT a mechanism of self tolerance: A. Down regulation of T cell receptor expression on self reactive T cells in the periphery B. Deletion of self reactive cells in the thymus C. T cell anergy in the periphery D. Generation of regulatory T cells in the thymus

A. Down regulation of T cell receptor expression on self reactive T cells in the periphery

Immunologic tolerance can best be defined as: A. Failure to mount an immune response to a particular antigen B. Immune cells mounting a response against the body's own cells or tissues C. An outbreak of a disease that simultaneously affects a large number of people D. None of the above E. All of the above

A. Failure to mount an immune response to a particular antigen

Toll-like receptors (TLRs) are a family of homologous receptors expressed on many cell types and are involved in innate immune responses. Some TLRs are expressed on the plasma membrane, Others are expressed on endosomal membranes indie the cells. Which of the following is a shared property of the TLRs in the endosomes? A. Recognition of nucleic acids B. Recognition of bacterial cell wall molecules C. Signaling by JAK-STAT pathways D. Inhibition of inflammatory responses E. Secreted into the blood

A. Recognition of nucleic acids

What cell adhesion protein is found on endothelial cells and binds to ligands on the neutrophil? This cell adhesion protein functions like Velcro to grab neutrophilsin orderto slow them down. The neutrophils are then described as "rolling." A. Selectin B. Intigrin C. ICAM D. GPCRs E. TNF

A. Selectin

Complement damage is generally limited to the immediate area in which complement isactivated because of the: A. short half-lives of the activated complement components and their rapid inactivation. B. very low concentrations of the inactivated complementcomponents in serum. C. the inability to activate the system in the presence of IgG antibodies. D. once activated, the destructive activities of complement are non-specific. E. none of the above

A. short half-lives of the activated complement components and their rapid inactivation.

Which of the following are examples of passive immunity? First breast milk Colostrum Acquired naturally by the fetus All of the above

All of the above

Which pipette are used in the lab? Graduated Serologic Volumetric

All of the above

Type 4

Allergy to soap is due to which of the following hypersensitivities?

Who first described inflammation?

Aulus Cornelius Celsus

CD4+ T cells that respond to intracellular pathogen by recruting and activating phagocytic cells are termed A. antigen-presenting cells B. cytotoxic T cells C. Th0 cells D. Th1 cells E. Th2 cells

D

Antigen presenting cells that can take up antigen specifically are the _____.

B cells

What is the responding lymphocyte in humoral immunity?

B lymphocytes

The final step to the complement system is: A. C3 convertase cleaves C3 into C3a and C3b B. A complex of complement protein assembles into a pore in the microbe's membrane C. The chemoattractant C5a is produced and cause blood vessels to leak plasma proteins and fluid into sites of infection D. Mannose-binding lectin recognizes terminal mannose residues on microbial glycoproteins and glycolipids E. The microbe is opsonized with C3b

B. A complex of complement protein assembles into a pore in the microbe's membrane

Which of the following is an enzymatic function of most types of inflammasomes? A. Generate active granzyme B from an inactive precursors B. Generate an active form if the inflammatory cytokine IL-1β from an inactive precursors C.Proteolytically inactivate caspase 1 D.Degrade dsDNA E.Phosphorylate interferon regulatory factors (IRFs)

B. Generate an active form if the inflammatory cytokine IL-1β from an inactive precursors

When would integrin on a leukocyte most likely be found in the extended state? A. Integrin is usually found in the high affinity, extended state. B. Leukocytes are slowed to "rolling" speeds so that chemokines on the endothelial cells bind to receptors on the leukocyte activating the integrin. C. When integrin first captures the leukocyte causing it to roll down the endothelium D. After migration through the endothelial cells E. While the leukocyte is freely moving through the blood vessel

B. Leukocytes are slowed to "rolling" speeds so that chemokines on the endothelial cells bind to receptors on the leukocyte activating the integrin.

Naïve B and T cells migrate preferentially to the A. Venules B. Lymph nodes C. Bone marrow D. Thymus E. Infected tissue

B. Lymph nodes

Tissue macrophages are derived from which type of circulating blood cell? A. Polymorphonuclear leukocyte B. Monocyte C. Small lymphocyte D. Basophil E. Lymphoblast

B. Monocyte

All of the following statements describe a neutrophil except: A. Neutrophils are phagocytic cells B. Neutrophils are the least abundant type of circulating white blood cell C. Neutrophils are classified as innate immune cells D. Neutrophils mediate acute inflammatory responses to bacterial infections E. All of the above statements correctly describe a neutrophil

B. Neutrophils are the least abundant type of circulating white blood cell

Which of the following comparisons of the innate and adaptive immune systems is FALSE? A. The innate immune system is more likely to recognize normal self, and therefore cause autoimmunity, than is the adaptive immune system. B. Receptors used for recognition in innate immunity are encoded in the germline, whereas those of the adaptive immune system are encoded by genes generated via somatic recombination of germline receptor gene loci. C. The innate and adaptive immune systems share some of the same effector mechanisms. D. Both the innate and adaptive immune systems can recognize nonmicrobial substances. E. The innate immune system less long lived memory than the adaptive immune

B. Receptors used for recognition in innate immunity are encoded in the germline, whereas those of the adaptive immune system are encoded by genes generated via somatic recombination of germline receptor gene loci.

The technique of ELISA can measure the following parameter: A. The RNA level inside the cell nucleus. B. The protein level of a secreted cytokine. C. The DNA structure inside a mitochondria. D. The lipid structure of an endosome.

B. The protein level of a secreted cytokine.

In type 1 diabetes (insulin-dependent diabetes mellitus), the target of the autoimmune attack is: A. All of the cells in the islets of Langerhans. B. The β-cells in the islets of Langerhans. C. The somatostatin-producing cells in the islets of Langerhans. D. The glucagon-producing cells in the islets of Langerhans. E. Cells throughout the body which have an insulin receptor.

B. The β-cells in the islets of Langerhans.

Which of the following is a proinflammatory cytokine of major importance in innate immunity that has been successfully targeted by drugs to treat rheumatoid arthritis (RA)? A. Tumor necrosis factor (TNF) B. Transforming growth factor-β (TGF-β) C. Interleukin 10 D. Interleukin 2 E. Interferon γ

B. Transforming growth factor-β (TGF-β)

The ability to modify the immune response is the basis for immunologic memory, the hallmark distinguishing the ___ from the ___ system. A.) Complement; Activation B.) Adaptive; Innate

B.) Adaptive; Innate

What three antimicrobial peptides are secreted by many cell types within the skin, and all act directly on the microbes damaging the membranes causing lysis; they act as chemoattractants for cells of the innate system and facilitate ingestion and destruction of microbes by phagocytosis. A.) Gamma, Mu, and Episolon defensins B.) Alpha- defensins, Beta-defensins, and cathelicidin

B.) Alpha- defensins, Beta-defensins, and cathelicidin

What do plasma cells differentiate from? A.) T-cells B.) B-cells C.) CD4 T-cells D.) CD8 T-cell

B.) B-cells

What is the major opsonin of the complement system? A.) C5 convertase B.) C3b C.) Cq1 D.) Membrane Attack Complex

B.) C3b

What receptors are involved in phagocytosis? A.) Killer Activation Receptors B.) Clatherine-Coded Pits C.) Killer Inhibition Receptors D.) All of the above

B.) Clatherine-Coded Pits

What type of cells are bi-lobes, and mainly found in parasitic or worm infections? A.) Neutrophils B.) Eosinophils C.) Basophils D.) All of the above

B.) Eosinophils

Which fragment is constant? A.) Fab B.) Fc C.) Antigen-binding D.) Antigen-Binding & Fab

B.) Fc

A 7 year old girl has a history of peanut allergy with symptoms that include generalized itching and hives after eating peanuts. Her symptoms became more severe with subsequent accidental exposure to peanuts. For this child, a peanut is most likely a(n): A.) Hapten B.) Immunogen C.) Innate immune system antigen D.) Toleragen

B.) Immunogen

What type of cells make up 60% of peripheral blood leukocytes, and are multi-lobed, and take about 2 weeks to mature? A.) Basophils B.) Neutrophils (aka PMN's) C.) Eosinophils D.) a & b E.) None of the above

B.) Neutrophils (aka PMN's)

The final step to the complement system is: A.C3 convertase cleaves C3 into C3a and C3b B.A complex of complement protein assembles into a pore in the microbes membrane C.The chemoattractant C5a is produced and cause blood vessels to leak plasma proteins and fluid into sites of infection D.Mannose-binding lectin recognizes terminal mannose residues onmicrobial glycoproteins and glycolipids E.The microbe is opsonized with C3b

B.A complex of complement protein assembles into a pore in the microbes membrane

It has been postulated that regulatory T cells reduce available B7 costimulators on APCs, and this is one possible mechanism by which the Tregs inhibit immune responses. Which of the following molecules is expressed on regulatory T cells (Treg) and functions to block B7-mediated costimulation A.PD-1 B.CTLA-4 C.CD28 D.CC127 (IL-7 receptor) E.Fas ligand

B.CTLA-4

How do gram-positive bacteria and some fungi evade the complement system? A.Produce specific proteins that mimic human complement regulatory proteins. B.Express thick walls that prevent the binding of complement proteins, such as MAC. C.Complement mediated inflammation. D.Recruiting host complement regulatory proteins E.None of the above.

B.Express thick walls that prevent the binding of complement proteins, such as MAC.

What cell adhesion protein-ligand pair creates a very strong interaction stopping the neutrophil from rolling. A.Selectin-VCAM B.Integrin-ICAM C.C5a-LPS D.GPCR-Cell Adhesion Ligand (CAL) E.Arrestin-C5a

B.Integrin-ICAM

These dome like structures are found in the small intestine. They are enriched in lymphoid tissue making them key sites for coordinating immune responses to pathogens. They also help promote tolerance to harmless microbes and food. A.Germinal centers B.Peyers patches C.IRCs (Immune response centers) D.VilliE.Mesenteric lymph node

B.Peyers patches

Which of the following statements about negative selection during lymphocyte development is most accurate? A.The mechanism of negative selection is the death of cells whose antigen receptors are not stimulated by self antigens. B.The mechanism of negative selection is apoptosis actively induced by antigen-receptor recognition of self antigens. C.Negative selection ensures a self MHC restricted repertoire of T lymphocytes. D.Negative selection occurs only in T cell maturation and not in B cell maturation. E.The major mechanism of negative selection is inhibition of antigen

B.The mechanism of negative selection is apoptosis actively induced by antigen-receptor recognition of self antigens.

The technique of ELISA can measure the following parameter: A.The RNA level inside the cell nucleus. B.The protein level of a secreted cytokine. C.The DNA structure inside a mitochondria. D. The lipid structure of an endosome.

B.The protein level of a secreted cytokine.

CTLA-4

B7-1, B7-2 B7-1 and B7-2 on antigen-presenting cells bind to CTLA-4 on activated T cells, stimulating negative signals in the T cells.

natural killer cells

Based only on cells listed below, which cell type is best known for both identifying and destroying virus-infected cells? -Natural killer cells -erythrocyte -macrophage -lymphocyte

The classic complement pathway is activated by

Binding antigen-antibody complexes

Which of the following is required for the sensitization of mast cells?

Binding of antibody to high-affinity Fc receptors on the mast cell

Describe three different lymphoid organs and their function in the immune system.

Bone marrow (BM); where pluripotent stem cells differentiate to the different leukocytes, including thymocytes that go to the thymus to become T cells, and B cells that will mature in the BM. Also maturation of NK cells OK.Lymph nodes (LN); where activated APCs (mainly DC) migrate after taking up antigen to present for naïve T cells, both CD4 and CD8, to activate and mature the T cells to effector cells. B cell germinal centers where naïve B cells are activated to mature and proliferate, and later to isotype switch and affinity maturation. Lymphatic vessels; antigen and immune cells migrate, circulate, via the lymphatic vessels to the peripheral lymph organs, e.g. lymph nodes etc. Lymph is drained into the blood system. Spleen; where blood-borne antigens are taken care of. Many phagocytic cells, e.g. macrophages and DCs that take up antigen, and also degrade red blood cells. T cell and B cell compartments, similar functions as LN.

!!!!call the process in which phagocytes eliminate apoptotic bodies: a) netosis b) necrosis c) eferocytosis d) neutralization e) osmotic lysis

C

!!Which cytokines induces IgA class switching? a) IL-4 b) IFN-γ c) TGF- 𝛽 d) a, b and c are correct e) no correct answer

C

A state of t-lymphocytes nonresponsiveness that occurs following peptide + major histocompability complex engagement is known as a) allergy b) apoptosis c) anergy d) autoimmunity e) hypersensitivity

C

CTLA-4 a) is costimulatory molecule b) binds to B7.1 or B7.2 on T cell c) transmits inhibitory signal to T cell answers a and b and c are correct e) answers a and c are correct

C

Fragments of cytoplasmic pathogen are presented by T cells by: A. direct arrangement of cell surface pattern recognition receptors B. endocytosis C. MHC class I molecules D. phagocytosis E. B cell receptor.

C

In a patient who later developed an allergy to a certain antigen, the initial response to the antigen consisted of immunoglobulin of the IgM class. However, over time antigen-specific IgE came to be predominant. This change form an IgM to an IgE response is caused by: A. affinity maturation B. allelic exclusion C. isotype switching D. junctional diversity E. somatic hypermutation

C

Naive T lymphocytes are characterised by: a) low expression of lymph node homing receptor b) expression of CD45RO c) no expression of CD25 d) efficient proliferation e) migration to inflamed tissues f) high expression of integrins

C

Name tissue resident cells with cytoplasmic granules that produce heparin, histamine and other mediators involved in inflammation. a) basophil b) B cell c) mast cell d) macrophage e) dendritic cell

C

Process of somatic hypermutations a) leads to junctional diversity b) occurs in bone marrow c) depends on CD40-CD40L interaction d) answers a and b are correct e) answers b and c are correct

C

Th2 cells a) secrete IFNy b) differentiate upon IL12 activation c) need GATA-3 transcription factor d) answer a and bb are correct e) answers a and b and c are correct

C

The process of somatic recombination of TCR gene segments a) depends on self antigen b) depends on foreign antigen c) depends on RAG proteins d) answers a and c are correct e) answers b and c are correct

C

These cells combat microbes by recruiting leukocytes, mainly neutrophils, to sites of infection. They play an especially important role in defense against extracellular bacteria and fungi. This is the description of? a) Th2 cells b) Tc cell c) Th17 cell d) Treg cells e) B cells

C

Which molecule expressed by T cell has the same function as ig(alpha) / ig(beta) expressed by B cells A. CD4 B. CD28 C. CD3 D. TCR E. INTEGRIN

C

Which molecule expressed by T cells has the same function on igG(?) expressed by B cells? A. CD4 B. CD25 C. CD3

C

Which molecule is part of B cell co-receptor complex? a) CD3 b) CD8 c) CD19 d) CD16 e)answers b and d are correct

C

Which of the following cells have been implicated in the prevention of autoimmune response? A. Antigen-presenting cells B. Th 17 cells C. CD4+ CD25 + Treg cells D. plasmacyted dendritic cells E. naive T cells

C

Which of the following complement fragments is a component of membrane attack complex (MAC)? a) C3b b) C3d c) C5b d) C5a e) a and c are correct

C

Which of the following does not apply to "innate" immune mechanism? A. Absence of specificity B. acid pH in the stomach C. a memory component D. involvement of phagocytes E. active of lyzozyme

C

Which of the following sentences about CTLA-4 is/are correct? I. CTLA-4 is an inhibitory receptor that binds B7 with higher affinity than CD28 II. CTLA-4 is expressed only on naive T-cells III. CTLA-4 blocks T cell anergy IV. CTLA-4 signaling induces expression of CD28 molecules. A. I, II, III and IV are correct B. II and III are correct C. I is correct D. II and IV are correct E. IV is correct

C

What type of cell is third in line in lymphoid lineage and are large, nonphagocytic granular lymphocytes named for their ability to kill abnormal host cells? A.) B cells B.) T cells C.) Natural Killer (NK) cells D.) None of the above

C.) Natural Killer (NK) cells

Which statement about lymphocyte migration is true? a) Naive lymphocytes migrate to lymph nodes via afferent vessels. b) Naive lymphocytes migrate to inflamed tissues via high endothelial venules (HEV). c) Memory lymphocytes migrate to inflamed tissues due to integrins mediation. d) Memory lymphocytes migrate to lymph nodes via efferent vessels. e) Effector lymphocytes migrate to peripheral tissues due to L-selectin interaction. f) Effector lymphocytes migrate to lymph nodes due to E- and P-selectin interaction. (they migrate to inflamed tissue)

C

Poly-Ig receptor a) does not transport IgA through epithelial cells b) is composed of Iga and IgB proteins c) has the same function as CD3 in T cells d) answers b and c are correct e( answers a and b and c are incorrect

C (A: It does transport IgA through epithelial cells. B: composed of IgA and IgM. The J chain of secreted dimeric IgA and pentameric IgM contains a domain required for poly-Ig receptor binding)

RAG proteins a) are the costimulatory molecules b) add nucleotides at junctions during process of somatic recombination c) are expressed only in T and B lymphocytes d) answers a b and c are incorrect e) answers b and c are correct

C (B: thats TdT)

The antibody with no hinge region a) IgA b) IgG c) IgE d) IgD e) all antibody isotypes have hinge regio

C (ME=no hinge)

Which of the following is responsible for changing the antigen binding site of a B cell receptor AFTER stimulation with foreign antigen? A. junctional diversity B. combinatorial diversity C. somatic hypermutation D. proliferation E. lack of correct answer.

C (Somatic hypermutation occur in the dark zone, after B cell has encountered an antigen)

Type I interferons a) are IFNa, IFNB and IFNy b) increase the expression of MHC molecules on APC c) activate NK cells d) answers a and b and c e) answers b and c are correct

C (a: IFN-gamma er Type 2 inferferon)

Name the proteins that binds to microbes/antigens and serves as a ligand for b cell coreceptor (CR2) that enhances its response to antigen. a) C3a b) C4b c) C3d d) CRP e) CD3

C (B cells express a complex of the CR2 complement receptor, CD19, and CD81. Microbial antigens that have bound the complement fragment C3d can simultaneously engage both the CR2 molecule and the mem- brane Ig on the surface of a B cell. This leads to the initiation of signaling cascades from both the BCR complex and the CR2 complex, because of which the response to C3d-antigen complexes is greatly enhanced compared with the response to antigen alone.)

Which of the following statements about mycobacterium tuberculosis is correct? I) It belongs to extracellular pathogens its intracellular II) It infects macrophages III) one of its antigens belongs to Hsp proteins IV) The activity of Th2 cells is required to combat the infection V) It prevents acidification of the phagosome, true a) statement I, IV and V are correct b) statement II, III and V are correct c) only statement II is correct D) statements II and III are correct E) statement I, II, III, IV and V are correct

C (HOWEVER V IS ALSO CORRECT)

CD4: a) is a costimulatory molecule of Th cells b) is a corceptor of Tc cells c) is a costimulatory molecule of Tc cells d) is a coreceptor of Th cells e) is a coreceptor of Tc and Th cells

D

What type of adhesion molcules are limited to their tissue distribution and are designed to identify particular tissues and facilitate the interactions of particular cell combinations? A.) Selectins B.) Addressins C.) a & b D.) Integrins

C.) a & b

Helper-T cells are __________-positive cells and can present to MHC _________ proteins on the surface of ___________________. A. CD4+, Class I, all nucleated cells B. CD4+, Class II, all nucleated cells C. CD4+ Class II; B-lymphocytes, macrophages, and dendritic calls D. CD8+, Class I, all nucleated cells E. CD8+, Class I, B-lymphocytes, macrophages, and dendritic cells F. CD8+, Class II; B-lymphocytes, macrophages, and dendritic cells

C. CD4+ Class II; B-lymphocytes, macrophages, and dendritic calls

Helper-T cells are __________-positive cells and can present to MHC _________ proteins on the surface of ___________________. A.CD4+, Class I, all nucleated cells B.CD4+, Class II, all nucleated cells C.CD4+ Class II; B-lymphocytes, macrophages, and dendritic callsD.C D8+, Class I, all nucleated cells E.CD8+, Class I, B-lymphocytes, macrophages, and dendritic cells F.CD8+, Class II; B-lymphocytes, macrophages, and dendritic cells

C. CD4+ Class II; B-lymphocytes, macrophages, and dendritic callsD.C

The principal function of the immune system is: A. Defense against cancer B. Repair of injured tissues C. Defense against microbial infections D. Prevention of inflammatory diseases E. Protection against environmental toxins

C. Defense against microbial infections

The receptors for the innate immune system are A. encoded by genes produced by somatic recombination of gene segments B. May recognize nonmicrobia antigens C. Encoded in the germline D. IgG E. TCR

C. Encoded in the germline

Which of the following is a feature of Natural Killer (NK) cells? A.They express clonally distributed antigen receptors that directly bind antigens on the surface of microbes B.They are activated by recognizing microbial peptides bound to host class I MHC molecules C.They kill virally infected cells by a perforin/granzyme dependent mechanism D.Upon activation, they secrete abundant interleukin-4 E.They secrete natural IgM antibodies

C. They kill virally infected cells by a perforin/granzyme dependent mechanism

Which of the following statements about the recognition receptors of the innate immune system, such as Toll like receptors (TLRs), is correct? A.They are encoded by genes produced by somatic recombination of gene segments that are separated in inherited germline DNA B.Each clone of macrophages express a unique set of these receptors that differs in specificity from those receptors on all other clones of macrophages C.They recognize pathogen associated molecular patterns, such as viral nucleic acids, bacterial cell wall constituents, and terminal mannose residues. D.They function to neutralize microbes, but do not initiate signal transduction pathways that activate cells. E.They undergo somatic mutation and affinity maturation in response to microbial antigens.

C. They recognize pathogen associated molecular patterns, such as viral nucleic acids, bacterial cell wall constituents, and terminal mannose residues.

What is/are the uniquepart(s) or feature(s) of adaptive immunity? A. Non-reactivity to self B. Very large diversity of receptors as a result of them being germline encoded C. Very large diversity of receptors as a result of them being produced through somatic recombination of gene segments D. Both A and B E. Both A and C

C. Very large diversity of receptors as a result of them being produced through somatic recombination of gene segments

How many liters of mucus do goblet cells secrete daily, though most is resorbed in the large intestine? A.) 1 liter B.) 20 liters C.) 4 liters D.) 40 liters

C.) 4 liters

What are 2 common bacterial products that contain pathogen associated molecular patterns? A.) Cytoplasm B.) Lipopolysaccharide C.) A & B D.) none of the above

C.) A & B

What does C4a act on? A.) Phagocytic cells B.) Mast cells C.) A and B D.) None of the above

C.) A and B

Pathogen-associated molecular patterns: A.) Allow for B and T lymphocytes to recognize bacteria and destroy them B.) Are cysteine-rich peptides that are in channels in bacterial membranes C.) Are recognized by pattern recognition receptors of the innate immunity system D.) Closely resemble host cell surface proteins and sugars E.) Induce secretion of interferons by virally infecting host cells

C.) Are recognized by pattern recognition receptors of the innate immunity system

Which of the following initiates the Membrane Attack Complex? A.) C1q B.) C9 C.) C5 convertase D.) C3b E.) C1r

C.) C5 convertase

What are secreted from mucous membranes, and are cysteine riche peptides, and form channels in the cell membranes of bacteria, causing an influx of certain ions and bacterial death? A.) Opsonins B.) Mucous C.) Defensins D.) None of the above

C.) Defensins

What is the most common inherited B cell defect ? A.) IgM deficiency B.) IgE deficiency C.) IgA deficiency D.) IgG deficiency

C.) IgA deficiency

Natural killer cells lyse Epstein-Barr virus infected B cells with deficient MHC-1 expression. The NK receptors that initiate the lytic activity are: A.) Compliment receptors B.) Fc receptors C.) Killer activation receptors D.) Killer inhibition receptors E.) T cell receptors

C.) Killer activation receptors

These cells monitor MHC-1, and normally are displayed on the cells surfaces of all nucleated cells in the body. A.)Killer activation cells B.)Complement receptors C.)Killer inhibition cells D.)Toll-like receptors

C.) Killer inhibition cells

Antigen-presenting cells (APCs) perform which of the following functions in adaptive immune responses? A.Display major histocompatibility complex (MHC)-associated peptides on their cell surfaces for surveillance by B lymphocytes B.Initiate T cell responses by specifically recognizing and responding to foreign protein antigens C.Display MHC-associated peptides on their cell surfaces for surveillance by T lymphocytes D.Display polysaccharide antigens on their cell surfaces for surveillance by B lymphocytes E.Secrete peptides derived from protein antigens for binding to Tcell antigen receptors

C.Display MHC-associated peptides on their cell surfaces for surveillance by T lymphocytes

Which of the following statement best describes the nature of Lipopolysaccharide (LPS)? A.It is a viral product. B.It is a bacterial exotoxin. C.It is a bacterial endotoxin. D.It is a fungal protein.

C.It is a bacterial endotoxin.

Which type of T cell is most important for defense against extracellular bacteria and fungi? A.TH1 B.TH2 C.TH17 D.Cytotoxic T lymphocyte E.NKT cell

C.TH17

The following receptor can selectively recognize bacterial lipopolysaccharide (LPS) A.(Toll-Like-Receptor 1) TLR1 B.A cytokine receptor C.TLR4 D.A chemokine receptor.

C.TLR4

Hypersensitivity to penicillin and hypersensitivity to poison oak are both A.mediated by IgE antibody B.mediated by IgG and IgM antibody. C.initiated by haptens. D.initiated by Th-2 cells.

C.initiated by haptens.

Which of these is present in the plasma in the largest quantities?

C3

Describe three functions of the complement system.

C3b coated microbes are phagocytosed via type 1 complement receptor on phagocytes. The complement cascade leading to pore forming and lysis of bacteria, the MAC - membrane attack complex. The "a" cleavage products function as chemotactic agents for neutrophils.

Which cell act through their capacity to help B cell make antibodies?

CD-4 T cells

Which phenotypic marker does not belong to the helper T cells

CD16

HIV binds to

CD4

Serological pipette are marked with

Calibration lines in the tip TD or an etched band

You have been assigned the mission to design a professional antigen presenting cell. Which molecules and functions would you include in your cell? Motivate your choices by explaining how these molecules and functions cooperate with each other.

Capacity to do phagocytosis, receptors to facilitate uptake and endocytosis, i.e. complement receptors, Fc receptors. Capacity to recognize non-self, danger signal, via pattern recognition receptors such as toll like receptors. These activate the cells to: Upregulate/express chemokine receptor to home for LN, i.e. CCR7. Express/upregulate co-stimulatory receptors, i.e. CD80/86 (B7.1, B7.2) to allow activation of naïve T cells in LN. Upregulate receptors/molecules to decrease immune response after action, i.e. Fas and IDO.

Explain the pathology behind graft-versus-host disease. Include cells involved and how the disease is induced in your answer.

Cell and tissue damage from conditioning regimen. Antigen presenting cells from the host will present host, which have matched MHC but different minor histocompatibility antigens (peptides, due to allelic differences between individuals), and activate donor T- lymphocytes. The activated T-cells will attack donor cells due to minor antigen differences. Also mature donor T cells can react against minor epitopes of the host, i.e. in case of complete match of MHC, the donor T cells have been activated to be effector in the donor and react against minor antigens on host, in case of not complete match of MHC molecules mature donor T cells react against whole MHC complex of host cells.

You have participated in developing a vaccine against the new swine flu. You now want to check the robustness of the cellular memory response in vaccinated people against different sub-structures (or parts) of the virus. How would you set up this study? Include method of choice, relevant controls and reagents needed

Cellular immune response, i.e. T cell response. Would measure cytokine production with preferably EliSpot to be able to measure the frequency of cells responding to be able to tell which virus sub-structure would give the best response. T cells recognize peptide antigens on MHC molecules. Would use target cells expressing same MHC molecules as the effector cells tested to be MHC restricted, can use total peripheral blood mononuclear cells (PBMC) that contain both T cells and cells able to present antigen on MCH class I. Would add the different peptide antigens to the cells to allow binding of the peptides to the MHC molecules, and then add the cells to the EliSpot plates for over night incubation. Controls would be for negative control, no peptides, and positive control could be a very common antigen like EBV derived peptide, or even better, a T cell stimulus like antibody against CD3 that would stimulate all T cells. Reagents needed are EliSpot plates, PBMC, peptides, cytokine detection antibodies, substrate.

T-lymphocytes are for ——- and B-lymphocytes are for——

Cellular immunity, Humoral immunity

Endogenous proteins are paired with MHC ____ in the ______.

Class I, E.R.

What kind of shoes could be worn in a lab?

Closed toed shoes

The _____ had the most microbial numbers and diversity.

Colon

T

Combined Immunodeficiency Disease is due to a complete absence of ______ Cells

immunogen

Compound A is injected into a non-immune animal. Later antibodies to compound A are found in the animal's blood. Which of the following best describes compound A in the first sentence? Compound A is an ______________

A molecule has been isolated from an immune cell and you have been given the task to identify whether it is cytokine or chemokine. How would you do this? What are the characteristics which define cytokines and chemokines?

Cytokines are intra- and inter-cellular signaling molecules which can alter differentiation of cells, activate cells induce more cytokine production, proliferation, etc. Cytokine receptors are complex and often consist of several chains. Chemokines are primarily involved in inducing cell migration. Chemokine receptors are normally serpentine seven spanning transmembrane G-coupled receptors. Tests: Cytokine; add to cells, look for proliferation, differentiation, activation, cytokine production etc. E.g. for T cells, look for proliferation, CD25 upregulation, altered cytokine profiles. Chemokines; look for migration against a gradient with a variety of immune cells, addition of pertussis toxin to cultures to inhibit G-coupled receptors would inhibit migration.

!!!!Which one is counterpart of CD3? a) B7 b) CD19 c) BCR d) Ig-α /β e) missed answer;/

D

!!!Lymphokine- activated killer (LAK) cells are indistinguishable from a) B lymphocytes b) macrophages c) malignant somatic cells d) NK cells e) T lymphocytes

D

!!Which of the following molecules is expressed on the surface of activated Th1 cells and promotes macrophage activation? a) Interferon- y b) Interleukin (IL)- 10 c) IL-12 d) CD40L e) CD4o

D

CD3+CD4+CD8- describes: a) nk cells b) B cells c) macrophages d) T helper cells e) T cytotoxic cells

D

The usual sequence of events in the development of an effective immune response to a viral infection is a) interferon secretion, antibody synthesis, cellular immune response b) antibody synthesis, interferon secretion, cellular immune response c) interferon secretion, antibody synthesis, cellular immune response d) interferon secretion, cellular response, antibody synthesis e) antibody synthesis, cellular immune response, interferon secretion

D

elper T cells a)can activate macrophages b)can stimulate antibody production c)are CD3+CD4- d)answers a and b are correct e)answers a and c are correct

D

Type I interfenrons a) are IFNa and IFNB b)lower the expression of MHC molecules on APC c) activate Nk cell d) answers a and c are correct e) answers a and b and c are correct

D (B: increases the expression of MHC molecules on APCs. )

B cells a) belong to APCs b) recognize antigens by MHC class II c) present antigens to T cell d) a and c are correc e) a and b are correct

D (B: recognise antigens by their antibodies)

Double positive thymocytes a) undergo positive selection b) are CD4+ CD8+ c) are found in the medulla thymus d) answers a and b are correct e) answers a and b and c are correct

D (C: medulla cortex)

CD28 a) is expressed on T cell b) transmits activating signal to T cell c) is expressed on APCs d) answers a and b are correct e) answers a, b and c are correct

D (CD28 + B7)

Recombination signal sequences a) consist of conserved coding sequence of seven and nine nucleotides separated by 12 or 23bp spacer b) form signal joint during somatic recombination of Ig genes c) form coding joint during somatic recombination of Ig genes d) answers a and b are correct e) answers a and c are correct

D (Conserved heptamer (7 bp) and nonamer (9 bp) sequences, separated by 12- or 23-bp spacers)

pTa chain a) is expressed on thymocytes after positive selection b) is expression stimulates the production of CD4 and CD8 c) is expressed on pre-T cells d) answers b and c are correct e) answers and b and c are correct

D (a: Expressed before positive selection)

Viruses can inhibit the immune response by a) immunosupression of host b) inhibition of proinflammatory cytokines production c) increasing the expression of adhesion molecules on infencnted cells d) answer a and b are correct e) answer a b and c are correct

D (?)

How are immune responses initiated? A.) Receptor B.) Ligand C.) Antigen D.) Both A & B

D.) Both A & B

What are 3 main types of initial defenses? A.) Barriers B.) Immune response C.) Adaptive response D.) All of the above E.) None of the above

D.) All of the above

Before neutrophils are able to migrate across the endothelial cells into the inflamed tissue, what step(s) must occur? A. Selectins must slow the neutrophil to "rolling" B.Integrinson the neutrophil must engage ICAM on the endothelial cells causing the neutrophil to stop C. Neutrophils must detect the "scent" of fMLP and C5a D. All of the above steps must occur for neutrophils to cross the endothelium E.None of the above steps must occur. Neutrophils migrate randomly.

D. All of the above steps must occur for neutrophils to cross the endothelium

Which of the following best describes passive immunity? A. The seasonal flu shot causes antibodies to develop in the body in about two weeks. These antibodies provide protection against infection from that specific flu strain. B. A TB test is done by injecting a small amount of tuberculosis antigen under the top layer of skin in your forearm. If you have previously been exposed to the TB bacteria, your skill will react and a firm red bump will appear within 48 hours. C. The Varicella vaccine is an injection with a live attenuated virus so that the recipient can build immunity to the chickenpox. It stimulates the immune system so that memory cells are made so if there is an encounter with the disease, they body can produce the nessicary antibodies to remain healthy. D. Antivenom is a medication made from harvested antibodies that confer rapid immunity in the case of a bite from a highly poisonous animal. Antivenom, however, leaves the recipient with no memory. E. None of the above

D. Antivenom is a medication made from harvested antibodies that confer rapid immunity in the case of a bite from a highly poisonous animal. Antivenom, however, leaves the recipient with no memory.

The type of white blood cell that often arrives at the site of infection first, is a granulocyte, and contains granules that stain light purple is a _____________. A. Basophil B. Eosinophil C. Neutrophil D. Monocyte

D. Monocyte

Migration of leukocytes out of the blood into tissues mainly occurs in which type of vessel? A. Arteries B. Arterioles C. Capillaries D. Venules E. Veins

D. Venules

The immunoglobulin monitor contains _________ identical like chains and _____________identical heavy chains connected by disulfide bonds. A.) 2 and 1 B.) 1 and 2 C.) 3 and 1 D.) 2 and 2

D.) 2 and 2

What is true of the characteristics of IgM? A.) Found either as cell-surface bound monomer or as secreted pentamer B.) Most B cells display it on their cell surfaces C.) It's the first Ig to be formed following antigenic stimulation D.) All of the Above E.) None of the Above

D.) All of the Above

C3A does the following? A.) Increase phagocytosis B.) Activation of vascular endothelium C.) Mast cell degranulation D.) All of the above E.) None of the above

D.) All of the above

What are defensins? A.) Cells that cause a massive influx of T-Cells B.) Saccharides that activate a huge WBC influx C.) they elicit defense mechanisms from goblet cells D.) Peptides that form channels int he cell membrane of bacteria which cause the influx of certain ions and eventually cause bacterial death

D.) Peptides that form channels int he cell membrane of bacteria which cause the influx of certain ions and eventually cause bacterial death

All blood-borne cells ultimately derive from what? A.) White blood cells B.) Red blood cells C.) Plasma D.) Pluripotent hematopoietic stem cells

D.) Pluripotent hematopoietic stem cells

What types of cells are involved in the type IV (delayed, cell type) hypersensitivity reaction? A.) Eosinophils B.) Mast cells C.) Neutrophils D.) T cells

D.) T cells

During an early part of its development, the binding of a lymphocyte's antigen receptor to its specific epitope may result in the activation or death of that cell. Under these circumstances, the epitope in question would be described as a: A.) Carrier B.) Hapten C.) Immunogen D.) Tolerogen

D.) Tolerogen

Which of the following is an adhesion molecule? A.) Integrin B.) Selectin C.) Addhessin D.) all of the above

D.) all of the above

The principal mechanisms of innate immunity to extracellular bacteria are: A.Complement activation B.Phagoctyosis C.Inflammatory response D.All of the above E.None of the above

D.All of the above

The principal mechanisms of innate immunity to extracellular bacteria are: A.Complement activation B.Phagoctyosis C.Inflammatory response D.All of the above E.None of the above

D.All of the above

All of the following components can be part of innate immune responses EXCEPT A.natural killer cells B.alternate pathway of complement activation C.MAC D.B cells E.Macrophages

D.B cells

Among the genes that are associated with common autoimmune diseases, the strongest associations are with which of the following? A.Cytokine genes B.Autophagy genes C.Apoptosis genes D.MHC genes E.Antigen receptor signaling genes

D.MHC genes

Infectious microbes may contain antigens that cross-react with self antigens, so immune responses to the microbes may result in reactions against self antigens. This phenomenon is called _________. A.Defective tolerance B.Anergy C.Deletion D.Molecular mimicry E.Central tolerance

D.Molecular mimicry

Explain the terms dolor, rubor, calor in the context of inflammation.

Dolor - pain - caused by substances released by leucocytes or dead cells that triggers the nerves. Rubor - redness - caused by increased blood flow and vasodilation by mediator released by leucocytes during for example an infection. Calor - heat - caused by the vasodilation and the increased blood flow.

Which statement(s)concerning macrophages is/are correct? I. Macrophages are derived from megakaryocytes. II. Macrophages present antigens by MHC class I to CD4+ lymphocytes. III. Macrophages present antigens by MHC class I to CD8+ lymphocytes. IV. Macrophages present antigens by MHC class II to CD4+ lymphocytes. V. Macrophages circulate in blood up to 5 days and then migrate into tissues. a) IV, V b) I, II c) I, IV d) III, IV e) IV f) lack of correct answer.

E (I: derived from monocytes. V: Monocytes become macrophages in the tissues and perform their effector functions during the course of many days to weeks. Some macrophages may migrate into lymph nodes through draining lymphatics.)

The figure 1 below represents the changes in antibody titers of two subclasses of immunoglobulins (IgM and IgG) during the course of a primary and secondary infection with Dengue virus. Described the molecular mechanims that lead to the changes in antibody titers of the different immunoglobulin classes over time. What determines the switch to the specific IgG class? Where are B lymphocytes activated? Why are T lymphocytes require for B cell activation? Which are the three key events that occurs in the germinal center during the process of B cell activation Figure 1. 1°Ag represents the primary infection with the pathogen X, while 2° Ag indicates a subsequent secondary infection with the same pathogen.

During the intial phase of B cell activation, when the B cell encounters the cognate antigen for the first time, the cell is activated (with the help of the appropriate T helper lymphocyte), undergoes clonal expansion, and part of the progeny differentiates in plasma cells, which start to secrete IgM: the default immunoglobulin class produced (thus higher titers of IgM in the initial phase of the primary response). Part of the clonally expanded B cells instead will form the germinal center, where they undergo several processes, including class switch, producing a different class of antibody and production of memory B cells (in this case IgG). After the second encounter with the antigen, memory B cells will be immediately activated and induce a faster and stronger response, directly producing the "switched" antibody class for that specific antigen (IgG, thus explaining the higher titers of IgG during the secondary infection with a shorter period of lag time compared to the primary inefction).Cytokines regulate the type of class switch. B cells are activated in the B cell zone of the secondary lymphoid organs. A pre-activated T cell delivers two signals that are require for B cell acivation: CD40- CD40L interaction and secretion of IL4. The T helper step is require for the activation of the B cells only when it is strictly required, meaning the perceived presence of a danger (e.g. invading pathogen) that has previously induced activation of the T cell by dendritic cells. somatic hypermutation (random mutations within the re-arranged V region of the immunoglobulin genes); affinity maturation (selection of the B cell clone expressing Ab with higher affinity; class switch, induced by the interaction with the specific T cell

!!!! Choose the correct statement/s about superantigens. a) superantigens bind to the peptide binding groove on MHC molecule b) superantigens induce polyclonal activation of T cells c) superantigens may induce the exhaustion of T cells, which is due to increased expression of inhibitory receptors d) a and c are correct e) b and c are correct

E

Antigen binding groove in MHC class II molecules: a) binds lipids b) is built of α1 and β1 domains c) is formed only by α chain d) presents peptide antigens e) b and d are correct

E

Effector lymphocytes migrate to a) lymph nodes by arteries b) lymph nodes by afferent lymphatic vessels c) lymph nodes efferent lymphatic vessels d) tissues by afferent lymphatics vessels e) tissues by blood vessels

E

Immune response leads to prevent that tumor growth because: A. tumours are weakly immunogenic B. tumour are strongly immunogenic D. tumors can be recognised by NK cells D. tumours may suppress the immune response E. A and D are correct.

E

Mature B cell a) may enter lymph node b) has IgM on its surface c) has IgD on its surface d) answers b and c are correct e) answers a, b and c are correct

E

Naive T cell a) requires specific recognition of antigen by TCR to be activated b) requires IL-2 for proliferation c) requires costimulation to be activated d) answers a and c are correct e) answers a and b and c are correct

E

Negative selection of T cells occur in the: A. blood vessels B. bone marrow C. lymph node D. spleen E. thymus

E

Opsonins include a) C3a b) C3b c) IgG d) answers a b and c are correct e)answers b and c are correc

E

Plasma cell a) can not switch isotypes of Ig b) requires Blimp1 for differentiation c) can present antigens to T cells d) answers a and b are correct e) answers a b and c are correct

E

Process of somatic hypermutations a) may lead to apoptosis of B cell b) occure after contact with self antigen c) occure after contact with foreign anigen d) a and b are correct e) a and c are correct

E

Process of somatic hypermutations a) may lead to improved affinity of Ig to antigen b) may lead to disadvantageous affinity of Ig to antigen c) occurs in lymph node d) answers a and b are correct e) answers a, b and c are correct

E

Recombination signal sequences a) are recognized by RAG proteins b) code for protein c) are removed during somatic recombination d) answers a and b are correct e) answers a and c are correc

E

Recombination signal sequences a) consist of conserved coding sequence of seven and nine nucleotides b) are required for isotype switching c) form coding joint during somatic recombiation of Ig genes d) answers a b and c are correct e) lack of correct answer

E

T regulatory lymphocytes a) are CD4+ CD25 + cells b) produced IL-10 c)inhibit the action of autoreactive lymphocytes d) are part of peripheral tolerance e) a and b and d are correct

E

The mechanism that protects against viral infection include a) production of neutralizing antibodies b) production of specific cytoxic t cells c) action of Nk cells d) answers b and c are correct e) answers a b and c are correct

E

Which of the following antibody isotypes facilitate the classical pathway of the complement system a) IgA and IgD b) IgA and IgE c) IgA and IgM d) IgE and IgG e) IgG and IgM

E

Proteasome: a) cuts peptides inside of ER b)participates in pathway of antigen presentation by MHC class II molecules c) transports peptides into ER lumen d) is an enzymatic complex e) lack of correct answer

E (a: cuts peptides outside of ER b: no for MHC I . c: TAP transports it. d: no, organelles.)

Costimulatory signal a) is required for activation of naive T cells b) may lead to activation or inhibition of T cells c) can be mediated by CD28 and B7 molecules d) b and c are correct e) a b and c are correct

E (A: naive T cells do not require costimulation B: Co*Stimulation*. cannot inhibit. )

Dendritic cells: a) express pattern recognition receptors that stimulate them to produce cytokines b) express C-type lectin receptors that are involved in antigen presentation c) activate naive T lymphocytes only when fully mature d) are not able to cross-present antigens e) answers a, b and c are correct

E (D: They are able to cross-present. Dendritic cells have 2 dendritic receptors called: Dectin-1 (C-type lektin) Dectin-2. They are *pattern recognition receptors*. Both dectins will stimulate the secretion of *cytokines*.)

TGF-β: A. inhibits the proliferation and effector functions of T cells and the activation of macrophages. B. inhibits the proliferation and effector functions of T cells and the activation of macrophages. B. regulates the differentiation of functionally distinct subsets of T cells. C. stimulates production of IgA antibodies by inducing B cells to switch to this isotype. D. promotes tissue repair after local immune and inflammatory reactions subside. E. All of the above.

E. All of the above.

What is the smallest individually identifiable part of an antigen that is bound by a receptor? A.) Enzyme B.) Protein C.) Macrophage D.) Isotype E.) Epitope

E.) Epitope

Immature B cells (Fraction E) express ___ heavy chains plus kappa or delta chains on their surfaces. A.) Alpha B.) Beta C.) Delta D.) Gamma E.) Mu

E.) Mu

NK cells: A.Recognize ligands on infected cells and kill the host cell B.Recognize ligands on injured cells and kill the cell C.Respond to IL-12 produced by macrophages D.Secrete INF-gamma, which activates the macrophages to kill phagocytosed microbes. E.All of the above

E.All of the above

The first immunoglobulin heavy chain class to be expressed on the surface of a newly produced B-cell is: A.IgA B.IgD C.IgE D.IgG E.IgM

E.IgM

Which pair of molecules are found in cytotoxic T lymphocyte (CTL) granules and are important for CTL killing of target cells? A.Perforin and Fas ligand B.P-selectin and tumor necrosis factor C.Major basic protein and granzyme B D.C9 and interferon-γ E.Perforin and granzyme B

E.Perforin and granzyme B

Cytosolic DNA sensors are a large group of molecules that detect microbial dsDNA in the cytosol of infected cells, and situate innate responses. Most of these sensors signal though a pathway that includes which protein? A. Mitochondrial antiviral-signaling protein (MAVS) B. Cytochrome C C. RIG-like receptors ( RLRs) D. TLR9 E.Stimulator of interferon genes (STING)

E.Stimulator of interferon genes (STING)

Cytosolic DNA sensors are a large group of molecules that detect microbialdsDNA in the cytosol of infected cells, and situate innate responses. Most of thesesensors signal though a pathway that includes which protein? A.Mitochondrial antiviral-signaling protein (MAVS) B.Cytochrome C C.RIG-like receptors ( RLRs) D.TLR9 E.Stimulator of interferon genes (STING)

E.Stimulator of interferon genes (STING)

Clonal selection: A.begins with inflammation. B.occurs for all leukocytes C.occurs in response to self-antigens. D.leads to innate immunity E.leads to proliferation of antigen-specific lymphocytes F.all of the above G.none of the above

E.leads to proliferation of antigen-specific lymphocytes

You are involved in designing a vaccine trial, and you are interested in measuring theefficiency of the vaccine by analyzing the number of cytotoxic T cells that produces IL-2. Which method would you chose to measure number (percentage, fraction) of IL- 2 secreting cytotoxic T cells, how does the method work, and why would you chose this method?

ELISpot of sorted cytotoxic (CD8+) T cells (similar to the lab in immunology that you did). Or flow cytometry (FACS) of CD8 stained lymphocytes together with intracellular staining of IL-2, using fluorescent antibodies. ELISA not correct since it measures total production of excreted IL-2 and not number of/fraction of producing cells.

CD80 and CD86 are costimulatory molecules. What is costimulation and why is it important for T cell recognition of de novo infections? How can these molecules also suppress immune responses?

Expected answer: Costimulation is the necessary second signal needed to allow T cell activation. This is true both for CD4+ and CD8+ T cells. First signal comes from the specific recognition by the TcR of a peptide in association with MHC. Second signal via CD28 on the T cells that allow them to mature and become fully functional T cells, i.e. to be able to kill target cells expressing MHC class I molecules with the peptides the T cells have been activated to for CD8+ T cells, and to produce cytokines and cellular dependent help for CD4+ T cells. Only antigen presenting cells are expressing CD80/86, since these are the only cells allowed to activate T cells. This process is an important checkpoint to allow activation of T cells. If a T cell recognize MHC + peptide without engaging CD28, i.e. without the second signal, T cells are not allowed to mature and they instead become anergic. Over time after the T cell have matured, they upregulate another receptor for CD80/86, CTLA4, which act as an inhibitory receptor that counteract the activating signals from CD28.

Mention two pro-inflammatory and two anti-inflammatory cytokines, and shortly describe the function of respective cytokine, i.e. why they belong to respective group of cytokines.

Expected answer: Pro-inflammatory: e.g. TNF-alfa, IL-1, IL-6, IL-8, IFN plus function of the cytokines Anti-inflammatory: e.g. IL-10, TGF-beta, plus function of the cytokines

Which immunoglobulin can cross the placenta?

IgG

The immunoglobulins that activates complement the easiest or best is

IgM

Which kind of cytokine response would you like to generate against extracellular infections? Mention cell type involved and cytokines needed.

Extracellular infections, e.g. bacteria, usually evoke a humoral response, i.e. CD4+ TH2 cells produce IL-4 (and IL-5, IL-13) to activate B cells to proliferate and to switch to produce IgG4 and IgE. IFN- from TH1 induces B cells to make IgG2 (complement and opsonizing antibody). Also OK with activation of macrophages by INF- from NK and T-cells to increase the capacity to kill phagocytosed microorganisms by ROS and NO.

Central tolerance is connected with: a) anti-inflammatory cytokines production b) anergy induction c) lack of response to atypical antigens d) immune privilege e) Treg cells influence f) negative selection

F

T lymphocytes: a) could be distinguished within leukocyte population due to CD3 expression b) could be distinguished within leukocyte population due to CD25 expression c) do not produce cytokins d) could be stimulated by Treg cells e) are activated via signal induced by Lck phosphatase f) do not undergo maturation in generative lymphoid organs, but in thymus.

F

While B-lymphocytes originate and mature in the ________, T-lymphocytes precursors originate in the ________ and mature in the ________. A. Bone marrow, Thalamus, Thalamus B. Bone marrow, Thymus, Thymus C. Thymus, Bone marrow, Bone marrow D. Thalamus, Bone marrow, Bone marrow E. Bone marrow, Bone marrow, Thalamus F. Bone marrow, Bone marrow, Thymus

F. Bone marrow, Bone marrow, Thymus

An increase in TH cells the primary cause of the impaired humoral reaponse in older adults

False

FACS stands for Fluorescence-Activated Cell Surface

False

HLA class II antigens are expressed on all nucleated cells.

False

In a secondary response, antibody levels attain a lower titter

False

Lipopolysaccharide is an example of a tumor antigen

False

Mostly, currently licenced vaccines elicit good cell-mediated but poor humoral immunity.

False

Mouth pipetting is always acceptable in the clinical laboratory

False

Phagocytes are a part of your adaptive immunity

False

Toll-like receptors are examples of pattern recognition receptors.

False

heavy chains

Functional and biological properties of immunoglobulins such as binding to Fc receptors are associated with

Poison ivy leaves are coated in a mixture of chemicals including the small molecule urushiol. Ironically, on it's own, urushiol is harmless. However, when linked to larger proteins in the skin, it mounts an uncomfortable and itchy hypersensitivityreaction. Urushiol is best described as a:

Hapten

To deliver pipettes Have an etched band up top Need to be blown out Cannot be blown out

Have an etched band up top Need to be blown out

Type 1

Hay fever is what type of allergy?

What do we know about the etiology of rheumatoid arthritis? Please choose one risk factor OR a specific autoantibody and discuss from an immunological point of view which role it might play for the disease.

Here, we would like the students to mention both genetic and environmental risk factors, specifically HLA / MHC II genes (HLA-DRB1 shared epitope) and cigarette smoking; that more women than men are affected by RA (like in many other autoimmune diseases); that the majority of patients have autoantibodies to citrullinated proteins (ACPA / anti-CCP antibodies), that these autoantibodies are thought to drive disease, i.e. the chronic synovial inflammation.

What is the most abundant and fastest acting cytokine released by mast cells?

Histamine

1-2 weeks

How long is the lag period during a primary response?

Allergy is an exaggerated immune response that is manifested by what reaction?

Hypersensitivity

!!!!Which sentence/s about antigen recognition is/are correct? I) T lymphocytes recognize processed protein antigens and B lymphocytes recognize whole protein II) T lymphocytes and B lymphocytes require antigen presentation by antigen presenting cells III) Only T lymphocytes require antigen presentation IV) Naive B lymphocytes recognise antigens captures by T cells V) T lymphocytes recognize antigens which are presented to them by B lymphocytes VI) Receptors of B lymphocytes bind directly to antigens including proteins, lipid and sugars sentences I, III and VI are correct sentences III and VI are correct sentences III and IV are correct only sentence III is correct only sentence VI is correct

I, III, VI

1

If a virus infects a liver cell, virus antigens will be presented to T cells in the context of Class _____ HLA

IgE

If an individual who is highly allergic to cat dander is exposed to a pet cat in a friend's house which class of immunoglobulins would most likely be found to be elevated soon after this exposure?

Which antibody is an important protective substance in tears and saliva?

IgA

Which is the predominant antibody that is found in body secretions?

IgA

Which immunoglobulins function is still not completely known?

IgD

What differentiates a B-cell from a T-cell?

IgD is on the surface of B-Cells

!!!!J chain is characteristic for dimer of:

IgE

Which determinant is found on ALL animals of a species?

Isotype

Constant Region

Isotype differences are due to changes in the structure of which of the following?

During affinity maturation of antibody molecule a) variable region of both chain changes b) constant region of both chains changes c) constant regions of both chains remain the same d) a and b are correct e) a and c are correct

Jag tror D ( i boken står det mycket om ändring i V genes men ingenting om ändring i C)

When an allergy induces synthesis and release of mediators and the recruitment of eosinophils and Th2 lymphocytes, it is in the _______ phase.

Late

What is the second phase of the primary antibody response?

Log phase?

The major cellular component of acquired immunity is the

Lymphocytes

The major humoral components of acquired immunity are the

Lymphocytes

Antigens from cancer and virus cells (intracellular peptides) will be presented on:

MHC 1

All of the following have a direct role in mediating human immunodeficiency virus (HIV) entry into susceptible cells EXCEPT:

MHC class 1

Discuss why the MHC class II genes play a major role in autoimmune diseases

MHC class II is one of the most polymorphic genes in humans. The variability in the encoded protein is mainly in how the MHC class II molecules dock to the TCR and how the MHC class II molecules bind different peptides. Certain MHC class II molecules have a stronger association to autoimmune diseases, e.g. between HLA- DR4 and Rheumatoid arthritis. This is probably due to that they have higher specificity to peptides derived from self-proteins, and/or that they can present microbe derived peptides that is very similar to self-peptide and by this can activate CD4+ T cells to become autoreactive, i.e. via molecular mimicry. Another possible option is that specific MHC class II alleles are more prone to select for potentially autoreactive CD4+ T cells during thymic selection. A good answer should cover the first and second aspect.

Eosinophils along with NK cells have been described as the cytotoxic cells of the innate immune system. Describe the differences between these cells based on lineages and modes of killing.

NK cells are members of the lymphocytes and have the same cytotoxic machinery as T cells, with granules containing perforin and granzyme B. NK cells can be triggered by activating receptors specific for stress induced ligands on the target cells, and by down regulation of MHC class I on the targets. NK cells do also kill target cells via Fas/FasL and Trail/TrailL interactions. While eosinophils are members of the granulocytes of the myeloid lineage and are triggered by e.g. antibody coated targets via Fc receptors which leads to degranulation of toxic proteins (major basic protein), pore forming proteins (eosinophil cationic protein) and production of reactive oxygen species (via eosinophil peroxidase).

Your boss wants you to test what happens if you immunize mice with a self-protein expressed in the central nervous system (e.g. MOG) together with a strong adjuvant (e.g. complete Freund's adjuvance (CFA)) that contains ligands for Toll-like receptors. After a while the mice developed symptoms that resembled multiple sclerosis, with T cell infiltration of the brain. Give a plausible explanation why this happened, based on what you know about activation of T cells and T cell tolerance. Why were the mice not sick before vaccination?

MOG is a self-protein, and as such, T cells that express a T cell receptor specific for MOG should normally have been eliminated during T cell selection in the thymus, where T cells that recognize self-proteins are eliminated in the process called "negative selection". For negative selection to work, the protein (MOG) has to be expressed, processed and presented to thymocytes in the thymus. So, evidently, there are T cells specific for MOG that have escaped negative selection. A possible explanation for this is that MOG was not expressed in the thymus, why negative selection was not able to mediate the "central tolerance". The brain is an immunoprivileged site, so normally (in the absence of inflammation), T cells can' t migrate to the CNS. So the, the naïve T cells in the healthy (non-vaccinated mice) will not encounter APCs presenting MOG peptides on MHC class I/II. However, by vaccinating the mice together with an adjuvant, the APCs in the mouse gets activated, start expressing co-stimulatory molecules (CD80/86), and can present the antigen to T cells in the lymph nodes. This way, naïve T cells can become activated and migrate to the CNS, where the antigen is expressed. In addition, vaccination with a strong adjuvant like CFA induces inflammation, which can make the blood brain barrier more permeable. This way the activated T cells may get increased access to the brain, and can cause the symptoms of the mice.

Discuss why MHC class II has a strong association to autoimmune diseases. You can exemplify it with a specific autoimmune disease.

Many different answers possible, but the major thing I wanted to see is that you have an understanding for the role of the MHC class II molecule in the immune system - Binding self-peptides; Binding modified self antigens, -Interaction with T cells, Presenting peptides to CD4+ T cells, Leading to CD4+ T cell activation, expansion of antigen specific T cell clones -CD4+ T cell activation leading activation of B cells and autoantibody -CD4+ production, and/ or cytotoxic T cells -Molecular mimicry -In genetic analysis the MHC area shows the strongest association to several autoimmune diseases. If you had chosen RA you could have used citrullinated antigens as your example, with antiCCP antibodies. Also a discussion on that the MHC II can be the same in a healthy individual and an autoimmune patient would have generated points. We have few MHC class II alleles, thus each one must be able to bind many different antigens. Specificity of immune response lies in the very specific T cells, and B cells.

Select an autoimmune disease and discuss what role autoantibodies specific for the disease may play in the initiation or perpetuation of the disease.

Many possible answers depending on which disease was chosen, but a good answer should contain a discussion about the role of autoantibodies in terms of initiating or perpetuating the disease, e.g. formation of antibody/antigen complexes, activation of cells, inducing inflammation etc.

Give an example of an autoantigen and shortly explain why it is an autoantigen.

Many possible answers. The answer should countain a named autoantigen with a description of the function of the autoantigen.

Which of the following would not be present in blood?

Mast cells

Plasma cells are also known as

Mature B-Cells

Infectious microbes may contain antigens that cross-react with self antigens, so immune responses to the microbes may result in reactions against self antigens. This phenomenon is called _________.

Molecular mimicry

All are granulocytic cells EXCEPT:

Monocytes

Small

Most drugs are not good immunogens because they are too ___________

Chronic rejection in organ transplantation is not a "true" immunological reaction and can not be treated with immunosuppressive drugs. Explain how chronic rejection develops.

Multi factorial (scarring after acute rejection, ischemia, recurrence of underlying disease and toxicity from immunosuppressive drugs). Results in decrease organ function.

Explain in detail how NK cell activation is controlled.

NK cell function is regulated by both activating and inhibitory signals. Activation of NK cells is mediated via receptors that recognize: -stress inducedligands, e.g. NKG2G - MICs, Rae-1s, ULBPs -MHC class I molecules, e.g. activating KIRs in human and activating Ly-49s in mouse -the Fc part of antibodies, i.e. FcrγRIII (CD16). Mediates antibody dependent cellular cytotoxicity (ADCC). NK cells also need stimulation via cytokines to become activated, e.g. IL-2, IL-15, IL- 18 and IL-12. To balance the activating signals, there is inhibition via MHC class I specific inhibitory receptors, i.e. inhibitory KIRs in humans and inhibitory Ly-49s in mice and rats. In most circumstances the inhibitory signals override the activating signals. Reduced expression of MHC class I then becomes one key mechanism to activate NK cells.

NKT cells and gamma delta T cells are said to have both innate and adaptive immune functions. What does that mean in turns of function of these cells? Discuss from a perspective of receptor usage and compare this to the function of classical T cells.

NKT and gamma delta T cells have T cells receptors with very limited variation. NKT cells are restricted to the non-classical MHC molecule CD1d. Gamma delta cells recognize lipids and other non-protein molecules as antigens. Both NKT, gamma delta T cells and classical T cells require RAG enzymes to form their T cells receptors. The response of NKT and gamma delta T cells is quick since they are already activated after maturation, need no APC dependent activation or clonal expansion and are able to react directly upon antigen recognition, compared to classical T cells that takes days to be efficient since they need to be activated by APCs and clonally expand. Both NKT, gamma delta T cells and classical T cells express TLR receptors. • Eosinophils along with NK cells have been described as the cytotoxic cells of the innate immune system. Describe the differences between these cells based on lineages and modes of killing.

Describe the mechanisms by which naïve T cell are activated. Describe how you would interfere with these mechanisms to enhance or inhibit a specific immune response.

Naïve antigen specific T cells circulate between the blood and secondary lymphoidtissues, e.g. lymph nodes. During an infection, antigen presenting cells (APC), e.g. dendritic cells, pick up antigen (or are themselves infected). Certain structures, e.g. Toll!like receptor-ligands, are expressed by pathogens, and these can via pattern! recognition receptors, e.g. Toll!like receptors, activate the APCs. Activation of APC leads to upregulation of co!stimulatory molecules on the APCs, e.g. B7 (CD80/CD86), and the APC migrate to the LNs, where they will meet naïve T cells. To activate a naïve T cell, two signals are required. The first signal is mediated via the T cell receptor- MHC/peptide interaction. The T cell thus has to express a T cell receptorwhich is specific for the MHC/peptide complex that the activated APC presents. Thesecond signal is mediated via B7 (CD80/CD86), expressed on activated APCs, and CD28, expressed by naïve T cells. The interactions between TCR/MHC/peptide and B7/CD28leads to clustering of the TCR and co!stimulatory molecules on the cell surface of the T cells and recruitment of tyrosine kinases (Lck/Fyn) which phosphorylates the ITAM of the CD3e chain of the TCR/CD3 complex. This in turn leads to recruitment of ZAP70, which in the end leads to activation of certain transcription factors (NFkb, NFAT and AP1). Following activation, the T cells upregulate the IL!2 receptor alpha (CD25), and starts to produce IL!2. This autocrine loop results in a massive proliferation of the naïve T cell (clonal expansion), and differentiation into effector T cells. The differentiation from naïve to effector T cell involves changes in expression of a number of proteins in the Tcell, including decreased expression of CD28, CCR7 and CD62L, and increased expression of other adhesion molecules and chemokine receptors (e.g. LFA!1 and VLA4). Activated T cells also upregulate expression of CD40L, which can further activate the APCs via the CD40/CD40L interaction. The quality and nature of the T cell response isalso influenced by the cytokines in the environment where they are activated. Forexample, if naïve CD4+ T cells are activated in the presence of IL!12, they are likely to differentiate into a Th1 CD4+ T cell, whereas If they are activated in the presence of IL!4, they can differentiate into a Th2 CD4+ T cell. If the naïve T cell is only stimulated via the T cell receptor (signal 1), but is not receiving co!stimulation via B7/CD28 (signal 2), the T cell is rendered "anergic", i.e. it will not respond to subsequent stimulation, even in the presence of co!stimulation. This tolerance mechanism can be exploited to induce tolerance to antigens by vaccination to a specific antigen. One could for example envision making a vaccine where an antigen, e.g. a food antigen, is given in the absence of compounds that activate the APCs (i.e. no induction of co!stimulatory molecules) which will "anergize" the naïve T cells that might be specific for such a food antigen. Conversely, one could create a vaccine that includes a specific antigen that we want to generate a T cell response against by including adjuvants (helper compounds) that activate the APCs to express co!stimulatory molecules. Such adjuvants could for example be Toll!like receptor ligands. In addition, one could think of developing compounds that inhibits or block theB7/CD28 signals, and thus blocks the co!stimulation and inhibits the naïve T cell response. This could be antagonizing antibodies against CD28 or B7, or smallcompounds that inhibits the signaling via CD28. Similarly, one could envision interfering with trafficking of naïve T cells to the lymphnodes by e.g. blocking CCR7 or CD62L. This would however NOT per se inhibit a SPECIFIC antigen response. However, by combining administration of a specific antigen with blocking/anatagonizing B7 or CD28 one mightgenerate anergic T cells specific for the antigen one use in the vaccine. One could think of ways to help polarize a CD4+ T cell response to a Th1, Th2, Th17 or TReg response by modulating the cytokines (either by making cells produce them or byblocking the cytokines/cytokine receptors or the down stream signals). For example, inducing IL!12 production in the APCs during a vaccination with a specific antigen, could help in generating a Th1 type CD4 + T cell response.

Explain the role and functions of neutrophils in host defense

Neutrophil granulocytes, generally referred to as neutrophils, are the most abundant type of white blood cells in humans and form an essential part of the immune system. They form part of the polymorphonuclear cell family (PMN's) together with basophils and eosinophils. Neutrophils are normally found in the blood stream. However, during the beginning (acute) phase of inflammation, particularly as a result of bacterial infection, neutrophils migrate toward the site of inflammation, firstly through the blood vessels, then through interstitial tissue, following chemical signals (such as Interleukin-8 (IL- 8), Interferon-gamma (IFN- gamma), and C5a) in a process called chemotaxis. They are the predominant cells in pus, accounting for its whitish/yellowish appearance. Neutrophils are phagocytes, capable of ingesting microorganisms or particles. They can internalise and kill many microbes, each phagocytic event resulting in the formation of a phagosome into which reactive oxygen species and hydrolytic enzymes are secreted. The consumption of oxygen during the generation of reactive oxygen species has been termed the "respiratory burst," although it actually has nothing to do with respiration or energy production. The respiratory burst involves the activation of the enzyme NADPH oxidase, which produces large quantities of superoxide, a reactive oxygen species. Superoxide dismutates, spontaneously or through catalysis via enzymes known as superoxide dismutases (Cu/ZnSOD and MnSOD), to hydrogen peroxide, which is then converted to hypochlorous acid by enzyme myeloperoxidase. Degranulation Neutrophils also release an assortment of proteins in three types of granules by a process called degranulation:specific granules (or "secondary granules") Lactoferrin and Cathelicidin, azurophilic granules (or "primary granules") myeloperoxidase, bactericidal/permeability increasing protein (BPI), Defensins and the serine proteases neutrophil elastase and cathepsin G, tertiary granules cathepsin, gelatinase

The innate inmune system consist of

Neutrophils

All of the following can lead to false-positive errors in serology EXCEPT

Omitted reagent from mixture

What is the average concentration of CTCs in whole blood of cancer patient?

One in a billion

produce two Fab molecules and one Fc fragment

Papain digestion of IgG antibodies specific for antigen hen egg albumin (HEA) will

ELISpot is a sensitive method for analyzing immune responses towards e.g. viruses. You seed 100.000 peripheral blood mononuclear cells (PBMCs) from a patient into duplicate wells of an ELISpot- plate with anti-IFNγ antibodies immobilized onto the bottom of the wells. You stimulate the cells overnight with a peptide pool of a concentration of 0.1 μg/mL from one protein (pp65) of the cytomegalovirus (CMV) and a peptide pool of equal concentration from one protein (EBNA3) of the Epstein- Barr virus (EBV). IFNγ is secreted from the specific lymphocytes and is captured by the immobilized antibodies. On day 2 you add a secondary antibody towards IFNγ conjugated to an enzyme. A substrate is subsequently added which is cleaved by the enzyme on the secondary antibodies resulting in development of visible spots. In the duplicate wells where cells have been stimulated with CMV-peptides you count 500 and 700 spots, respectively. In the duplicate wells where cells have been stimulated with EBV-peptides you count 0 and 0 spots, respectively. Interpret these results.

Patient has cells specific for this particular CMV protein Quantification of the CMV-response: (500+700)/2 = 600!600/100.000 cells are specific (behöver inte räkna längre än så, men om de är duktiga på huvudräkning: 600/100.000*100 = 0,6%) No (IFNg-secreting) specific lymphocytes for peptides from this particular EBV- protein - does not mean that the patient has no immunity versus EBV, just no response for this particular protein.

Which of the following molecules is involved in the principal mechanism by which the adaptive immune system fights cancer?

Perforin

Only CD38 appears in which cell?

Plasma cell

Which of the following genes is responsible for the coding of reverse transcriptase?

Pol

Macrophages

Polymorphonuclear leukocytes (PMNs) include all of the following cell types except: -neutrophils -eosinophils -macrophages -basophils -all of the above

Hemolyzed specimens are frequently rejected mostly because they will falsely increase the levels of

Potassium

Describe two situations where formation, induction, of anti-HLA antibodies can take place.

Previous transplantations, previous blood transfusions, previous pregnancies.

Which of the following favors the development of immediate hypersensitivity?

Production of interleukin-4 in response to antigens

Karl is walking bare feet on the beach and suddenly cut his left foot with a shell. What are the first events happening within the first hour of the inflammatory process? Mention cells and cytokines involved and explain shortly what happens

Production of pro-inflammatory cytokines (IL-1, IL-6, IL-8 and TNF- ), and the effect of the cytokines, i.e.: -Vaso-dilatation and vaso-permeabilization (recruitment of the serum components such as complement and clotting system, and already formed antibodies)( TNF) -Upregulation of adhesion molecules on endothelial cells (selectins) (TNF) - Recruitment of neutrophils and mastcells (IL-8) -Systemic effects (fever, liver->acute phase proteins) (IL-1 and IL-6) (not necessarily within first hour)

Describe in detail how peptides are presented on MHC class I molecules. Include cellular compartments involved, how the peptides are processed and how the molecules end up on the cell surface. How do MHC class I-peptide complexes affect (i) T cells and (ii) NK cells?

Proteins are degraded in the cytosol by the proteasome. Part of the product are peptides. These peptides are transported into the ER by the TAP1/2 (transporter associated with antigen presentation) complex. The MHC class I heavy chains have been synthesized into the ER and are stabilized by chaperoning proteins, are associated with beta-2-microblobuline and with the peptides able to bind the specific MHC allele. The MHC class I - β2m - peptide complex is transported to the cell surface via the golgi apparatus. CD8+ T cells are selected on the MHC class I complexes in the thymus, where only the T cells that have rearranged T cell receptors that recognize the MHC class I complexes with correct affinity will survive, MHC restriction. Mature CD8+ T cells that have been allowed to be activated by APCs and to recognize a specific MHC class I - foreign peptide complex will be able to kill cells expressing these complexes. NK cells express activating and inhibitory receptors for MHC class I complexes. In contrast to T cells, the NK cell receptors are not as peptide specific as the TcR. NK cells are regulated by activating and inhibitory signals, where the major and most influencing/important inhibitory signal is via inhibitory NK cell receptors. A subpopulation of T cells can also express NK inhibitory MHC specific receptors and thus be regulated in an inhibitory way by MHC class I complexes.

Blood type as well as HLA type are barriers in organ transplantation. Even so, AB0- incompatible kidney transplantation has become a routine at Karolinska University hospital. Treatment before and after transplantation is given to achieve two specific effects. Mention the treatment and explain the two effects of the treatment.

Remove existing anti-A and/or anti-B antibodies, to prevent hyperacute rejection. Prevent rebound of anti-A and/or anti-B antibodies, to prevent acute rejection.

Precision is described as How close the test results are to the true value Reproducibility How close the test results are to one another

Reproducibility How close the test results are to one another

Briefly describe what SCID is (max 2 sentences) What sort of SCID do you think is most severe; X- linked SCID gamma common chain or MHC class II deficiency? Motivate your answer.

SCID, severe combined immunodeficiency, is a congenital (genetic) immunodeficiency that affects both T and B cells. X-linked common gamma chain SCID is the most abundant SCID. This is a mutation in a cytokine receptor component common for several cytokine receptors, i.e. IL-2, -4 -7, -9, -15 (do not need to know all the receptors) that lead to severe reduction of T and NK cells, and hence also B cell function. Affects mainly men, but also homozygous women. MHC class II deficiency leads to impaired CD4+ T cell selection and activation -> poor cellular and humoral adaptive immune responses. If well motivated answers, both can be considered to be more severe.

Breaking of tolerance is a central theme in explaining autoimmunity and autoimmune diseases. Please provide an example and explain how this mechanism can lead to an autoimmune disease.

Several different answers possible, e.g. based on molecular mimicry (infection with microbes that has motifes, peptides when degraded, that is very similar to self antigens), MHC class II alleles predominantly involved in autoimmunity, defect in AIRE (lead to lack of negative selection of T cells to antigens expressed in the periphery and to autoimmune disease APS-1), etc. Answer should contain understanding how/why the autoimmunity develops and an example disease where it happens.

What is an autoantigen? Please explain it in a way that a patient with rheumatoid arthritis without knowledge in immunology can understand it.

Should be possible to understand that an autoantigen is something from the patient that the patient is reacting on, that it is when the immune system reacts on its own body and that depending on what it reacts on it can give rise to different diseases. In the case of RA the immune system reacts to components in the joint and therefore induces an inflammation. Many ways of writing this is possible.

P-selectin

Sialyated Lewis X Sialylated Lewis X is a carbohydrate structure on glycoprotein ligands of E-selectin expressed by leukocytes. E-selectin binding to sialylated Lewis X mediates leukocyte rolling interactions with activated endothelium. The same is true of P-selectin.

E-selectin

Sialylated Lewis X Sialylated Lewis X is a carbohydrate structure on glycoprotein ligands of E-selectin expressed by leukocytes. E-selectin binding to sialylated Lewis X mediates leukocyte rolling interactions with activated endothelium. The same is true of P-selectin.

What signals are provided by the professional antigen presenting cells (e.g. dendritic cells) for activation of naïve T cells, and what is the function of these signals?

Signal 1: presentation of the specific antigen via the MHC complex (MHC class I or class II). This signal confers specificity to the T cells activationSignal 2: costimulation via the B7- CD28 interaction, which ensures that T cells are activated only when required Signal 3: polarization (in case of CD4+ T cells)

Describe three functions of antibodies. Include subclass and receptor and/or soluble molecule involved in the respective function in the answer.

Subclass IgG IgG IgM o IgG IgE (Antibody subtype etc.) Receptor FcRIIIγ on NK cells FcR on macrophages C3 (complement cascade) FcRε on mast cells neutralisation Function Killing of target cell Phagocytation Killing of bacteria (MAC) Activation of Mast cells, degranulation

What kind of lymphocytes is indicated by CD4 and CD8?

T lymph

Which type of T cell is most important for defense against extracellular bacteria and fungi?

TH17

IgG

Tetanus is a disease that caused by the presence of toxin. Which immunoglobulin is best at neutralizing this toxin?

Most hematopoietic stem cell transplantation are performed with the intention to cure hematological malignancies using the immunological graft-versus-leukemia effect. Which donor would you choose to have the strongest graft-versus-leukemia effect, an HLA-identical twin or an HLA- identical sibling?

The GVL-effect is stronger in the HLA-identical sibling donor situation. This is due to that in identical twins you would have no minor histocompatibility antigens, i.e. peptides, while you will have minor difference between siblings, giving the possibility for minor histocompatibility antigen (peptide) specific immune reactions and therefore a better GvL effect also.

What is the Rheumatoid factor and what does presence of Rheumatoid factor indicate?

The Rheumatoid factor is autoreactive antibodies (directed to the Fc part of IgG) that form immune complexes. Presence of Rheumatoid factor indicates rheumatoid arthritis (only present in appr 80% of cases though.) (Also present in appr 70% of cases of Sjögren' s syndrome).

Bacteria can vary their surface antigens.

True

Both helper and cytotoxic T cells recognize antigen that is presented in association with MHC molecules.

True

Eosinophils are responsible for suppressing inflammatory reactions

True

HIV retains infectivity for more than 3 days in dried specimens at room temperatures

True

What are the differences between MHC class I and II antigen presenting pathways?

The answer should contain: MHC class I molecules present antigens that comes from intracellular proteins that have been processed/degraded by the proteasome to peptides that are transported into the ER via TAP where they are bound to the heavy chains of MHC class I that has been synthesized into the ER and associated with beta 2 microglobuline. The MHC class I trimolecular complexes are then transported to the cell surface. MHC class II molecules heavy chains (α and β) are synthesized into the ER but do not bind peptides from the ER since the peptide binding cleft is blocked, protected, by the invariant chain. Instead the MHC class II molecules are transported in vesicles that fuse with phagosomes that contain proteins that has been taken up by the cells and been degraded in the phagolysozome. The invariant chain is degraded and the clip peptide is released from the MHC class II molecules and peptides from the proteins that has been taken up externally are then bound to the MHC II molecules and transported to the cell surface.

variable regions of the heavy and light chains

The idiotype of an antibody molecule is determined by the amino acid sequence of the

Mast cells have important roles in the development of allergies. This is due to all but which of the following reasons.

Their ability to produce IgE

!!!!Choose Correct sentence about NK cells:

Their activity is dependent on activating to supresing signals ratio.

IgM

This antibody is the best agglutinator of cells

IgA

This antibody is thought to bind bacteria and thereby prevent them from binding to you

Type 2

This hypersensitivity is caused by antibodies binding to cells and activating complement

IgD

This immunoglobulin has a regulatory role

IgA

This immunoglobulin protects the surface of the cornea

What are haptens?

Tiny molecules

The function of the immune system is to

To recognize self from nonself To defend the body against nonself

Before kidney transplantation HLA-typing is performed, but different HLA-types does not prevent transplantation between donor and recipient. Explain why HLA-match is not required and also analyse if there is a benefit in receiving a HLA-matched organ.

With the immunosuppressive drugs we have today, HLA is not a barrier for transplantation if the recipient doesn't have HLA-specific antibodies. However, a HLA- matched organ graft has a longer graft survival.

!!!Which of the following interactions leads to apoptosis of an infected/abnormal cell? a) FAS-FASL b) CD28-B7 c) CD40-CD40L d) CTLA4-B7 e) lack of correct answer

a

A circulating B cell that has never before encountered antigen expresses _____ on the cell surface: a. IgM and IgD b. IgG c. IgD d. IgM and IgG

a

A genetic defect in the gene coding for C3 of the complement pathway would be the most devastating of complement defects because: a. C3 is common to all complement pathways b. C3 is required for the removal of antigen antibody complexes c. C3a is cytotoxic for parasitic worms d. C3 is directly involved in lysis of susceptible pathogens.

a

A plasma cell secretes: a. antibody of a single specificity related to that on the surface of the parent B-cell b. Antibody of two antigen specificities c. The antigen it recognizes d. many different specificities of antibody e. lysozyme

a

A plasma cell that secretes IgA dimers is also producing: a. J-chain b. S-piece c. both d. neither

a

An important advantage of having two gene loci (κ and λ) for the light chain is a. that the likelihood of a successful rearrangement of light-chain genes increases. b. that immunoglobulins are homogeneous and not heterogeneous in mature B cells. c. that different effector functions are conferred by the two different light-chain loci. d. that surrogate light-chain transcription cannot compete with κ and λ transcription and enables B-cell development.

a

An important advantage of having two gene loci (κ and λ) for the light chain is a. that the likelihood of a successful rearrangement of light-chain genes increases. b. that immunoglobulins are homogeneous and not heterogeneous in mature B cells. c. that different effector functions are conferred by the two different light-chain loci. d. that surrogate light-chain transcription cannot compete with κ and λ transcription and enables B-cell development. e. all of the above.

a

Antibody light chain rearrangement occurs in the ____ stage of B cell development and rearrangement of the heavy chain occurs in the _____ stage. a. Pre-B, Pro-B b. Pro-B, Pre-B c. Double negative, Double positive d. immature B, mature B

a

Antigenic viral peptides produced within a virus-infected epithelial cell would be likely to be presented by which class of MHC? a) I b) II c) III D) A

a

B-1 B cells: a. secrete mostly IgM b. are found in the medulla of the thymus c. home to the skin first in the fetus d. give rise to memory B cells.

a

During a secondary response to infection, the stimulation of naïve B cells is suppressed. This is an advantage for most secondary responses to infection because: a. Antibodies from newly stimulated B cells would be of lower affinity b. Naïve B cells would produce only IgG antibody c. Naïve B cells do not produce IFN-gamma d. All of the above.

a

Fill in the sentence; Langerhans cells are ______ located in _____. a) immature dendritic cells, the skin B) mature dendritic cells, the skin c) macrophages, pancreas d) macrophages, the skin e) immature dendritic cells, pancreas

a

Fill in the sentence; peptide binding groove in MHC class II molecules binds _____ derived from ____. a) peptides, exogenous antigens b) intact proteins, extracellular environment c) intact proteins, intracellular environment d) lipids, lipoproteins e) peptides, endogenous antigens

a

Human breast milk will contain antibodies specific for microorganisms encountered in the gastrointestinal tract of the mother because: a. The progeny of T cells responding to antigens from the gut will home to mucosal epithelium b. Antigens are specifically carried from the gut to the mammary gland associated lymphoid tissue c. The microflora of the mammary glands contains microorganisms similar to those that reside in the gut d. None of the above

a

If a person had impaired negative selection in the thymus, this individual would have an increased likelihood of suffering from: a. autoimmunity b. increased virus infections c. a lack of antibody production d. cancer.

a

IgG: (choose one) a. is transferred across the placenta b. exists in a divalent form in mucosal epithelial tissues c. is mainly membrane bound (not secreted) d. is the predominant Ig in primary responses e. is involved in allergic responses

a

Inherited immunodeficiency diseases usually involve ________ whereas autoimmune diseases usually involve ________. a. a single defective gene, a combination of causes b. a combination of causes, a single defective gene c. females, males d. adults, children

a

Interferons activate NK cells and: a) make infected cells susceptible to NK cells b) kill viruses directly c) activate complement d) act as opsonins

a

Lipopolysaccharide is an example of a _____ antigen that is recognized by TLR4 when bound to _____. a. thymus-independent (TI)-1; LPS-binding protein b. thymus-dependent; MHC class II c. thymus-independent (TI)-2; B-cell co-receptor d. thymus-dependent; LPS-binding protein e. immune complex; iccosomes.

a

Lymphoid tissues in which B cells and T cells are produced: a) primary lymphoid tissues b) secondary lymphoid tissues c) thyroid d) thyroid and anterior pituitary

a

Many bacterial polysaccharides act as TI-2 antigens and can activate B cells without the need for T cell help. This is because: a. they cross-link antibodies on the B cell surface b. TLR-9 on the bacterial surface binds these polysaccharides c. activated macrophages help the B cells in the place of T cells d. none of the above

a

Secretory IgA is best described as _____. a. a non-inflammatory immunoglobulin that restricts the passage of antigens across mucosal surfaces b. a complement-activating immunoglobulin that causes destruction of invasive microflora through the membrane-attack complex c. an inflammatory immunoglobulin that stimulates the chemotaxis of neutrophils into mucosal surfaces d. a monomeric IgA that neutralizes antigen effectively at mucosal surfaces.

a

Septic shock results from the action of ____ on ____ ____. a. TNF-alpha, endothelial cells b. toxins, endothelial cells c. TNF-alpha, epithelial cells d. toxins, epithelial cells.

a

T cells are targeted directly against ______. a. Abnormal self cells b. bacteria c. protozoa d. viruses

a

T cells recognize epiptopes they have never before encountered by a) randomply generating enormous number of TCRs prior to antigenic encounter b) sampling the environment using phagocytosis and pinocytosis c) synthesizing immunoglobulins specific for a wide variety of epiptopes d) using genomically encoded specific for a wide variety of epiptoes e) answers a and d are correct

a

The T-cell co-receptor CD8 interacts with _______ bound to the surface of _______: a. MHC class I; virus-infected cells b. MHC class I; T cells c. MHC class II: antigen-presenting cells d. MHC class II: T cells

a

The antibody transported across mucosal epithelia is _____: a. IgA b. IgD c. IgE d. IgG

a

The cytokine which is a growth factor for activated mature T cells is: a. Il-2 b. Il-8 c. TNF d. GM-CSF

a

The enzyme that is involved in both isotype switching and somatic hypermutation is: a. cytodine deaminase b. aminopeptidase c. Tdt d. RAG-2

a

The major mechanisms of physical innate immunity in the respiratory tract include: a) cilia b) macrophages c) pepsin d) peyers patches

a

The presence of Fc receptors on natural killer cells allow these cells to play a major role in _____. a. antibody dependent cell-mediated cytotoxicity b. cognate interaction with B cells c. the activation of macrophages d. the removal of antigen antibody complexes

a

The process of isotype switching occurs during the differentiation of: a. B cells b. T cells c. both d. neither

a

The role of CD3 and zeta chains on T cells is to: a. transduce signals to the interior of the cell b. bind to antigen associated with MHC molecules c. bind to CD4 or CD8 d. facilitate antigen processing of antigens that bind to the surface of T cells

a

The role of the CD3 proteins and Zeta chain on the surface of the cell is to: a. Transduce signals to the interior of the T cell b. Bind to MHC molecules c. Bind to CD4 or CD8 molecules d. Facilitate antigen processing of antigens that bind to the surface of T cells

a

The spleen differs from other secondary lymphoid tissues because it provides an environment for responses to antigens entering from the: a. blood b. kidney c. lymph d. thymus.

a

What is the function of endoplasmic reticulum aminopeptidase? a. shortens peptides for MHC class I loading b. mediates membrane to secreted Ab switching c. stimulates the loss of CD4 or CD8 d. catalyzes the random insertion of nucleotides

a

When a naive T cell has an antigen-MHC specific interaction with an antigen presenting cell, the central-supramolecular activation complex (c-SMAC) on the surface of the T cell usually contains which of the following? a. the T cell receptor (TCR) and CD4 or CD8 b. integrins and cell adhesion molecules c. Ig-alpha and Ig-beta d. CD40 and CD41

a

Where do N nucleotides come from in the generation of diversity? a. Addition by the TdT enzyme b. Randomness in cutting by the endonuclease c. Satellite DNA d. Random cutting by the ligase

a

Which antibody isotype is found predominantly in mucosal secretions? a) IgA b) IgM c) IgE d) IgG e) IgD

a

Which cytokine induced IgE class switching? a) IL-4 b) IFN-Y c) TGF-B d) a, b and c are correct e) no correct answear

a

Which of the following autoimmune diseases could occur in a person with agammaglobulinemia? a. multiple sclerosis b. systemic lupus erythematosus c. myasthenia gravis d. Grave's disease

a

Which of the following complement components is an opsonin that binds to complement receptor 1 (CR1) on macrophages? a. C3b b. C3a c. Bb d. Ba

a

Which of the following does not describe defensins? a. are produced solely by macrophages b. contain a large proportion of arginine residues c. amphipathic, with hydrophobic and hydrophilic regions d. disrupt pathogen membranes by penetrating and disrupting their integrity.

a

Which of the following explains why the first baby born to a Rh- mother and a Rh+ father does not develop hemolytic disease of the newborn? a. The antibodies made by the Rh- mother during the first pregnancy are predominantly IgM and have low affinity for the rhesus antigen. b. Maternal macrophages in the placenta bind to anti-rhesus antibodies and prevent their transfer to the fetus. c. Hemolytic disease of the newborn is a T-cell-mediated disease and maternal T cells do not cross the placenta during pregnancy. d. The rhesus antigen is not immunogenic and does not stimulate an antibody response.

a

Which of the following gene segments encodes the isotype of antibody expressed as receptor on immature B cells? a) Cμ b) Cᵧ c) C𝜷 d) Cɑ e) C∈

a

Which of the following pairs is mismatched? a. B and T lymphocytes: innate immune response b. natural killer cell: kills virus-infected cells c. macrophage: phagocytosis and killing of microorganisms d. eosinophil: defense against parasites.

a

Which of the following statements regarding immunoglobulins is correct? a. Immunoglobulins make up five classes (or isotypes) called IgA, IgD, IgE, IgG, and IgM b. Regardless of their isotype, immunoglobulins all have the same effector function c. Antibodies consist of four identical heavy chains and four identical light chains d. Peptide bonds hold the heavy and light chains together e. The constant regions make up the antigen-binding site.

a

Discuss pros and cons with different kinds of donors for patients undergoing allogenic hematopoietic stem cell transplantation: a, HLA-identical monozygotic twin b, HLA-identical sibling c, Matched unrelated donor d, Cord blood unit

a) well matched, no risk for GVHD, less chance of graft-versus-leukemia effect (increased relapse rate), genetic profile same (risk for the same disease) b) best possible donor, ethical issues when children are sibling donors c) takes time to identify in the registry, good match is required, higher risk for GVHD d)low cell dose- manly used for children, more mismatches are acceptable, not possible to use the same donor for re-transplantation or additional cell therapy, readily available

Which of the following accurately completes this statement: "the function of _____ T cells is to make contact with ____ and _____"? (Select all that apply) a. CD8; virus-infected cells; kill virus-infected cells b. CD8; B cells; stimulate B cells to differentiate into plasma cells c. CD4; macrophages; enhance microbicidal powers of macrophages d. CD4; B cells; stimulate B cells to differentiate into plasma cells e. all of the above are accurate

a, c, d

Reasons complicating the development of vaccines to combat chronic diseases include _____. (select all that apply) a. evasion of the host's immune system by the pathogen b. the polymorphic diversity of MHC class I and class II molecules c. the generation of inappropriate immune responses that do not eradicate the pathogen d. survival of the infectious agent for long periods inside the host e. high mutation rates in the pathogen

a,b,c,d,e

Which of the following are characteristics of innate immunity? (select all that apply) a. inflammation b. recognition of the pathogen improves during the response c. fast response d. highly specific for a particular pathogen e. cytokine production

a,c,e

What percentage of the resting cardiac output perfuses the spleen? (slide 80) a. 5% b. 10% c. 25% d. 50%

a. 5% *30 minutes to screen all the blood in your body

If aberrant T cell escapes deletion in the thymus, what is a common fate of that cell? (slide 164) a. Activation induced cell death b. Becomes a regulatory T cell c. Converted to a Th0 cell in the tissue d. Source of autoimmune disease

a. Activation induced cell death

In which of the following situations are enzymes and chemicals from the dying cell neatly contained in vesicles, which are eaten and disposed of by macrophages? (slide 49) a. Apoptosis b. Necrosis c. Both d. Neither

a. Apoptosis

What T cells are activated by MHC I being presented on cells? (slide 5) a. CTLs b. regulatory T cells c. helper T cells d. natural killer cells

a. CTLs

What thymic cells are associated with positive selection test of T cells? (slide 142) a. Cortical thymic epithelial cells b. Follicular dendritic cells c. Thymic dendritic cells d. Medullary thymic epithelial cells

a. Cortical thymic epithelial cells

What is the function of clustering of B cell receptors? (slide 64) a. Easier to activate b. Harder to activate c. Cross presentation d. Promotes class switching

a. Easier to activate *the slide says, "FDCs make clustering of BCRs more likely. More clustering means the signal to the nucleus to active will happen." I assume it means to activate, not active.

Killer T cells require activation before they can function, but helper T cells do not. (slide 8) a. true b. false

b. false *Both helper and killer T cells require activation before they can function.

A Fas ligand attaching to a Fas protein on a target cell causes what result? (slide 45) a. Target cell suicide (apotosis) b. Respiratory burst of H2O2 (hydrogen peroxide) c. Cytokine production from the target cell d. Anergizes (neuters) the target cell

a. Target cell suicide (apotosis) *Virgin T cells are wired to be insensitive to ligation of their Fas proteins. Experienced T cells that have been activated and re-activated many times, their wiring changes so that they become increasingly more sensitive to ligation of their FAS proteins either stimulated by their own Fas ligand or other T cells Fas ligand. These exhausted T cells are target for either suicide or homicide (process known as activation induced cell death = AICD)

What additional layers of protection against rogue T cells fits the following description: naïve T cells circulate through the secondary lymphoid organs but are not allowed out into the tissues? (slide 159) a. Tolerance by ignorance b. Tolerance induction in secondary lymphoid organs c. Peripheral tolerance d. Tolerance due to activation-induced cell death

a. Tolerance by ignorance *this traffic pattern takes the naïve T cells to areas where they are likely to encounter their "cognate antigen" on APCs & be activated & helps to preserve tolerance of self antigen due to the fact that these naïve T cells are unlikely to encounter rare self antigens here (2o lymphoid organs) as the rare self antigens that these cells may react to would more likely be found in tissues that these cells are restricted from.

In humans about 500 billion lymphocytes circulate each day through the various secondary lymphoid organs. (slide 98) a. True b. False

a. True

What is the function of CD28 on the T cell when activated by B7 on APC? (slide 20) a. makes it easier for T cell to activate b. makes it harder for T cell to activate c. binds to the MHC on APC strengthening attachment d. downregulates the TCR

a. makes it easier for T cell to activate *CD28 is a receptor molecule on the T cell. When activated, it amplifies the signal and lowers the number of TCR crosslinks needed for activation.

All mature T cells will recognize antigen presented by MHC molecules. (slide 145) a. true b. false

a. true

All the B cells in the germinal center are identical (clones). (slide 65) a. true b. false

a. true

B cells that recognize self are given a chance to change known as receptor editing. (slide 171) a. true b. false

a. true

B7 binding to naïve T cell's CD28 causes co-stimulation and activation. (slide 111) a. true b. false

a. true

Bone marrow and thymus are primary lymphoid organs. (slide 54) a. true b. false

a. true

CTLA-4 binds all the B7 co-stimulatory molecule and blocks co-stimulation and activation. (slide 111) a. true b. false

a. true

Complete activation of the helper T cell takes about 4 to 10 hours. (slide 28) a. true b. false

a. true

Endothelial cells in blood vessels usually overlap like shingles and offer less room for the passage of cells. (slide 75) a. true b. false

a. true

High endothelial venule cells in blood vessels are columnar and offer more room for the passage of cells. (slide 76) a. true b. false

a. true

Immunological memory is the most important biological consequence of adaptive immunity. (slide 179) a. true b. false

a. true

Lymph nodes, spleen, and mucosal-associated lymphoid tissue (MALT) are secondary lymphoid organs. (slide 55) a. true b. false

a. true

MHC restriction or positive selection is responsible for about 95% of the double positive (DP) T cell deaths. (slide 141) a. true b. false

a. true

Medullary thymic epithelial cells (MTEC) play an important role by expressing tissue specific proteins (antigens) that are unique to different tissues/organs in the body. (slide 149) a. true b. false

a. true

Once an invader has been vanquished, more than 90% of the T cells which responded to the attack usually die off. (slide 113) a. true b. false

a. true

T cells only recognize peptides presented by MHC I or MHC II molecules (slide 13) a. true b. false

a. true

T cells that are double positive (DP) cells are the most common lymphocyte in the thymus (80%). (slide 139) a. true b. false

a. true

The combination of co-stimulation molecules will depend upon the pathogen and the area of the body (regional identity). (slide 22) a. true b. false

a. true

There are no afferent lymphatics bringing lymph to the spleen. (slide 80) a. true b. false

a. true

Thymic dendritic cells check to see if T cells recognize any of the self peptides displayed by MHC molecules on these cell's surface. (slide 148) a. true b. false

a. true

Which of the following explains why dendritic cells are more efficient than macrophages at stimulating naïve T cells? a. Macrophages do not express MHC class II molecules b. Dendritic cells are migratory and transport antigen to neighboring secondary lymphoid tissue c. Dendritic cells do not repair damaged tissues d. Macrophages do not process antigens e. Dendritic cells, but not macrophages, endocytose foreign antigen.

b

Which of the following is a central tolerance mechanism of B cells? a) antigen recognition without T cell help b) receptor editing c) anergy d) regulatory T cell-mediated suppression e) antigen recognition with costimulation

b

Which of the following is a property of plasma cells? a. Surface Ig b. Ig secretion c. growth d. isotype switching

b

Which of the following is mismatched? a. surface immunoglobulin: B cell antigen receptor b. affinity maturation: isotype switching c. constant region of antibodies: binding to complement proteins d. activation induced cytosine deaminase: somatic hypermutation e. switch sequences: class switching

b

Which of the following is mismatched? a. surface immunoglobulin: B cell antigen receptor b. affinity maturation: isotype switching c. constant region of antibodies: binding to complement proteins d. activation-induced cytidine deaminase: somatic hypermutation e. switch sequences: class switching

b

Which of the following statements is correct? a. Macrophages are granulocytes. b. Macrophages derive from monocytes. c. Macrophages are non-phagocytic. d. Macrophages bind IgE

b

Which of the following statements regarding neutrophils as false? a. Neutrophils are mobilized from the bone marrow to sites of infections when needed b. Neutrophils are active only under aerobic conditions c. Neutrophils are phagocytic d. Neutrophils form pus that is comprised of dead neutrophils

b

Which of the following statements regarding neutrophils is false? a. Neutrophils are mobilized from the bone marrow to sites of infection when needed b. Neutrophils are active only in aerobic conditions c. Neutrophils are phagocytic d. Neutrophils form pus which is comprised of dead neutrophils.

b

_____ is secreted into the bloodstream, whereas _____ is secreted into mucus such as gastrointestinal fluid, colostrum, saliva, tears, and sweat: a. Monomeric IgM: pentameric IgM b. Monomeric IgA: dimeric IgA c. Monomeric IgA: dimeric IgG d. Monomeric IgA: monomeric IgM e. Dimeric IgA: pentameric IgM.

b

________ is secreted into the bloodstream, whereas _____ is secreted into mucus such as gastrointestinal fluid, colostrum, saliva, tears, and sweat: a. Monomeric IgM: pentameric IgM b. Monomeric IgA: dimeric IgA c. Monomeric IgA: dimeric IgG d. Monomeric IgA: monomeric IgM e. Dimeric IgA: pentameric IgM

b

CD1 molecule: a) presents protein antigens b) presents lipid antigens c) is structurally related to MHC class II molecules d) answers a and c are correct e) answers b and c are correct

b (A: They only present lipid antigens. C: CD1 resemble MHC I proteins.)

CD28 binds all the B7 co-stimulatory molecule and blocks co-stimulation and activation. (slide 111) a. true b. false

b. false *CTLA-4 binds all the B7 co-stimulatory molecule and blocks co-stimulation and activation.

he tumor cells in a patient with leukemia are examined and found to express membrane-bound kIgM. This suggests that the original cell that underwent malignant transformation in this patient was most likely a: a) pre-B-cell b) naïve B-cell c) plasma cell d) memory B-cell Motivate your answer.

b), naïve B cells that have not met yet the antigen and did not undergo the germinal center process, including class switch.

During the course of an attack, T cells can change the class of antibodies they produce? (slide 191) a. True b. False

b. False *During the course of an attack, B cells can change the class of antibodies they produce?

In humans about 100 billion lymphocytes circulate each day through the various secondary lymphoid organs. (slide 98) a. True b. False

b. False *In humans about 500 billion lymphocytes circulate each day through the various secondary lymphoid organs.

What cells that are resident to the lymph nodes bind antigen and display that antigen to B cells? (slide 59) a. Th cells b. Follicular dendritic cells c. Medullary dendritic cells d. Resident activated macrophages

b. Follicular dendritic cells *Normal function is to catch & display opsonized antigen to B cells with high affinity receptors.

What Interleukin is produced by activated T helper cells, which promotes cell division (proliferation) of these activated T helper cells? (slide 29) a. IL1 b. IL2 c. IL5 d. TNF

b. IL2 *When activated helper T cells make more IL-2 it becomes a positive feedback for division. They are all specific to combat 1 particular antigen. In class he said, "another one that is not up there is IL-4."

What cytokine causes B cells to make IgA in the GI tract? (slide 95) a. IL4 b. IL5 c. IL10 d. IL13

b. IL5 *IgA is the most abundant cytokine in the body, and IgG is the most abundant cytokine in the blood

Which memory cells continue to function? (slide 192) a. Memory T cells b. Long lived plasma cells c. Both d. Neither

b. Long lived plasma cells

What cells/structures in Peyer's patches function to uptake antigen from the GI tract? (slide 88) a. Afferent lymphatics b. M cells c. High endothelial venules d. NK cells

b. M cells *on slide 91, it is stated that M cells are easily accessible to microorganisms in the intestine because they lack microvilli and mucus

The spleen holds a large number of what type of cell that can migrate to the battle site and become either a dendritic cell or a macrophage? (slide 85) a. Follicular dendritic cell b. Monocyte c. Eosinophil d. T cell

b. Monocyte

What is the positive test maturing T cells must pass in the thymus? (slide 141) a. Must not recognize self peptides b. Must only recognize peptides presented on MHCs c. Must recognize self peptides d. Must not recognize peptides presented on MHCs

b. Must only recognize peptides presented on MHCs

In which of the following situations are enzymes and chemicals from the dying cell released into the the surrounding tissue, which is extremely damaging? (slide 49) a. Apoptosis b. Necrosis c. Both d. Neither

b. Necrosis

Where are T cells likely to hang out in the spleen? (slide 81) a. Marginal sinuses b. Periarteriolar lymphocyte sheath (PALS) c. Between marginal sinus & PALS d. Splenic vein

b. Periarteriolar lymphocyte sheath (PALS)

The cytokines TNF, IFN-γ, & IL-2 are produced primarily by what T helper cells? (slide 35) a. Th0 b. Th1 c. Th2 d. Th17

b. Th1 *this response is known as the "classical cytokines"

Which Th bias is normally geared for bacterial infection from a wound? (slide 34) a. Th0 b. Th1 c. Th2 d. Th17

b. Th1 *this would also work for a viral infection.

What additional layers of protection against rogue T cells fits the following description: subset of thymic CD4+ T cells become "natural" regulatory T cells (nTregs) via expression of a gene called Foxp3? (slide 160) a. Tolerance by ignorance b. Tolerance induction in secondary lymphoid organs c. Peripheral tolerance d. Tolerance due to activation-induced cell death

b. Tolerance induction in secondary lymphoid organs *people who lack the Foxp3 protein tend to die young of autoimmune disease

All naïve T cells will recognize antigen presented by MHC molecules. (slide 145) a. true b. false

b. false *All mature T cells will recognize antigen presented by MHC molecules.

All the B cells in the germinal center are not identical (clones). (slide 65) a. true b. false

b. false *All the B cells in the germinal center are identical (clones).

T cells that recognize self are given a chance to change known as receptor editing. (slide 171) a. true b. false

b. false *B cells that recognize self are given a chance to change known as receptor editing.

B7 binding to naïve T cell's CTLA-4 causes co-stimulation and activation. (slide 111) a. true b. false

b. false *B7 binding to naïve T cell's CD28 causes co-stimulation and activation.

Bone marrow and thymus are secondary lymphoid organs. (slide 54) a. true b. false

b. false *Bone marrow and thymus are PRIMARY lymphoid organs.

What T cells keep the immune system from overreacting? (slide 5) a. CTLs b. regulatory T cells c. helper T cells d. natural killer cells

b. regulatory T cells

Non traditional T cells express what receptors. (slide 13) a. αβ b. γδ c. Εζ d. φχ

b. γδ *most abundant in the intestine, uterus, and tongue; less is known about nontraditional T cells than traditional T cells

CD3+ CD4+ CD8+ describes a) NK cells b) B cells c) macrophages d) T helper cells e) T cytotoxic cells

b?? (t-cyt: CD3+ CD4- cd8+ t-help: cd3 + cd4 + cd8 -

Graduated pipettes deliver liquid

between 2 calibration lines

!!!!!Which of the following statements about induction or maintaince of T cell tolerance is NOT true? a) central tolerance is induced when immature T cells in thymus bind to self antigen with too high affinity b) Peripheral T cell tolerance results when mature naive T cells recognize antigens without adequate B7-1 or B7-2 mediated costimulation c)Peripheral T cell tolerance is induced when T cells recognize antigen in the bone marrow d) Peripheral T cell intolerance to some antigens is induced when mature T cells recognize antigen and bind B7-1 or B7-2 via the inhibitory CTLA-4 receptor e) regulatory T lymphocytes are a subset of CD4+ T cells whose function is to suppress immune response and maintain self-tolerance

c

!!!!Which molecule expressed by T cells has the same function as Iga/Igb expressed by B cells? a) CD4 b) CD28 c) CD3 d) TCR e) integrin

c

!!!Fragments of a cytoplasmic pathogen are presented on T cells by? a) direct engagement of cells surface recognitions receptors b) endocytosis c) MHC class I molecules d) phagocytosis e) B cell receptor

c

!!!Which of the following enzymes is responsible for adding N nucleotide at junctions between gene segments? a) RAG-1 b) artemis c) TdT d) V(D)J recombinase e) exonuclease

c

!!!negative selection in bone marrow is associated with: a) mature T cells b) immature T cells c) immature B cells d) immature mast cells e) immature NK cells

c

A defect in CD40 leads to a lack of production of IgG, IgA and IgE because: a. CD40 is required for B cell entry into lymphoid follicles b. CD40 is required for proper bone marrow development c. CD40 is required for cell surface signals from helper T cells d. CD40 is required for the expression of RAG-1 and RAG-2.

c

An example of type III hypersensitivity is _____. a. contact dermatitis b. asthma c. serum sickness d. tuberculin reaction e. allergic rhinitis.

c

An individual does not make an immune response to a self protein because a) self proteins cannot be processed into peptides b) peptides from self- proteins cannot bind to MHC.... c) lymphocytes that express a receptor reactive a self protein are inactivated by deletion or anergy d) lymphocytes cannot rearrangegenes required ... a receptor for self protein e) a b and d are correct

c

Antagonistic autoantibodies made against insulin receptors cause _____. a. type 1 or insulin dependent diabetes b. hypoglycemia c. type II, or insulin-resistant diabetes d. hyperthyroidism.

c

Antibody light chain rearrangement occurs during which stage of B cell development? a. plasma cell b. mature naive B cell c. pre B cell d. pro B cell.

c

CD4 on the surface of T cells binds to: a. complement receptors b. Toll-like receptors c. MHC class II d. the TCR.

c

Endogenous proteins are cleaved in the cytoplasm by: a. the SyK enzyme b. Zap 70 c. the proteosome d. TAP-1

c

Fill in the sentence; Peptides generated in the cytoplasm are transferred to the _____, via the _____, and bind to newly formed____ in the lumen of the endoplasmic reticulum. a) endoplasmic reticulum, HLA-DM, MHCII b) golgi network, proteasome, MHCI c) endoplasmic reticulum, TAP transporter, MHCI d) golgi network, HLA-DM, MHC II e) MIIC compartment, TAP transporter, MHC II

c

The benefit for trypanosomes in maintaining over 1000 genes for surface glycoproteins when only on of these is expressed at given time is: a. These glycoproteins form complexes that block the attachment of eosinophils b. These glycoproteins resemble host proteins and mask the organism from recognition c. The organism can switch which glycoprotein is expressed once antibodies are formed d. These glycoproteins are negatively charged and repel phagocytes

c

The function of negative selection of thymocytes in the thymus is to eliminate: a. single-positive thymocytes b. Pro-T cell thymocytes c. self peptide specific thymocytes d. MHC reactive thymocytes

c

The immunoglobulin light-chain gene contains _______ segments, whereas the immunoglobulin heavy-chain gene contains _______ segments. a. κ; λ b. VDJ; VJ c. VJ; VDJ d. RAG-1; RAG-2

c

The most abundant complement protein in the blood is: a) C1 b) C2 c) C3 d) C5

c

The mutational mechanism that results in the production of antibodies that bind antigen with higher affinity is called _____: a. somatic recombination b. isotype switching c. somatic hypermutation d. clonal selection

c

The pro-inflammatory cytokines which can act on distant tissues/organs include Il-1, TNF-alpha and ______. a) Il-2 b) CCR3 c) Il-6 d) L-selectin

c

The process by which a pathogen stimulates only those lymphocytes with receptors specific for that pathogen is called: a. germline recombination b. somatic recombination c. clonal selection d. antigen processing e. antigen presentation

c

The process of _____ results in change in the constant region of the heavy-chain of antibodies, causing a change in the effector function and transport properties of antibodies: a. complement fixation b. neutralization c. isotype switching d. somatic hypermutation

c

The process of _____ results in the amplification of particular T cells or B cells with specificity for antigen: a. germline recombination b. somatic recombination c. clonal selection d. antigen processing

c

The process of gene rearrangement in immunoglobulin and T-cell receptor genes is called: a. somatic hypermutation b. isotype switching c. somatic recombination d. apoptosis e. clonal selection

c

The two classes of receptors on Natural Killer cells include the immunoglobulin-like and the ______ receptors. a) complement b) Fc-like c) lectin-like d) mannose

c

Toll-like receptors can exist: a. only on the surface of cells b. only in the cytoplasm of cells c. both on the surface and in the cytoplasm d. only in the nucleus of cells.

c

Tuberculoid leprosy is less severe than lepromatous leprosy because: a. the mycobacteria are less virulent b. there is an influx of eosinophils c. a TH1 type response is induced d. a TH2 type response is induced

c

What class of antibody is most abundant in the body (all locations combined)? a. IgM b. IgG c. IgA d. IgE

c

Which of the following does not apply to ....... immune mechanisms a) absence of specificity b) acid pH in the stomach c) a memory component d) involvement of phagocytes e) action of lyzozome

c

Which of the following explains why Streptococcus pneumoniae can infect an individual recurrently? a. Previous infection with S. pneumoniae wears down the immune system over time. b. S. pneumoniae is never completely eradicated during an infection and can reactivate if the host is immunocompromised. c. Immune responses against S. pneumoniae are serotype-specific and protect only against strains that possess the same capsular polysaccharide antigens. d. Anti-capsular antibodies are cleared from the host quickly after an active infection.

c

_____ autoantibodies enhance receptor function. a. Neutralizing b. Opsonizing c. Agonist d. Antagonist.

c

Adaptive immune response a) develops earlier then innate immune response b) recognizes common structures shared by related microbes c) generates memory cells d) a and b are correct e) a and c are correct

c (innate response goes with b)

As a naive B-cell differentiates into a plasma cell, the amount of DNA in its nucleus: a. increases b. decreases c. remains the same d. may or may not change depending on the isotype being secreted

d

Which of the following is a feature of regulatory T cells (Treg)? (Select all that apply) a. Treg express CD8 and control effector cells by inducing apoptosis b. Treg express high levels of CD25 (IL-2 receptor α chain) and secrete pro-inflammatory cytokines such as IFN-γ c. Physical association between Treg and its target cells is mandatory for Treg function. d. By interacting with dendritic cells in secondary lymphoid tissue, Treg prevent the interaction and activation of naïve T cells e. Treg secrete TGF-β and suppress effector T cell function

c, d, e

What percentage of the cisterna chyli drain into the thoracic duct? (slide 80) a. 25% b. 50% c. 75% d. 100%

c. 75% *I believe this was an added note

What is the function of CD8 in a killer T cell receptor? (slide 17) a. Binds cognant antigen bound to MHC b. Intracellular signaling c. Attaches the TCR to MHC d. Suppresses immune response

c. Attaches the TCR to MHC *specifically MHC I, and it signals the T cell, likely to "kill"

Where are B cells likely to hang out in the spleen? (slide 81) a. Marginal sinuses b. Periarteriolar lymphocyte sheath (PALS) c. Between marginal sinus & PALS d. Splenic vein

c. Between marginal sinus & PALS *lymphoid follicles also hang out here

Which of the following sends a deactivation signal to experienced T cells? (slide 111) a. Binding to B7 receptor on T cell b. Binding to CD28 receptor on T cell c. Binding to CTLA-4 receptor on T cell d. Binding to CD8 receptor on T cell

c. Binding to CTLA-4 receptor on T cell *CTLA-4 has a higher affinity than CD28, the only way B7 will bind to CD28 is if there is no CTLA-4

How do lymphocytes easily leave the blood vascular system and enter the lymph node? (slide 68) a. Capillary fenestrations b. High endothelial capillaries c. High endothelial venules d. Via the thoracic duct

c. High endothelial venules *on slide 73 it states, lymphocytes may enter the node via the arteriole or via incoming lymph

What interleukin from inducible regulator T cells (iTreg) binds to receptors on T cells and blocks co-stimulatory signals (CD 28) to make more difficult to activate and reduce proliferation? (slide 109) a. TGFβ b. TNFα c. IL-10 d. IL-17

c. IL-10 *the distinguishing factor between TGFβ & IL-10 is that TGFβ reduces CTL killing and IL-10 blocks co-stimulatory signals

What conditions would cause a T cell to commit suicide via apoptosis? (slide 10) a. If it fails the positive selection test in the thymus b. If it fails the negative selection test in the thymus c. If the T cell recognizes self but doesn't get co-stimulated d. if the T cell recognizes non-self and gets co-stimulated e. if the T cell recognizes non-self with no co-stimulated

c. If the T cell recognizes self but doesn't get co-stimulated

What is characteristic of the antigen that follicular dendritic cells catch and display to B cells? (slide 59) a. Its presented on MHC I b. Its presented on MHC II c. Its opsonized d. Its fragmented

c. Its opsonized *antibodies (don't kill) and complement (kill) both opsonize their antigen which is then displayed by the follicular dendritic cell (FDC)

What is common to all secondary lymphoid organs? (slide 57) a. Afferent lymphatics b. M cells c. Lymphoid follicles d. Bone marrow

c. Lymphoid follicles

Where are T cells likely to hang out in a lymph node? (slide 71) a. Marginal sinus b. Cortex c. Paracortex d. Medullary sinus

c. Paracortex

What additional layers of protection against rogue T cells fits the following description: naïve T cells are prevented from activating due to the lack of co-stimulatory molecules on these potential self antigen targets found on ordinary tissue cells? (slide 161) a. Tolerance by ignorance b. Tolerance induction in secondary lymphoid organs c. Peripheral tolerance d. Tolerance due to activation-induced cell death

c. Peripheral tolerance *in addition to being unlikely for it to be activated due to a lack of co-stimulatory signal and not enough antigen to cluster receptors (e.g. B7), if a T cell recognizes its cognate antigen but does not receive the co-stimulatory signal it is neutered (a.k.a. anergized), cannot function, and will likely die quickly

Which Th bias is normally geared for parasitic attack or food contaminated with pathogoenic bacteria (GI tract)? (slide 35) a. Th0 b. Th1 c. Th2 d. Th17

c. Th2

Peyer's patches specialize in making Th cells that are biased in what direction? (slide 94) a. Th0 b. Th1 c. Th2 d. Th17

c. Th2 *Th2 bias out of place renders you susceptible to asthma

People with allergies and asthma have what type of Th bias? (slide 95) a. Th0 b. Th1 c. Th2 d. Th17

c. Th2 *this isn't specifically stated anywhere in the notes, but he told us to write it, in class, while we were on slide 95

What is the function of high endothelial venules in lymph nodes? (slide 68) a. Prevents lymphocytes that are immature from entering the lymph node b. The way lymph normally exits the lymph node c. The way lymphocytes exit the blood and enter a lymph node d. The way blood enters a lymph node

c. The way lymphocytes exit the blood and enter a lymph node *antigens can come in on APCs or opsonized with antibodies and/or complement

What thymic cells are associated with negative selection test of T cells? (Pick all correct answers) [slide 147] a. Cortical thymic epithelial cells b. Follicular dendritic cells c. Thymic dendritic cells d. Medullary thymic epithelial cells

c. Thymic dendritic cells d. Medullary thymic epithelial cells

As a white blood cell, CTLs are created in which organ? (slide 4) a. spleen b. thyroid c. bone marrow d. thymus

c. bone marrow *white blood cell is created there, travels to thymus to mature

Immature T cells in the thymus only express which of the following on their cell surface. (slide 17) a. CD4 b. CD8 c. both d. neither

c. both *When T cells are in the thymus, they express both co-receptors. As they mature one type is down regulated. These molecules serve to strengthen the signal

What T cells are activated by MHC II on antigen presenting cells? (slide 5) a. CTLs b. regulatory T cells c. helper T cells d. natural killer cells

c. helper T cells

!!!Clonal anergy:

could be an effect of chronic HCV infection

!!!!Binding of this molecule to Jun is needed for transcriptional activation of the IL-2 gene. This is? a) CD-4 b) NFAT c) TCR d) Fos e) NF-kB

d

!!!!Which of the following induces the expression of endothelial selectins and Ig superfamily adhesions molecules, promoting effector T cells migration into inflammatory sites? a) P-selectin b) L-selection c) CCR7 d) Tumour necrosis factor (TNF) e)VCAM-1

d

!!!In persistent or chronic exposure to viral antigens a) T cell exhaustion occurs b) T cell start to express inhibitory receptors c) T cells differentiate into effector cells and clear the infection d) a and b are correct e) a and c are correct

d

!!!Prostate specific antigen (PSA) is a glycoprotein that is expressed by both normal and cancerous prostate epithelial cells. Choose the class of tumour antigen to which it belongs. a) product of cellular oncogene b) overexpressed unmutated cellular proteins c) oncofetal antigen d) tissue differentiation antigen e) product of and oncogenic virus

d

!!!The phenotype of NK cells is? a) CD3+CD56-CD16+ b) CD3+CD16+ c) CD3-CD56-CD16- d) CD3-CD56+CD16+ e) a and d are correct

d

!!!tumor antigen is: a) product of mutated suppressor gene b) tissue antigen c) product of mutated proto oncogene d) A and C is correct e) A,B,C is correct

d

A genetic defect in the gene coding for C9 of the complement pathway would be the least devastating of complement defects because: a. C9 is only used in the alternative pathway b. C9 is only required for the removal of antigen antibody complexes c. C9 is only cytotoxic for parasitic worms d. Lysis is the least important of complement effector functions

d

A major function of type 1 interferons released from virus infected cells is to: a. activate macrophages to secrete acute phase reactants b. stimulate isotype switching in T cells c. activate eosinophils by binding to Toll-like receptors d. stimulate surrounding cells to make antiviral proteins

d

A primary focus forms after a circulating naive B cell forms a conjugate pair with _____ in the _____ of a lymph node. a. TH1 cell; B-cell zone b. cytotoxic T cell; T-cell zone c. follicular dendritic cell; germinal center d. TH2 cell; T-cell zone e. CD40 ligand; T-cell zone.

d

Antibody heavy chain rearrangement occurs during which stage of B cell development? a. plasma cell b. mature naive B cell c. immature B cell d. pro B cell.

d

Place the following stages of B-cell development in the correct chronological order. a. Early pro-B cell b. Large pre-B cell c. Immature B cell d. Stem cell e. Late pre-B cell f. Small pre-B cell

d, a, e, b, f, c

Experienced T cell trafficking, which restricts them from certain areas, depends on what factor? (slide 100) a. Whether they find their cognant antigen b. Whether they are bound to B cells c. Whether they are reactivated d. Expression of certain adhesion molecules

d. Expression of certain adhesion molecules *the big idea with this slide is that naïve T cells have less restriction than experienced T cells

Which thymic cell practices autophagy and regularly enclose portions of their cytoplasm within a membrane and present these samples on MHC II molecules. (slide 152) a. Cortical thymic epithelial cells b. Follicular dendritic cells c. Thymic dendritic cells d. Medullary thymic epithelial cells

d. Medullary thymic epithelial cells

What is the function of linking up T cells? (slide ?) a. Easier to activate b. Harder to activate c. Cross presentation d. Promotes class switching

d. Promotes class switching *this was said in class, while going over another question

IL6 and TGFβ from an activated dendritic cell promotes what T helper bias? (slide 37) a. Th0 b. Th1 c. Th2 d. Th17

d. Th17

Which Th bias is normally geared for fungal attack and some extracellular bacteria? (slide 37) a. Th0 b. Th1 c. Th2 d. Th17

d. Th17

What additional layers of protection against rogue T cells fits the following description: the aberrant T cell will get activated over and over again because it is surrounded by antigen (self) and is killed via Fas ligand apoptosis? (slide 164) a. Tolerance by ignorance b. Tolerance induction in secondary lymphoid organs c. Peripheral tolerance d. Tolerance due to activation-induced cell death

d. Tolerance due to activation-induced cell death

The antigen-recognition site of T-cell receptors is formed by the association of which of the following domains? a. Vα and Cα b. Vβ and Cβ c. Cα and Cβ d. Vα and Cβ e. Vα and Vβ.

e

The peptide binding groove of MHC class I molecules is formed using parts of the following extracellular domains (the larger protein is designated alpha and the domains are ordered beginning with the most extracellular domain): a. α1:β b. β1:B2 c. α2:β2 d. α2:α3 e. α1:α2

e

The peptide-binding groove of MHC class I molecules is composed of the following extracellular domains: a. α1: β1 b. β1: β2 c. α2: β2 d. α2: α3 e. α1: α2.

e

Which of the following autoimmune diseases directly affect the nervous system? a. myasthenia gravis b. Graves' disease c. multiple sclerosis d. a and b e. a and c

e

What conditions would cause a T cell to be acitvated? (slide 10) a. If it fails the positive selection test in the thymus b. If it fails the negative selection test in the thymus c. If the T cell recognizes self but doesn't get co-stimulated d. if the T cell recognizes non-self and gets co-stimulated e. if the T cell recognizes non-self with no co-stimulated

d. if the T cell recognizes non-self and gets co-stimulated

CTLs mature in which organ? (slide 4) a. spleen b. thyroid c. bone marrow d. thymus

d. thymus

Vα of TCR is encoded by gene segments a) V b) D c) C d) a and c are correct e) a and b are correct

d?

!!!!Which of the following sentences about MHC molecules are correct? I) MHC haplotype is a set of MHC genes on one chromosome, inherited from one parent II ) There are three human class I genes, HLA-A, HLA-B and HLA-Cc, and each gene has many allelic forms III) B2-microglobulin is encoded by a gene located in MHC III region IV) peptide-binding groove in MHC class II molecules binds peptides derived from endogenous antigens V) Non-specific regions of mHC molecules interact with co-receptor of T-cell a) Sentences I and III are correct b) Sentences I,II and IV are correct c) Only sentences V is correct d) Sentences II, III and IV are correct e) Sentences I, II and V are correct

e

!!!In allergen immunotherapy the aim is to induce or restore tolerance to the allergen. Patients are desensitized through the administration of escaping doses of allergen that gradually _____ production. a) Increased IgE b) decreased IgE c) Decrease IgG4 d) increased IgG4 e) b and d are correct

e

A deficiency of complement component C6 will affect the: a. classical pathway b. alternate pathway c. MBL pathway d. a and b only e. a, b, and c

e

Cytokine receptors are associated with cytoplasmic protein kinases called _____, which become activated when the cytokine receptors bind to their respective cytokines. a. ZAP-70 b. STATs c. Lck d. ITAMS e. JAKs.

e

Double positive cells a) undegoe positive selection b) are CD4+ CD8+ CD3+ cells c) are CD4+ CD8+ CD3+ cells d) are CD4+ CD8+ CD3+ cells e) a and c are corerct

e

Heavy chain of immunoglobulin IgM is encoded by gene segment/s: a) VJ b) VDJκ c) Cμ d) answers a and c are correct e) answers b and c are correct

e

Hyper IgM syndrome: a) is characterized by decreased concentration of serum IgM and normal or elevated IgG b) is characterized by decreased concentration of serum IgG and normal or elevated IgM c) develops when lymph node lack germinal centers d) a and c are correct e) b and c are correct

e

IgE: (choose one) a. is transferred across the placenta b. exists in a divalent form in mucosal epithelial tissues c. is mainly membrane bound (not secreted) d. is the predominant Ig in primary responses e. is involved in allergic responses

e

Immune response fails to prevent the tumor growth because a) tumours are weakly immunogenic b) tumours are strongly immunogenic c) tumors can be recognized by NK cells d) tumors may supress the immune response e) a and d are correct

e

Immunological tolerance in the B-cell repertoire is called _____ tolerance when it develops in primary lymphoid organs, and _____ tolerance when it is induced outside the bone marrow. a. Primary; secondary b. Apoptotic; anergic c. Stromal; follicular d. Receptor-mediated; systemic e. Central; peripheral.

e

Mucosal epithelia of the gastrointestinal tract, eyes, nose, throat, the respiratory, urinary, and genital tracts, and the mammary glands are protected by a. IgG b. IgM c. IgE d. monomeric IgA e. dimeric IgA.

e

Naive lymphocytes a) express high levels of adhension molecules b) express low levels of adhension molecules c) pass through lymphocytes (skulle det kanskje stå "lymph node?" d) migrate into peripheral tissues e) b and c are correct

e

Plasma cells: a. have a thin layer of cytoplasm b. are derived from T-cells c. develop into B cells d. secrete large amounts of gamma interferon e. have a highly developed rough endoplasmic reticulum

e

Primary immun response a) is characterized by activation of memmory T cells b) is characterized by activation of memmory B cells c) a and b are correct d) none e) leads to generation of memory T cells

e

Recombination of V, D, and J Ig gene segments: a. only occurs in mature B-cells b. only occurs in light chains c. involves heptamer-spacer-heptamer flanking sequences d. does not occur until the mRNA stage e. involves RAG-1 and RAG-2 enzymes

e

Secondary antibody responses are better because: a. they provide defense against unrelated antigens b. the antibody can be made by both T and B cells c. complement fixing antibodies are made d. they do not require T-cell help e. they are greater and faster

e

Which mutation in which cytokine/chemokine receptor do you think gives most clinical symptoms and problems, CXCR1 or Gamma-common chain? Why do you think that? Why do you think that the other is not as crucial?

he answer should show an understanding that a defect in the chemokine receptor CXCR1 does not have as severe effect as a defect in the common cytokine receptor gamma-common chain that is chared by several chemokine receptors (IL-2, IL-4, IL-7, IL-9, IL-15 and IL-21). A defect in the chemokine receptor will lead to less good chemotaxis in some specific instances, while a defect in the gamma-common chain will lead to X-linked severe combined immunodeficiency since development, maturation, activation and function of T and NK cells will be severly impaired.

B lymphocytes (B cells)

humoral immunity is considered to be orchestrated by which of the following cell types? -B lymphocytes - T lymphocytes - Monocytes -Phagocytes -None of the above

Mutations in both allelic copies of this gene are required for malignant transformation.

p53

Described the three events that characterize B cell activation in the germinal center, and what are the main outcomes of these processes

somatic hypermutation (random mutations within the re-arranged V region of the immunoglobulin genes) affinity maturation (selection of the B cell clone expressing Ab with higher affinity. Role for the follicular dentritic cells as antigen "supplier")class switch, induced by the interaction with the specific T cell Somatic hypermutations and affinity maturation: Production and selection of antibody molecules which has better and higher affinity for that specific antigen. Class switch: production of the most suitable antibody subclass (isotype) to fight that particular type of infection

Fc

the ______ portion of the antibody is where the immune system binds

FAB

the _______ portion of the antibody binds the antigen

Alternate

the ___________ complement pathway is part of your innate immune system?

There are ________ of microbes in and on the human body. (closest number choice)

trillions

bone marrow

where in the body is the origin of B lymphocyte development?

Spleen and tonsils

which of the following are considered lymphoid organs? -spleen -thymus -bone marrow -tonsils

a pathogenic microbe is connected/coated with antibodies

which of the following best describes an example of opsonization?

Which of following statements is not true? A) A single T cell will typically express many different TCRs so that it can react with many different antigens. B) A typical antibody has two antigen binding sites. C) A secondary response results from activation of memory cells. D) Newly formed B cells and T cells are called naïve because they have not yet been exposed to an activating antigen.

A) A single T cell will typically express many different TCRs so that it can react with many different antigens.

Which of the following statements about kappa and lambda light chains is false? A) A typical B cell will express both types. B) They form one half of Fab. C) Their genes are located on different chromosomes. D) Their genes do not have D segments.

A) A typical B cell will express both types.

The term double negative thymocyte is used to describe absence of _______ and ________? A) CD4; CD8 B) CD2; CD34 C) TCR; CD3 D) Pre-TCR; CD3

A) CD4; CD8

Which of the following statements about helminthic parasite infections is not true? A) Helminths are obligate intracellular pathogens. B) Eosinophils can offer some protection. C) Helminths are worms that can be large enough to see without a microscope. D) They are typically associated with chronic infection.

A) Helminths are obligate intracellular pathogens.

How do B memory cells active in secondary immune responses differ from the naive B-cell population activated in a primary immune response? A) The antibody produced is likely of higher affinity in a secondary response. B) The memory cells express IgM. C) The memory cells no longer require T cell help. D) The frequency of antigen specific B cells is lower in a secondary response.

A) The antibody produced is likely of higher affinity in a secondary response.

What would you predict to occur in a knock-out mouse strain that has a deletion in the Fas ligand gene? A) The mice will be able to mount normal Tc responses against infected cells by using the perforin pathway. B) The mice will not be capable of mounting Tc responses. C) The mice will have a deficiency in their ability to fight viral infections. D) Both b. and c. are likely to be true

A) The mice will be able to mount normal Tc responses against infected cells by using the perforin pathway.

What would you predict to occur in a mutant mouse strain that does not express MHC class I? A) The mice will not form CD8 T cells. B) The mice will not be capable of rearranging TCR genes. C) The mice will have a deficiency in humoral immunity. D) All of the above are likely to be true

A) The mice will not form CD8 T cells.

What would you predict to occur in a knock-out mouse with a deletion in CD28? A) The mouse will exhibit immunodeficiency due to an inability to effectively activate T cells. B) The mouse will be unable to produce B cells. C) The mouse will not have any T cells. D) Antigen presenting cells will be unable to process antigens.

A) The mouse will exhibit immunodeficiency due to an inability to effectively activate T cells.

Which of the following statements about neutrophils is not true? A) They differentiate into macrophages when they enter infected tissues. B) They have a multi-lobed nucleus. C) They are phagocytes. D) They are a major component of pus. E) All of the above are true statements.

A) They differentiate into macrophages when they enter infected tissues.

Which of the following statements about M cells is false? A) They process and present antigens directly to T cells. B) They are an important component of MALT. C) They continuously sample antigens from the lumen. D) They are specialized epithelial cells that reside within mucosal tracts.

A) They process and present antigens directly to T cells.

RAG1/RAG2 proteins A) mediate V/J and V/D/J recombination in developing B cells. B) are expressed in activated mature B cells only. C) mediate isotype switching. D) Both a. and c. are true

A) mediate V/J and V/D/J recombination in developing B cells.

_____ is an example of an autoimmune disease in which physical trauma may provide exposure of lymphocytes to an otherwise sequestered antigen. A) multiple sclerosis B) rheumatoid arthritis C) type 1 diabetes D) myasthenia gravis E) lupus

A) multiple sclerosis

Which of the following is not a likely origin of tumor-specific antigens? A) overexpression of normal genes B) peptide antigens containing point mutations C) proteins modified by abnormal posttranslational processes D) viral proteins

A) overexpression of normal genes

Numerous vaccination procedures require multiple booster shots because A) repeated exposure to an antigen builds up a larger pool of antigen specific memory cells. B) this procedure enhances innate responses. C) antibodies will only be produced in a secondary response. D) this is the only way to activate NK cells.

A) repeated exposure to an antigen builds up a larger pool of antigen specific memory cells.

Deficiency in the production of AIRE results in _____. A) the development of multi-organ autoimmunity B) incomplete VDJ rearrangement of developing T cells C) defective negative selection of B-cells D) death in infancy E) insufficient expression of tissue-specific proteins in the bone marrow

A) the development of multi-organ autoimmunity

A 32 year old pregnant woman has been diagnosed as having a selective IgG deficiency. The pregnancy is carried to term uneventfully, and the infant is delivered at 38 weeks of gestation. Which situation regarding the clinical scenario is correct? A.)The infant will be deficient in IgM B.)The infant will have low levels of IgG C.)The infant will be deficient in the pentameric form of immunoglobulins D.)The infant will not receive normal amounts of IgA

B.)The infant will have low levels of IgG

Which of the following statements is false? A) GvHD is caused by mature donor T cells mounting an immune response against tissue of the recipient. B) Umbilical cord blood may potentially serve as an effective treatment for leukemia. C) Most immunosuppressive therapies are specific to self reactive T and B cells. D) Organ transplantations frequently involve donors and recipients that do not have the same HLA molecules.

C) Most immunosuppressive therapies are specific to self reactive T and B cells.

Which of the following statements about naïve B cells that have been released from the bone marrow is not true? A) They express both IgM and IgD. B) They must acquire access to secondary lymphoid tissue in order to survive beyond a few days. C) Since they have survived the negative selection process in the bone marrow, these B cells are guaranteed to be tolerant to self tissues. D) If appropriately activated they will differentiate into IgM secreting plasma cells.

C) Since they have survived the negative selection process in the bone marrow, these B cells are guaranteed to be tolerant to self tissues.

During infection with HIV, a person is said to undergo seroconversion when A) there is a significant drop in the number of CD4 Th cells. B) they succumb to opportunistic infections. C) anti-HIV antibodies are detectable in their blood. D) the virus goes into latency.

C) anti-HIV antibodies are detectable in their blood.

The mechanism responsible for production of recombinant influenza viruses composed of a genome derived from two different influenza variants is called ______________________. A) antigenic drift B) latency C) antigenic shift D) gene conversion E) immune evasion

C) antigenic shift

The reason Edward Jenner's smallpox vaccination strategy was successful is likely due to the fact that A) Jenner conducted a well controlled clinical trial. B) the boy who was treated with the vaccine was very young and thus responded particularly well. C) cowpox virus has molecular structures that are very similar to that of the smallpox virus. D) smallpox was not a serious threat at the time. E) All of the above.

C) cowpox virus has molecular structures that are very similar to that of the smallpox virus.

Which of the following is correctly paired? A) FK506: cytotoxic drug B) hydrocortisone: calcineurin inhibitor C) cyclosporin A: T cell activation inhibitor D) methotrexate: steroid based anti-inflammatory agent

C) cyclosporin A: T cell activation inhibitor

Which of the following is not considered to be a mechanism by which tumors can evade immune detection? A) reduction in levels of HLA class I B) recruitment of regulatory T cells C) enhancement of inflammatory responses D) lowering of levels of MIC on cell surface E) secretion of TGF- SHAPE \* MERGEFORMAT

C) enhancement of inflammatory responses

T lymphocytes A) produce antibodies. B) mature in the bone marrow along with all other cells of the blood. C) require that their corresponding antigens be broken down and presented in MHC molecules on the surface of APCs. D) are considered to be part of innate immunity. E) are phagocytes.

C) require that their corresponding antigens be broken down and presented in MHC molecules on the surface of APCs.

The classical pathway complement activation begins with which component? A.) C2A B.) C3 C.) C1 D.) C5

C.) C1

Lipid rafts are associations of transmembrane molecules together for effective cellular signaling function. T cell receptor (TCR) can be considered a lipid raft with many adapter molecules, signaling molecules, and antigen recognition proteins all associated with each other to transducer the signal intracellular. What protein is intimately associated with the TCR? A.)CD80 B.)CD86 C.)CD3 D.)CD28

C.) CD3

Which bacteria is not most commonly found in the upper respiratory tract? A.) Corynebacterium catarrhali B.) Neisseria meningitides C.) Enterococcus faecalis D.) Streptococcus spp. (alpha-hemolytic)

C.) Enterococcus faecalis

What is the thinnest physical barrier on our body? A.) Soles of feet B.) Palms of hands C.) Eyelids D.) Neck

C.) Eyelids

What must take place for Natural Killer cells to take action? A.) Hemorrhage B.) Headache C.) Inflammation D.) Heart attack

C.) Inflammation

Dendritic cells use an additional mechanism to sample large amounts of soluble molecules. This process is known as: A.) Cross linking B.) Pinocytosis C.) Micropinocytosis D.) Phagocytosis

C.) Micropinocytosis

Leukocytes expressing CD-8 receptors are: A.)Plasma B.)B-memory cells C.)T-cytotoxic cells D.)B lymphocytes

C.) T-cytotoxic cells

A 16-year old boy is injured while playing football. He does not remember the last time he had a tetanus shot, so the attending physician administers a tetanus toxoid vaccine. This kind of immunity would be best classified as: A.)Natural active B.)Natural passive C.)Artificial active D.)Acquired passive

C.)Artificial active

During acute, nonspecific inflammation, the first leucocytes to arrive at the site of tissue damage are: A.)Monocytes B.)Eosinophils C.)Neutrophils D.)Basophils

C.)Neutrophils

Gene rearrangement events A) occur in developing B cells. B) occur in developing T cells. C) are required for assembly of antibodies and T cell receptors. D) generate diversity in antibodies and TCRs. E) All of the above statements are true.

E) All of the above statements are true.

Which of the following statements about HIV is correct? A) HIV has a DNA genome. B) HIV infects cells that express CD4. C) The HIV genome is copied by reverse transcriptase. D) Antibodies can not bind to HIV. E) Both b. and c. are true.

E) Both b. and c. are true.

CD molecules A) are useful for distinguishing between different cell types. B) are typically cell surface molecules. C) typically function as adhesion molecules or as receptors for cellular signaling. D) All of the above statements about CD molecules are true.

D) All of the above statements about CD molecules are true.

What would you predict to occur in a knock-out mouse with a deletion in the VpreB component of the surrogate light chain? A) The mouse will have no circulating antibodies. B) The mouse will still be able to rearrange heavy chain genes. C) The bone marrow will accumulate B cell precursors that are stalled at the pro-B cell stage. D) All of the above.

D) All of the above.

Which of the following pairs of terms is least like the others? A) B7/CD28 B) B7/CTLA-4 C) CD2/LFA-3 D) CD4/Fyn E) CD4/MHCII

D) CD4/Fyn

_______________ migrate to secondary lymphoid tissues, whereas _______________ migrate to inflamed tissues. A) Memory B cells; dendritic cells. B) Dendritic cells; plasma cells. C) Neutrophils; plasma cells. D) Central memory T cells; effector memory T cells.

D) Central memory T cells; effector memory T cells.

You are attempting to measure the quantity of influenza virus that is circulating in a patient's blood. What type of immunological test would you be most likely to use to do this? A) Isolate the patient's white blood cells and use FACS to measure the influenza specific B and T lymphocytes. B) Run the blood proteins out on an SDS-PAGE and test for the presence of influenza antigens on a western blot. C) Set up a column containing anti-influenza antibodies conjugated to chromatography beads. Run the blood proteins through the column and then collect the proteins that have been bound to the antibodies. D) Dispense samples of the patient's blood proteins into designated wells in a plastic micro-titer dish alongside appropriate standards and controls. Use an antibody or antibody mixture that is specific for influenza antigens in an ELISA. E) Any of the above tests would be appropriate.

D) Dispense samples of the patient's blood proteins into designated wells in a plastic micro-titer dish alongside appropriate standards and controls. Use an antibody or antibody mixture that is specific for influenza antigens in an ELISA.

Which of the following statements about cytotoxic T cells is false? A) Naive Tc cells do not express IL-2 receptors. B) Naive Tc cells frequently require CD4 T cell help in order to become activated. C) Naive Tc cells can be stimulated by APCs that present appropriate peptide on MHCI and that also express B7. D) Naive Tc cells have lytic granules. E) All of the above are true statements.

D) Naive Tc cells have lytic granules.

People with a deficiency in the AIRE protein suffer from a broad spectrum autoimmune disease that involves immune attack against a diverse set of body tissues and organs. Which statement below most accurately provides a mechanistic explanation of this disease? A) These patients have a defect in MHCII expression which results in an inability to positively select for Th cells. B) These patients do not have a thymus gland and thus are unable to create T lymphocytes. C) These patients suffer from a defect in negative selection of immature B cells. D) These patients suffer from a defect in negative selection of thymocytes.

D) These patients suffer from a defect in negative selection of thymocytes.

Which of the following statements is true regarding adaptive immune responses? A) They are considered to be a first line of defense. B) They are directly activated by PAMPs. C) They are associated with NK cells and mast cells as the key effectors. D) They are associated with antigen specific activation and the generation of memory. E) None of the above.

D) They are associated with antigen specific activation and the generation of memory.

Which of the following is not a feature of Treg cells? A) Treg cells express CD4. B) Treg cells express IL-2 receptor (CD25). C) Treg cells suppress Th cell responses. D) Treg cell differentiation is driven by IL-12 and IFN- .

D) Treg cell differentiation is driven by IL-12 and IFN- .

The indirect protection of unvaccinated individuals as a consequence of sufficient prevalence of vaccination in the population in which they reside is known as__________. A) acquired immunity B) suppressive immunity C) indirect immunity D) herd immunity E) passive immunity

D) herd immunity

Defensins A) are part of the complement system. B) are secreted only by activated lymphocytes. C) activate NK cells. D) play a role in innate immune defenses in the gut. E) None of the above statements is true.

D) play a role in innate immune defenses in the gut.

Proteins not expressed on normal cells but found on tumor cells are called _____. A) cancer stem cell antigens B) oncogenic antigens C) tumor-associated antigens D) tumor-specific antigens E) proto-oncogene products

D) tumor-specific antigens

Which of the following is not associated with rheumatoid arthritis? A) joint inflammation B) antibodies against IgG C) anti-TNF treatment strategies D) type I hypersensitivity E) cigarettes and HLA-DR susceptibilities

D) type I hypersensitivity

People with a deficiency in the FoxP3 protein suffer from a broad spectrum autoimmune disease called IPEX. Which statement below provides a mechanistic explanation of this disease? A) These patients have a defect in central tolerance. B) These patients have a deficiency in regulatory T cells. C) These patients suffer from a defect in peripheral tolerance. D) These patients suffer from a defect in negative selection of thymocytes. E) Both b. and c. provide explanations.

E) Both b. and c. provide explanations.

Which of the following cells contain or release histamine? A.)Mast cells B.)Basophils C.)Platelets D.)All of the above

D.) All the above

Variable regions (in both heavy and light chains) are so named for their variation in amino acid sequences between immunoglobulin's synthesized by different what? A.) T Cells B.) Natural Killer Cells C.) Eosinophils D.) B Cells

D.) B Cells

KARs on NK cells allow them to recognize the presence of stress related molecules in humans called? A.) MICA B.) MICB C.) TLR D.) Both a and b

D.) Both a and b

Which type of cells use Killer activation receptors (KARs)? A.) B cells B.) T cells C.) Monocytes D.) Natural Killer Cells

D.) Natural Killer Cells

Which one of the following is a specific defense mechanism? A.)Skin B.)Mucous membrane C.)Tears D.)Cell mediated immunity

D.)Cell mediated immunity

Leucocytes expressing CD-4 receptors are: A.)Plasma cells B.)B-memory cells C.)T-cytotoxic cells D.)T-helper cells

D.)T-helper cells

Which of the following is true about HAART? A) It contains a cocktail of viral replication inhibitors. B) It is responsible for a drop in HIV associated mortality rates in populations that can afford it. C) It may be associated with severe side effects. D) It is not a cure. E) All of the above are true.

E) All of the above are true.

Which of the following is true about both T cell and B cell activation signaling pathways? A) They both involve phosphorylation of ITAMs on the cytoplasmic tails of cell surface proteins. B) They both involve cleavage of membrane phospholipids called PIP2. C) They both branch out into the same three parallel pathways. D) They both involve release of Calcium ions from the ER. E) All of the above are true.

E) All of the above are true.

Which of the following statements is true regarding the structure of a typical antibody? A) They are symmetrical. B) They have two light chains and two heavy chains connected by disulfide bridges. C) Fab domains bind to antigen. D) Fc domains are composed of heavy chain constant regions. E) All of the above are true.

E) All of the above are true.


Ensembles d'études connexes

Ch 4 Federal Privacy Protection and Consumer Identification Law

View Set

Supply Chain Chapter 9 & 10 Review Question

View Set

Women's health, the Menstrual cycle part 1

View Set

MSM 6610 Test question Chapter 6, 8-11

View Set

Cholinergic Agonists + Antagonists

View Set